13.08.2013 Views

RITE 2007 - Yale School of Medicine

RITE 2007 - Yale School of Medicine

RITE 2007 - Yale School of Medicine

SHOW MORE
SHOW LESS

You also want an ePaper? Increase the reach of your titles

YUMPU automatically turns print PDFs into web optimized ePapers that Google loves.

Anatomy<br />

Question 1: Anatomy - Peripheral/Autonomic Nervous Systems<br />

Discussion:<br />

Entrapment <strong>of</strong> the suprascapular nerve may cause shoulder pain, weakness <strong>of</strong> shoulder abduction, and sparing <strong>of</strong><br />

sensation about the shoulder.<br />

References:<br />

Devinsky O, Feldmann E. Examination <strong>of</strong> the cranial and peripheral nerves. New York: Churchill Livingston,<br />

1998.<br />

Question 2: Anatomy - Peripheral/Autonomic Nervous Systems<br />

Discussion:<br />

The tibialis posterior muscle is derived from the L5 and S1 myotomes, but it is supplied by the tibial nerve. An<br />

abnormality in this muscle would rule out a peroneal mononeuropathy at the fibular head. The peroneus longus<br />

and extensor hallicus longus could both be involved in an L5 radiculopathy or peroneal mononeuropathy and<br />

would therefore not distinguish the two. The gastrocnemius is an S1 muscle supplied by the tibial nerve, and the<br />

vastus lateralis is an L3 and L4 muscle supplied by the femoral nerve.<br />

References:<br />

Perotto A. Anatomical guide for the electromyographer. Springfield: Charles C. Thomas, 1994.<br />

Question 9: Anatomy - Cortex and Connections<br />

Discussion:<br />

Asomatognosia is a category <strong>of</strong> neglect in which the patient denies ownership <strong>of</strong> a limb contralateral to a lesion<br />

<strong>of</strong> the supramarginal gyrus <strong>of</strong> the parietal lobe (usually nondominant).<br />

References:<br />

Brazis PW, Masdeu JC, Biller J. Localization in clinical neurology. 4th ed. Philadelphia: Lippincott, Williams &<br />

Wilkins, 2001.<br />

Question 10: Anatomy - Neuromuscular Junction and Muscle<br />

Discussion:<br />

The radial nerve supplies the extensor pollicis longus and brevis, abductor pollicis longus, and extensor<br />

digitorum longus. The abductor pollicis brevis is innervated by the median nerve.<br />

References:<br />

No author. Aids to the examination <strong>of</strong> the peripheral nervous system. 2nd ed. London: Bailliere Tindall, 1986.<br />

Brazis PW, Masdeu JC, Biller J. Localization in clinical neurology. 4th ed. Philadelphia: Lippincott, Williams &<br />

Wilkins, 2001.<br />

Question 11: Anatomy - Spinal Cord<br />

Discussion:<br />

<strong>2007</strong> <strong>RITE</strong> Discussion & Reference Manual<br />

The patient has a dissociated sensory loss, with hypesthesias involving pain and temperature but sparing<br />

proprioception and vibration. The findings are most consistent with syringomyelia. A syrinx can involve the<br />

spinothalamic tracts bilaterally as they cross in the anterior white commissure that lies just anterior to the central<br />

canal. Lesions <strong>of</strong> the dorsal nerve roots would affect all sensory modalities. The fasciculus cuneatus and gracilis<br />

convey proprioceptive and vibration sensation, which are not involved in this patient. The lesion is at the level <strong>of</strong><br />

1


convey proprioceptive and vibration sensation, which are not involved in this patient. The lesion is at the level <strong>of</strong><br />

the cervical spinal cord so would not directly involve the thalamus.<br />

References:<br />

Campbell WW. DeJong’s the neurologic examination. Philadelphia: Lippincott, Williams & Wilkins, 2005.<br />

Question 27: Anatomy - Brainstem/Cerebellum<br />

Discussion:<br />

The findings are most consistent with a left dorsolateral medullary syndrome (Wallenberg syndrome) due to<br />

ischemia in the distribution <strong>of</strong> the posterior inferior cerebellar artery. Ipsilateral limb ataxia in this case is due to<br />

involvement <strong>of</strong> the left inferior cerebellar peduncle.<br />

References:<br />

Carpenter MB. Core text <strong>of</strong> neuroanatomy. Baltimore: William & Wilkins, 1991<br />

Question 38: Anatomy - Spinal Cord<br />

Discussion:<br />

Anesthesia over the perineal area ("saddle anesthesia") in conjuction with loss <strong>of</strong> sphincter tone and erectile<br />

dysfunction can be caused by a lesion <strong>of</strong> the lower cauda equina or the conus medularis. The region <strong>of</strong> sensory<br />

abnormality is in the S2 to S5 dermatomes. There is no weakness noted indicating that the lumbar and<br />

lumbosacral plexi are likely spared. Since this happend 3 weeks ago and his sphincter function remains flacid,<br />

this is most likely to be a peripheral (lower motor neuron) problem. These factors implicate the bilateral S2-S5<br />

roots as the most likely culprit. An intramedullary lesion occuring 3 weeks ago would be expected to have some<br />

component <strong>of</strong> spasticity at this point. A medial frontal lesion would explain the bladder incontinence but would<br />

not explain the sensory findings and there is no lower extremity weakness. Onuf's nucleus is a sphincteromotor<br />

nucleus located at S2 and S4 and would only explain part the patient's urinary symptoms. Given the sesory and<br />

bowel and bladder symptoms, this process is involving the distal rather than the proximal cauda equina. Since<br />

the more distal roots <strong>of</strong> the cauda equina are located more medially, a central disc herniation would be entirely<br />

consistent with this clinical vignette.<br />

References:<br />

Blumenfeld H. Neuroanatomy through clinical cases. 1st ed. Sunderland: Sinauer Associates, Inc., 2002.<br />

Question 39: Anatomy - Basal Ganglia and Thalamus<br />

Discussion:<br />

The anterior nucleus <strong>of</strong> the thalamus receives information from the mammilothalamic tract and hippocampus<br />

and sends its outputs to the cingulate gyrus. The dorsomedial (or mediodorsal) nucleus connects prefrontal,<br />

olfactory, and limbic cortex with prefrontal cortical regions. These last two nuclei are the only two <strong>of</strong> those<br />

listed that would be reasonably expected to affect multimodal attention and motivational drive. The ventral<br />

anterior and ventral lateral nuclei receive input from the basal ganglia and cerebellum, respectively, and send<br />

their outputs to the motor cortex. The ventral posterior medial and lateral nuclei receive primary somatosensory<br />

information from the face and body on its way to primary sensory cortex. The pulvinar and lateral posterior<br />

nuclei are both involved in visual processing and subcortical modulation <strong>of</strong> visual attention. The medial and<br />

lateral geniculate receive primary auditory and visual information, respectively.<br />

References:<br />

Murray S, Guillery R. Exploring the thalamus. San Diego: Academic Press, 2001.<br />

Question 41: Anatomy - Embryology<br />

Discussion:<br />

<strong>2007</strong> <strong>RITE</strong> Discussion & Reference Manual<br />

The preganglionic sympathetic neurons are located in the intermediolateral cell column in all thoracic and the<br />

2


upper two lumbar spinal segments. Thus, they are formed from the neural tube.<br />

References:<br />

Haines DE. Fundamental neuroscience. 2nd ed. New York: WB Saunders, 2002.<br />

Benarroch EE, Westmoreland BF, Daube JR, et al. Medical neurosciences--an approach to anatomy, pathology<br />

and physiology by systems and levels. 4th ed. New York: Lippincott, Williams & Wilkins, 1999.<br />

Question 45: Anatomy - Brainstem/Cerebellum<br />

Discussion:<br />

The patient with multiple sclerosis and "rhythmic clicking" in her ears has palatal myoclonus. A lesion <strong>of</strong> the<br />

pathway connecting the red nucleus, central tegmental tract, inferior olivary nucleus, and dentate nucleus<br />

(Guillain-Mollaret triangle) results in palatal myoclonus. Of the choices given in the question the only correct<br />

answer is dentate nucleus.<br />

References:<br />

Brazis PW, Masdeu JC, Biller J. Localization in clinical neurology. 4th ed. Philadelphia: Lippincott, Williams &<br />

Wilkins, 2001.<br />

Question 47: Anatomy - Blood Supply <strong>of</strong> Brain/Spinal Cord<br />

Discussion:<br />

This patient presents with a classic subcortical infarction located in or around the internal capsule and involving<br />

fibers in the posterior limb and the genu. The anterior choroidal artery, a branch <strong>of</strong> the internal carotid artery,<br />

supplies the posterior limb <strong>of</strong> the internal capsule and at least a portion <strong>of</strong> the genu. Hemorrhage from the<br />

internal carotid, anterior cerebral, or middle cerebral arteries would be expected to produce cortical findings,<br />

which are not seen in this patient. The posterior communicating artery connects the posterior cerebral artery to<br />

the internal carotid artery.<br />

References:<br />

Blumenfeld H. Neuroanatomy through clinical cases. 1st ed. Sunderland: Sinauer Associates, Inc., 2002.<br />

Question 56: Anatomy - Brainstem/Cerebellum<br />

Discussion:<br />

The patient has components <strong>of</strong> Parinaud's syndrome (conjugate upgaze paresis, nystagmus retractorious, and<br />

unreactive pupils). A lesion producing these findings occurs in the midbrain tectal region. It <strong>of</strong>ten occurs by<br />

extraaxial compression on the quadrigeminal plate (particularly the superior colliculi). Pineal region masses as<br />

well as obstructive hydrocephalus may also cause the syndrome.<br />

References:<br />

Gilman S, Newman S. Manter & Gatz's essentials <strong>of</strong> clinical neuroanatomy and neurophysiology. 7th ed.<br />

Philadelphia: FA Davis, 1987.<br />

Question 64: Anatomy - Cranial Nerves, Roots, and Plexus<br />

Discussion:<br />

A lesion distal to the geniculate ganglion and proximal to the stapedius muscle would cause weakness in the<br />

muscles <strong>of</strong> facial expression, alter sublingual and submandibular gland function, cause loss <strong>of</strong> taste to the<br />

anterior two thirds <strong>of</strong> the tongue, and cause hyperacusis but would spare lacrimation.<br />

References:<br />

<strong>2007</strong> <strong>RITE</strong> Discussion & Reference Manual<br />

Brazis PW, Masdeu JC, Biller J. Localization in clinical neurology. 3rd ed. Boston: Little, Brown and Co., 1996.<br />

3


Question 69: Anatomy - Brainstem/Cerebellum<br />

Discussion:<br />

A unilateral lesion <strong>of</strong> the ventrocaudal pons results in ipsilateral lateral rectus and facial paresis and a<br />

contralateral facial sparing hemiparesis. This is known as the Millard-Gubler syndrome.<br />

References:<br />

Haines DE. Fundamental Neuroscience. 2nd ed. New York: WB Saunders, 2002.<br />

Question 77: Anatomy - Cortex and Connections<br />

Discussion:<br />

The suprachiasmatic nucleus <strong>of</strong> the hypothalamus is the dominant circadian pacemaker <strong>of</strong> the mammalian brain.<br />

The intergeniculate leaflet and raphe nuclei mediate photic entrainment <strong>of</strong> the suprachiasmatic nucleus--light is<br />

the major entraining stimulus <strong>of</strong> the circadian system. The pineal gland is more important in seasonal rhythm<br />

control. The neurohypophysis does not have a major role in circadian pacemaking.<br />

References:<br />

Kandel ER, Schwartz JH, Jessel TM. Principles <strong>of</strong> neural science. 4th ed. New York: McGraw-Hill, 2000.<br />

Question 89: Anatomy - Cortex and Connections<br />

Discussion:<br />

The nucleus accumbens, a component <strong>of</strong> the basal ganglia, recieves extensive input from the limbic system and<br />

the orbit<strong>of</strong>rontal cortex. It is involved in anticipating rewards and is therefore implicated in substance abuse and<br />

addiction. Alzheimer's disease is a cortical dementia that does not primarily affect the basal gangila.<br />

Creutzfeldt-Jakob disease, a spongiform encephalopathy caused by prions, does not have a predilection for deep<br />

gray nuclei such as the nuclues accumbens. Epilepsy is a also a cortical process without basal ganglia<br />

involvement. Although Huntington's disease involves the basal ganglia, it is primarily striatal neurons <strong>of</strong> the<br />

caudate and putamen that degenerate.<br />

References:<br />

Schoenbaum G, Roesch M, Stalnaker T. Orbit<strong>of</strong>rontal cortex, decision-making and drug addiction. Trends<br />

Neurosci 2006 Feb;29(2):116-24.<br />

Blumenfeld H. Neuroanatomy through clinical cases. 1st ed. Sunderland: Sinauer Associates, Inc., 2002.<br />

Question 99: Anatomy - Cortex and Connections<br />

Discussion:<br />

The symptoms experienced by the patient in question would localize to the frontal lobe (behavioral and<br />

personality changes) as well as the temporal lobe (semantic memory). This would be a common presentation <strong>of</strong><br />

frontotemporal dementia. The posterior aspect <strong>of</strong> the superior temporal gyrus receives information from the<br />

primary auditory cortices and is integral to the network <strong>of</strong> areas which decode the meaning <strong>of</strong> words. This area,<br />

in the left lateral temporal cortex, is most likely responsible for the patient's semantic problems. The<br />

hippocampus is involved in memory and is part <strong>of</strong> the mesial temporal lobe. The orbit<strong>of</strong>rontal cortex is more<br />

likely involved in the patient's impulsive behavior and personality changes. The supplementary motor area is<br />

involved in complex motor programming.<br />

References:<br />

Kirshner H. Neurology in clinical practice, volume 1. In: Bradley W, Dar<strong>of</strong>f R, Fenichel G, Marsden C, editors.<br />

Language disorders: aphasia. 3rd ed. Boston: Butterworth Heinemann, 2000; 141-159.<br />

Question 125: Anatomy - Cortex and Connections<br />

Discussion:<br />

<strong>2007</strong> <strong>RITE</strong> Discussion & Reference Manual<br />

4


Prosody, the ability to produce and understand the emotional quality <strong>of</strong> speech, is processsed by the<br />

non-dominant hemisphere. Production <strong>of</strong> prosody depends upon nondominant dorsolateral frontal lobe while<br />

comprehension <strong>of</strong> prosody is a function <strong>of</strong> the nondominant temporal lobe.<br />

References:<br />

Mitchell R, Elliott R, Barry M, Cruttenden A, Woodruff P. The neural response to emotional prosody, as<br />

revealed by functional magnetic resonance imaging. Neuropsychologia 2003;41(10):1410-1421.<br />

Question 130: Anatomy - Cortex and Connections<br />

Discussion:<br />

The amygdala lies in the anterior pole <strong>of</strong> the temporal lobe just deep to the uncus and is an important structure in<br />

processing the emotional significance <strong>of</strong> stimuli, including pain. The caudate is the principal basal ganglia<br />

nucleus involved in processing oculomotor and prefrontal information. The nucleus acumbens is involved in<br />

anticipating reward and habit formation rather than the emotional component <strong>of</strong> pain. The hippocampus is part<br />

<strong>of</strong> the limbic system and is an essential component <strong>of</strong> memory formation. The pulvinar is an association nucleus<br />

in the thalamus involved in visual processing.<br />

References:<br />

Blumenfeld H. Neuroanatomy through clinical cases. 1st ed. Sunderland: Sinauer Associates, Inc., 2002.<br />

Nolte J. The Human Brain. 4th ed. St Louis: Mosby, 1999.<br />

Question 134: Anatomy - Cranial Nerves, Roots, and Plexus<br />

Discussion:<br />

The patient has neurological evidence to support a left abducens palsy and a left Horner’s syndrome. The sixth<br />

nerve and oculosympathetic fibers are close together in the cavernous sinus, near the internal carotid artery.<br />

Therefore, among the choices listed, the diagnosis that best explains his findings is a cavernous sinus aneurysm.<br />

A pontine glioma could produce similar symptoms, but one would also expect long tract signs. Vestibular<br />

shwannomas are usually in the cerebellopontine angle and can injure the seventh and eighth cranial nerves. An<br />

ecstatic basilar artery may compress the brainstem and injury multiple cranial nerves, but a Horner’s syndrome<br />

would be atypical. Similarly, tuberculous meningitis can cause a basilar meningitis and injure multiple cranial<br />

nerves, but the intraaxial location <strong>of</strong> the oculosympathetic fibers at that level would tend to be spared.<br />

References:<br />

Campbell WW. DeJong’s the neurologic examination. Philadelphia: Lippincott, Williams & Wilkins, 2005.<br />

Question 137: Anatomy - Cranial Nerves, Roots, and Plexus<br />

Discussion:<br />

Both the superior orbital fissure and the cavernous sinus contain cranial nerves III, IV, VI, V1, and sympathetic<br />

nerve fibers. V2 does not travel through the superior orbital fissure. A lesion in neither location would produce<br />

facial anhidrosis.<br />

References:<br />

Brazis PW, Masdeu JC, Biller J. Localization in clinical neurology. 4th ed. Philadelphia: Lippincott, Williams &<br />

Wilkins, 2001.<br />

Question 140: Anatomy - Peripheral/Autonomic Nervous Systems<br />

Discussion:<br />

<strong>2007</strong> <strong>RITE</strong> Discussion & Reference Manual<br />

The anterior interosseous nerve is a pure motor branch <strong>of</strong> the median nerve after it passes between the two<br />

heads <strong>of</strong> pronator teres. The anterior interosseous nerve innervates flexor pollicis longus, flexor digitorum<br />

pr<strong>of</strong>undis to the index and middle finger, and pronator quadratus. A lesion <strong>of</strong> this nerve impairs the ability <strong>of</strong> the<br />

5


patient to make an OK sign with the thumb and index finger producing instead a pinch attitude. There is mild<br />

weakness <strong>of</strong> forearm pronation and pain located in the proximal forearm.<br />

References:<br />

Brazis PW, Masdeu JC, Biller J. Localization in Clinical Neurology. Boston: Little, Brown, 1996; 12-14.<br />

Question 154: Anatomy - Cranial Nerves, Roots, and Plexus<br />

Discussion:<br />

The patient is presenting with a Bell’s palsy resulting in weakness <strong>of</strong> muscles supplied by the seventh cranial<br />

nerve. The platysmus is one <strong>of</strong> these muscles. The masseter is supplied by the trigeminal nerve, the levator<br />

palpebrae by the oculomotor nerve, the genioglossus by the hypoglossal nerve, and the stylopharyngeus by the<br />

glossopharyngeal nerve.<br />

References:<br />

Campbell WW. DeJong’s the neurologic examination. Philadelphia: Lippincott, Williams & Wilkins, 2005.<br />

Question 155: Anatomy - Neuromuscular Junction and Muscle<br />

Discussion:<br />

The tibialis anterior, which dorsiflexes and inverts the foot, is innervated by the deep peroneal nerve. The<br />

gastrocnemius plantar flexes the foot and is innervated by the tibial nerve (a branch <strong>of</strong> the sciatic nerve). The<br />

semitendinosus, one <strong>of</strong> the hamstring muscles, is innervated by the sciatic nerve. The tensor fasciae latae,<br />

innervated by the superior gluteal nerve, abducts and medially rotates the thigh. The sartorius muscle inwardly<br />

rotates the hip, and flexes the hip and knee and is innervated by the femoral nerve.<br />

References:<br />

No author. Aids to the examination <strong>of</strong> the peripheral nervous system. 2nd ed. London: Bailliere Tindall, 1986.<br />

Blumenfeld H. Neuroanatomy through clinical cases. 1st ed. Sunderland: Sinauer Associates, Inc., 2002.<br />

Question 157: Anatomy - Spinal Cord<br />

Discussion:<br />

A lesion <strong>of</strong> the accessory cuneate nucleus would spare pain and temperature. Cells <strong>of</strong> origin <strong>of</strong> the lateral<br />

spinothalamic tract are present in laminae I, IV, and V <strong>of</strong> the dorsal horn. They project to ventral posterolateral<br />

and intralaminar and posterior nuclei <strong>of</strong> the thalamus. Further projection to the cortex is to areas three, one, and<br />

two and to the secondary somatic sensory area.<br />

References:<br />

Parent A. Carpenter's human neuroanatomy. 9th ed. Baltimore: Williams & Wilkins, 1996.<br />

Question 158: Anatomy - Spinal Cord<br />

Discussion:<br />

On the side <strong>of</strong> a spinal cord hemisection there is an upper motor neuron syndrome, greatly impaired<br />

discriminatory tactile sense, loss <strong>of</strong> kinesthetic sense, and reduced muscle tone. Contralateral to the lesion there<br />

is loss <strong>of</strong> pain and temperature due to interruption <strong>of</strong> the ascending spinothalamic tracts.<br />

References:<br />

<strong>2007</strong> <strong>RITE</strong> Discussion & Reference Manual<br />

Brazis PW, Masdeu JC, Biller J. Localization in clinical neurology. 4th ed. Philadelphia: Lippincott, Williams &<br />

Wilkins, 2001.<br />

Question 166: Anatomy - Neuromuscular Junction and Muscle<br />

6


Discussion:<br />

The masseter, temporalis, medial and lateral pterygoids, tensor veli palati, tensor tympani, anterior belly <strong>of</strong> the<br />

digastric, and mylohyoid are innervated by the trigeminal nerve. The stapedius, buccinator, posterior belly <strong>of</strong> the<br />

digastric, frontalis, as well as other muscles <strong>of</strong> facial expression are all innervated by the facial nerve. The<br />

stylopharyngeus is innervated by the glossopharyngeal nerve.<br />

References:<br />

Brazis PW, Masdeu JC, Biller J. Localization in clinical neurology. 4th ed. Philadelphia: Lippincott, Williams &<br />

Wilkins, 2001.<br />

Question 195: Anatomy - Basal Ganglia and Thalamus<br />

Discussion:<br />

The globus pallidus externa (GPe) is a part <strong>of</strong> the indirect pathway through the basal ganglia that projects<br />

inhibitory fibers to the subthalamic nucleus. The GPe is not part <strong>of</strong> the direct pathway where fibers project<br />

directly from the striatum to the globus pallidus interna (GPi).<br />

References:<br />

Blumenfeld H. Neuroanatomy through clinical cases. 1st ed. Sunderland: Sinauer Associates, Inc., 2002.<br />

Question 331: Anatomy - Blood Supply <strong>of</strong> Brain/Spinal Cord<br />

Discussion:<br />

The lateral inferior or caudal pontine syndrome due to occlusion <strong>of</strong> the anterior inferior cerebellar artery (AICA<br />

syndrome) involves lesions in the fascicles <strong>of</strong> cranial nerve VII, the spinal tract, and nucleus <strong>of</strong> cranial nerve V,<br />

the lateral spinal thalamic tract, descending sympathetic fibers (lateral reticular nucleus), the middle cerebellar<br />

peduncle, the inferior surface <strong>of</strong> the cerebellum, and, in addition, the inner ear and cochlear nerve due to<br />

occlusion <strong>of</strong> the labyrinthine artery, a common branch <strong>of</strong> the AICA. Clinical findings include ipsilateral ataxia,<br />

loss <strong>of</strong> pain and temperature sensation <strong>of</strong> the face, Horner’s syndrome, deafness, and contralateral pain and<br />

temperature loss <strong>of</strong> the limbs.<br />

References:<br />

Campbell WW. DeJong’s the neurologic examination. Philadelphia: Lippincott, Williams & Wilkins, 2005.<br />

Question 341: Anatomy - Blood Supply <strong>of</strong> Brain/Spinal Cord<br />

Discussion:<br />

The recurrent artery <strong>of</strong> Heubner, a branch <strong>of</strong> the anterior cerebral artery, supplies the anteromedial part <strong>of</strong> the<br />

head <strong>of</strong> the caudate nucleus, adjacent parts <strong>of</strong> the internal capsule and putamen, and parts <strong>of</strong> the septal nuclei.<br />

References:<br />

Haines DE. Fundamental Neuroscience. 2nd ed. New York: WB Saunders, 2002.<br />

Question 349: Anatomy - Basal Ganglia and Thalamus<br />

Discussion:<br />

The caudate and the putamen serve as the primary input nuclei for the basal ganglia while the globus palidus,<br />

which projects to the ventral anterior nucleus <strong>of</strong> the thalamus, is the primary output nucleus. The substantia<br />

nigra pars compacta, located in the midbrain, sends dopamanergic fibers to the putamen. The subthalamic<br />

nucleus receives inhibitory input from the external part <strong>of</strong> the globus palidus and sends excitatory input to the<br />

globus palidus pars interna.<br />

References:<br />

<strong>2007</strong> <strong>RITE</strong> Discussion & Reference Manual<br />

Carpenter M, Sutin J. Human neuroanatomy. 8th ed. Baltimore: Williams and Wilkins, 1983.<br />

7


Blumenfeld H. Neuroanatomy through clinical cases. 1st ed. Sunderland: Sinauer Associates, Inc., 2002.<br />

Question 369: Anatomy - Peripheral/Autonomic Nervous Systems<br />

Discussion:<br />

The medial antebrachial cutaneous nerve is a branch <strong>of</strong> the medial cord <strong>of</strong> the brachial plexus and would be<br />

expected to be injured in the neurogenic thoracic outlet syndrome (TOS) and would be spared in an ulnar nerve<br />

mononeuropathy at the elbow (UNE). The dorsal ulnar cutaneous nerve may be abnormal in both neurogenic<br />

TOS or UNE. The sensory portions <strong>of</strong> the median nerve, the superficial radial nerve, and lateral antebrachial<br />

cutaneous nerve would be spared in UNE and neurogenic TOS.<br />

References:<br />

Campbell WW. DeJong’s the neurologic examination. Philadelphia: Lippincott, Williams & Wilkins, 2005.<br />

Question 373: Anatomy - Blood Supply <strong>of</strong> Brain/Spinal Cord<br />

Discussion:<br />

Contralateral hemianesthesia and hemiparesis followed by spontaneous pain in the affected limbs is due to<br />

involvement <strong>of</strong> the thalamoperforate branches <strong>of</strong> the posterior cerebral artery. Some <strong>of</strong> these branches supply<br />

portions <strong>of</strong> the posterior limb <strong>of</strong> the internal capsule and may produce contralateral hemiparesis in addition to<br />

the sensory changes and a central (thalamic) pain syndrome.<br />

References:<br />

Gilman S, Newman S. Manter & Gatz's essentials <strong>of</strong> clinical neuroanatomy and neurophysiology. 7th ed.<br />

Philadelphia: FA Davis, 1987.<br />

Question 378: Anatomy - Peripheral/Autonomic Nervous Systems<br />

Discussion:<br />

The patient has a light-near dissociation, which is not seen in disorders <strong>of</strong> the oculomotor or optic nerve. The<br />

absence <strong>of</strong> other neurological findings in this case would make a midbrain tectal or superior colliculus lesion<br />

unlikely. The slow, nonuniform constriction <strong>of</strong> the pupil is consistent with an Adie’s pupil that is due to an<br />

abnormality <strong>of</strong> the ciliary ganglion or short ciliary nerves.<br />

References:<br />

Campbell WW. DeJong’s the neurologic examination. Philadelphia: Lippincott, Williams & Wilkins, 2005.<br />

Question 390: Anatomy - Basal Ganglia and Thalamus<br />

Discussion:<br />

The paraventricular nucleus <strong>of</strong> the hypothalamus provides the bulk <strong>of</strong> the direct innervation <strong>of</strong> the preganglionic<br />

sympathetic neurons.<br />

References:<br />

Haines DE. Fundamental neuroscience. 2nd ed. New York: WB Saunders, 2002.<br />

Parent A. Carpenter's human neuroanatomy. 9th ed. Baltimore: Williams & Wilkins, 1996.<br />

Question 392: Anatomy - Basal Ganglia and Thalamus<br />

Discussion:<br />

<strong>2007</strong> <strong>RITE</strong> Discussion & Reference Manual<br />

The dorsomedial nucleus <strong>of</strong> the thalamus receives input from limbic structures and projects diffusely to the<br />

frontal cortex. As the main relay for information passing to the frontal association areas, it is felt to be the cause<br />

<strong>of</strong> the amnestic confabulation in Korsak<strong>of</strong>f's syndrome. The anterior nucleus, although it receives input from<br />

8


limbic structures, projects to the cingulate gyrus. The lateral posterior nucleus is involved in visual processing<br />

similar to the pulvinar. The ventral posteriolateral nucleus receives primary somatosensory information from the<br />

body. The reticular nucleus does not project to the cortex but instead regulates the activity <strong>of</strong> other thalamic<br />

nuclei.<br />

References:<br />

Carpenter M, Sutin J. Human neuroanatomy. 8th ed. Baltimore: Williams and Wilkins, 1983.<br />

Blumenfeld H. Neuroanatomy through clinical cases. 1st ed. Sunderland: Sinauer Associates, Inc., 2002.<br />

Question 397: Anatomy - Basal Ganglia and Thalamus<br />

Discussion:<br />

Fibers from the dentatorubrothalamic tract primarily synapse on the ventral lateral nucleus <strong>of</strong> the thalamus, but<br />

also on the ventral posterolateral. Fibers from these thalamic nuclei then project to the primary motor cortex<br />

(Brodmann area 4).<br />

References:<br />

Campbell WW. DeJong’s the neurologic examination. Philadelphia: Lippincott, Williams & Wilkins, 2005.<br />

Question 401: Anatomy - Cortex and Connections<br />

Discussion:<br />

The anterior commissure interconnects olfactory areas as well as homologous regions <strong>of</strong> the temporal lobe. The<br />

ansa lenticularis is an ipsilateral efferent pathway from the globus pallidus. The lateral olfactory stria arises in<br />

the olfactory bulb and projects to the ipsilateral prepyriform cortex and the amygdala. The median forebrain<br />

bundle projects to the hypothalamus and contains fibers from basal olfactory and periamygdaloid regions and<br />

the septal nuclei. The crus cerebri, on the ventral surface <strong>of</strong> the midbrain, contains the corticospinal and<br />

corticobulbar tracts. It does not connecct cortical regions.<br />

References:<br />

Carpenter M, Sutin J. Human neuroanatomy. 8th ed. Baltimore: Williams and Wilkins, 1983.<br />

Question 412: Anatomy - Cortex and Connections<br />

Discussion:<br />

Fifty patients with elevations <strong>of</strong> serum cardiac troponin levels had strokes involving the right posterior, superior<br />

medial insula, and the right inferior parietal lobule. Among patients with right middle cerebral artery strokes, the<br />

insular cortex was involved in 88% <strong>of</strong> patients with elevated serum cardiac troponin but in only 33% <strong>of</strong> patients<br />

without the elevation.<br />

References:<br />

Ay H, Koroshetz WJ, Benner T, et al. Neuroanatomic correlates <strong>of</strong> stroke-related myocardial injury. Neurology<br />

2006; 66: 13256.<br />

Question 419: Anatomy - Brainstem/Cerebellum<br />

Discussion:<br />

The patient has a crossed paresis with right arm and leg weakness as well as left facial paresis. This localizes to<br />

the left pons. The sensory deficits <strong>of</strong> fine touch on the right arm, trunk, and leg are due to involvement <strong>of</strong> the<br />

left medial lemniscus. The patient's diplopia with left lateral gaze is due to involvement <strong>of</strong> the left abducens<br />

nucleus.<br />

References:<br />

<strong>2007</strong> <strong>RITE</strong> Discussion & Reference Manual<br />

Brazis PW, Masdeu JC, Biller J. Localization in clinical neurology. 4th ed. Philadelphia: Lippincott, Williams &<br />

9


Wilkins, 2001.<br />

Question 421: Anatomy - Cortex and Connections<br />

Discussion:<br />

Damage to the fornix can occur with transcallosal surgery to remove a colloid cyst <strong>of</strong> the third ventricle, which<br />

interrupts Papez's circuit and results in loss <strong>of</strong> the ability to form new memories.<br />

References:<br />

Haines DE. Fundamental neuroscience. 2nd ed. New York: WB Saunders, 2002.<br />

Question 423: Anatomy - Spinal Cord<br />

Discussion:<br />

The dorsal and ventral spinocerebellar tracts are the most lateral tracts in the spinal cord and therefore would be<br />

expected to be affected first by an extrinsic lateral process. The lateral corticospinal tract lies just medial to the<br />

dorsal spinocerebellar tract while the anterior corticospinal tract is in the anterior midline. The fasciculus<br />

gracilis is the medial aspect <strong>of</strong> the dorsal columns. The tectospinal pathway lies just anterior to the anterior<br />

commisure and the reticulospinal tract lies just anterior to the lateral corticospinal tract. The spinothalamic tracts<br />

run just interal to the ventral spinocerebellar tract.<br />

References:<br />

Blumenfeld H. Neuroanatomy through clinical cases. 1st ed. Sunderland: Sinauer Associates, Inc., 2002.<br />

Nolte J. The Human Brain. 4th ed. St Louis: Mosby, 1999.<br />

Question 430: Anatomy - Cranial Nerves, Roots, and Plexus<br />

Discussion:<br />

The latissimus dorsi muscle is intervated by the thoracodorsal nerve, which is a branch <strong>of</strong> the posterior cord <strong>of</strong><br />

the brachial plexus.<br />

References:<br />

Aids to the Examination <strong>of</strong> the Peripheral Nervous System. 4th ed. Edinburgh: WB Saunders; 2000.<br />

Question 433: Anatomy - Blood Supply <strong>of</strong> Brain/Spinal Cord<br />

Discussion:<br />

The thalamoperforating branches <strong>of</strong> the posterior cerebral arteries perfuse the medial and anterior regions <strong>of</strong> the<br />

thalamus. The thalamogeniculate branches <strong>of</strong> the posterior cerebral arteries perfuse the pulvinar and lateral<br />

nuclei. The inferior thalamic arteries arise from the posterior communicating arteries and perfuse the inferior<br />

portions <strong>of</strong> the thalamus. The medial posterior choroidal artery supplies the superior and medial portions <strong>of</strong> the<br />

thalamus.<br />

References:<br />

Haines DE. Fundamental Neuroscience. 2nd ed. New York: WB Saunders, 2002.<br />

Question 440: Anatomy - Basal Ganglia and Thalamus<br />

Discussion:<br />

<strong>2007</strong> <strong>RITE</strong> Discussion & Reference Manual<br />

The dorsomedial nucleus connects prefrontal, limbic, and olfactory structures with prefrontal cortex. The<br />

intralaminar nuclei project to the cerebral cortex and the basal ganglia. The lateral dorsal nucleus receives input<br />

from the hippocampus and projects to the cingulate gyrus. The pulvinar is an associaton nucleus that receives<br />

inputs from parietal, temporal, and occipital cortex and then projects to these same areas. The reticular nucleus<br />

projects to other thalamic nuclei but not to the cortex.<br />

10


References:<br />

Nolte J. The Human Brain. 4th ed. St Louis: Mosby, 1999.<br />

Behavioral/Psychiatry<br />

Question 7: Behavioral/Psychiatry - Developmental Disorders<br />

Discussion:<br />

The most common cause <strong>of</strong> inherited mental retardation is fragile X syndrome. Nearly all affected boys manifest<br />

attention deficit disorder and have learning disabilities. The most frequent neurocognitive symptoms are abstract<br />

reasoning, complex problem solving, and expressive language. Many will also show manifestations <strong>of</strong> autism,<br />

with 33% meeting criteria for autism. Female carriers can have a milder form <strong>of</strong> the disease with learning<br />

disabilities, and about 50% will manifest attention deficit disorder. Characteristic physical features include a<br />

long thin face, prominent forehead and jaw, protuberant ears, hip dislocation, and club feet.<br />

References:<br />

Rittey CD. Learning difficulties: what the neurologist needs to know. J Neurol Neurosurg Psychiatr<br />

2003;74:30-36.<br />

Question 12: Behavioral/Psychiatry - General Psychiatry<br />

Discussion:<br />

This woman has obsessive-compulsive disorder, and flurodeoxyglucose PET consistently shows hypermetabolic<br />

activity in the caudate, anterior cingulate, and orbit<strong>of</strong>rontal cortex.<br />

References:<br />

Baxter LR, Phelps ME, Mazziotta JC, et al. Local cerebral glucose metabolic rates in obsessive-compulsive<br />

disorder: a comparison with rates in unipolar depression and normal controls. Arch Gen Psychiatry<br />

1987;44:211-218.<br />

Question 14: Behavioral/Psychiatry - Behavioral Complications <strong>of</strong> Systemic Disease<br />

Discussion:<br />

Forced normalization refers to a psychosis occurring after achievement <strong>of</strong> good clinical seizure control or<br />

resolution <strong>of</strong> interictal epileptiform discharges.<br />

References:<br />

Paraiso J, Devinsky D. Neurobehavioral Aspects <strong>of</strong> Epilepsy. In: Feinberg TE, Farah MJ, editors. Behavioral<br />

neurology and neuropsychology. 2nd ed. New York: McGraw-Hill, 2003.<br />

Question 15: Behavioral/Psychiatry - Psychopharmacology<br />

Discussion:<br />

Acetylcholine, vital to the formation and encoding <strong>of</strong> new memories is one <strong>of</strong> many neurotransmitters deficient<br />

in Alzheimer's disease. Medications including tricyclic antidepressants, antihistamines, and antiemetics with<br />

strong anticholinergic properties can worsen memory loss as well as cause confusion.<br />

References:<br />

<strong>2007</strong> <strong>RITE</strong> Discussion & Reference Manual<br />

Cummings JL, Mega MS. Neuropsychiatry and behavioral neuroscience. New York: Oxford University Press,<br />

2003.<br />

Question 16: Behavioral/Psychiatry - Occipital Syndromes<br />

11


Discussion:<br />

There are numerous types <strong>of</strong> reading disorders seen after focal lesions and in neurodegenerative disorders.<br />

Surface dyslexia is characterized by impairment linking the visual word form system with the phonological<br />

output lexicon. Therefore, patients are unable to access the visual word image to link to proper pronounciation.<br />

Patients have to rely on "print to sound conversion" and cannot read words that do not sound the way they are<br />

spelled.<br />

References:<br />

Cummings JL, Mega MS. Neuropsychiatry and behavioral neuroscience. New York: Oxford University Press,<br />

2003.<br />

Question 19: Behavioral/Psychiatry - Dementia<br />

Discussion:<br />

Patients with frontotemporal dementia have been shown to manifest a variety <strong>of</strong> behavioral changes, including<br />

hoarding <strong>of</strong> items and nascent musical and/or artistic expression. This combination <strong>of</strong> behaviors is usually not<br />

seen in other degenerative dementias.<br />

References:<br />

Miller BL, Cummings JL, Boone K, et al. Emergence <strong>of</strong> artistic talent in frontotemporal dementia. Neurology<br />

1998;51:978-981.<br />

Question 31: Behavioral/Psychiatry - Behavioral Complications <strong>of</strong> Systemic Disease<br />

Discussion:<br />

Wilson’s disease is an autosomal recessive disorder caused by a mutation in the ATP7B gene on chromosome<br />

13. Clinical symptoms are neurological, psychiatric, hepatic, or ocular. Other disorders affecting basal ganglia<br />

circuitry can mimic Wilson’s disease; however, only Wilson’s disease will manifest with copper abnormalities<br />

on laboratory screens. These syndromes include pantothenate kinase disease (caused by a mutation in the<br />

PANK2 gene) and Huntington’s disease (trinucleotide repeat disorder on chromosome 4).<br />

References:<br />

Online Mendelian Inheritance in Man (OMIM) TM [homepage on the Internet]. Baltimore: McKusick-Nathans<br />

Institute for Genetic <strong>Medicine</strong>, Johns Hopkins University; Bethesda: National Center for Biotechnology<br />

Information, National Library <strong>of</strong> <strong>Medicine</strong>; c2006 [cited 2006 Aug 4;]. Available from:<br />

www.ncbi.nlm.nih.gov/omim/.<br />

Kitzberger R, Madl C, Ferenci P. Wilson disease. Metab Brain Dis 2005;20:295-302.<br />

Question 35: Behavioral/Psychiatry - Behavioral Complications <strong>of</strong> Systemic Disease<br />

Discussion:<br />

Paroxysmal autonomic instability with dystonia (PAID) is a common symptom cluster similar to malignant<br />

hyperthermia and neuroleptic malignant syndrome. It commonly appears following severe traumatic or hypoxic<br />

brain injury. Treatment generally consists <strong>of</strong> beta-adrenergic blockers, opioid analgesia, dopamine agonists, and<br />

benzodiazepines. Dopamine antagonists can precipitate symptoms similar to PAID. Drugs acting on cholinergic<br />

and serotonin systems have not been found to be effective.<br />

References:<br />

Blackman JA, Patrick PD, Buck ML, Rust RS. Paroxysmal autonomic instability with dystonia after brain<br />

injury. Arch Neurol 2004;61:321-328.<br />

Question 62: Behavioral/Psychiatry - Behavioral Complications <strong>of</strong> Systemic Disease<br />

Discussion:<br />

<strong>2007</strong> <strong>RITE</strong> Discussion & Reference Manual<br />

12


Bilateral hilar adenopathy, bilateral cranial nerve palsies, and an encephalopathy due to a basilar<br />

meningoencephalitis is typical <strong>of</strong> neurosarcoid. Progressive multifocal leukoencephalopathy due to the JC virus,<br />

herpes simplex virus encephalitis, and HIV dementia all are characterized by white matter or cortical lesions on<br />

MRI. The case is inconsistent with strokes due to cardiac thrombi.<br />

References:<br />

Stern BJ. Neurological complications <strong>of</strong> sarcoidosis. Curr Opin Neurol 2004;17:311-316.<br />

Question 67: Behavioral/Psychiatry - Behavioral Complications <strong>of</strong> Systemic Disease<br />

Discussion:<br />

This patient with traumatic brain injury would most likely have frontal lobe impairment given the nature <strong>of</strong> his<br />

accident. Psychosis, visual hallucinations, delusions, and mania would all be atypical. Road rage, episodic<br />

dyscontrol, impulsivity, intrusiveness, apathy and aggression would be more common findings.<br />

References:<br />

Fann JR, Katon WJ, Uomoto JM, Esselman PC. Psychiatric disorders and functional disability in outpatients<br />

with traumatic brain injuries. Am J Psychiatry 1995;152:1493-1499.<br />

Question 70: Behavioral/Psychiatry - Temporal-Limbic Syndromes<br />

Discussion:<br />

Klüver-Bucy syndrome results from bilateral temporal lesions involving the amydala nuclei. Clinical features<br />

include hypermetamorphosis, hyperorality, hypersexuality, visual agnosia, and blunted emotional affect.<br />

Aggression is not a component <strong>of</strong> the syndrome. Hypermetamorphosis occurs when an individual is overly<br />

sensitive or acutely aware <strong>of</strong> minute stimuli in the environment, such as a speck <strong>of</strong> lint on someone's shirt or a<br />

scrap <strong>of</strong> paper on the floor. Patient's with Klüver-Bucy syndrome may become preoccupied with these stimuli<br />

by touching, picking or examining them, symptoms that are described as hypermetamorphosis.<br />

References:<br />

Mendez MF. Pick's disease. In: Feinberg TE, Farah MJ, editors. Behavioral neurology and neuropsychology.<br />

2nd ed. New York: McGraw-Hill, 2003.<br />

Question 75: Behavioral/Psychiatry - Language/Speech Abnormalities<br />

Discussion:<br />

Speech remains intact as language deteriorates with advancing Alzheimer's dementia, eventually producing an<br />

aphasia in which the patient is fluent and paraphasic, their speech is empty, and they have limited<br />

comprehension but repeat well, which is typical <strong>of</strong> transcortical sensory aphasia.<br />

References:<br />

Cummings JL, Darkins A, Mendez M, et al. Alzheimer's disease and Parkinson's disease: comparison <strong>of</strong> speech<br />

and language alterations. Arch Neurol 1988;38:680-684.<br />

Question 84: Behavioral/Psychiatry - Parietal Syndromes<br />

Discussion:<br />

Anosognosia (unawareness <strong>of</strong> deficit or illness) is usually seen associated with nondominant parietal lobe<br />

lesions. Achromatopsia is found after lesions <strong>of</strong> the inferior lip <strong>of</strong> the occipital lobe. Limb kinetic apraxia is seen<br />

after lesions <strong>of</strong> the anterior corpus callosum. Expressive aprosodia is seen after right frontal lesions. Semantic<br />

aphasia is seen after dominant hemisphere lesions.<br />

References:<br />

<strong>2007</strong> <strong>RITE</strong> Discussion & Reference Manual<br />

Feinberg TE, Farah MJ. Behavioral neurology and neuropsychology. 2nd ed. New York: McGraw-Hill, 2003.<br />

13


Question 86: Behavioral/Psychiatry - Occipital Syndromes<br />

Discussion:<br />

Disturbance with the recognition <strong>of</strong> color in one visual field, hemiachromatopsia, occurs only with inferior,<br />

posterior occipital lesions.<br />

References:<br />

Damasio A, Tranel D, Rizzo M. Disorders <strong>of</strong> complex visual processing. In: Mesulam MM, editor. Principles <strong>of</strong><br />

behavioral and cognitive neurology. New York: Oxford University Press, 2000.<br />

Question 96: Behavioral/Psychiatry - Neurobiology <strong>of</strong> Behavior<br />

Discussion:<br />

Depression is <strong>of</strong>ten associated with left anterior frontal vascular lesions.<br />

References:<br />

Cummings JL, Mega MS. Neuropsychiatry and behavioral neuroscience. New York: Oxford University Press,<br />

2003.<br />

Question 105: Behavioral/Psychiatry - Neurobehavioral/Neuropsychological Exam<br />

Discussion:<br />

Multiple system atrophy is a relatively uncommon disorder with a constellation <strong>of</strong> symptoms that include<br />

parkinsonism and autonomic and/or cerebellar dysfunction. Cognitive impairment can be a frequent<br />

manifestation <strong>of</strong> the syndrome and usually manifests with frontal executive dysfunction.<br />

References:<br />

Mendez MF, Cummings JL. Dementia: a clinical approach. 3rd ed. Philadelphia: Butterworth-Heinemann,<br />

2003;260-263.<br />

Question 110: Behavioral/Psychiatry - Dementia<br />

Discussion:<br />

Amyloid starts as an amyloid precursor protein. It is normally cleaved by a series <strong>of</strong> enzymes into a short<br />

version that can easily be excreted by the body. In pathological conditions such as Alzheimer’s disease, amyloid<br />

is incorrectly cleaved by beta-secretase and gamma-secretase. Presenilin-1 assists gamma-secretase in cleaving<br />

the amyloid precursor protein. This abnormal cleaving results in amyloid aggregation that ultimately leads to<br />

plaque formation.<br />

References:<br />

Hardy J, Selkoe DJ. The amyloid hypothesis <strong>of</strong> Alzheimer’s disease: progress and problems on the road to<br />

therapeutics. Science 2002;297:353-356.<br />

Question 111: Behavioral/Psychiatry - General Psychiatry<br />

Discussion:<br />

The loss <strong>of</strong> remote memory, including autobiographical memory, in the face <strong>of</strong> intact new learning ability is<br />

consistent with psychogenic amnesia.<br />

References:<br />

Mendez MF, Cummings JL. Dementia: a clinical approach. 3rd ed. Philadelphia: Butterworth-Heinemann, 2003.<br />

Question 112: Behavioral/Psychiatry - Dementia<br />

<strong>2007</strong> <strong>RITE</strong> Discussion & Reference Manual<br />

14


Discussion:<br />

Any patient with Alzheimer's disease who acutely develops symptoms <strong>of</strong> a confusional state and behavioral<br />

changes first warrants a workup to look for the underlying cause. Even mild changes in metabolic status,<br />

medications, or an infection such as <strong>of</strong> the urinary tract may precipitate confusion and behavioral changes.<br />

References:<br />

Mendez MF, Cummings JL. Dementia: a clinical approach. 3rd ed. Philadelphia: Butterworth-Heinemann,<br />

2003;260-263.<br />

Question 116: Behavioral/Psychiatry - Language/Speech Abnormalities<br />

Discussion:<br />

Conduction aphasia, also called associative aphasia, is a relatively rare form <strong>of</strong> aphasia caused by damage to the<br />

nerve fibers in the arcuate fasciculus, which connects Wernicke's and Broca's areas. Patients with conduction<br />

aphasia show the following characteristics: speech is fluent, comprehension remains good, oral reading is poor,<br />

repetition is poor, transpositions <strong>of</strong> sounds within a word (television/velitision) are common. To understand the<br />

symptoms, recall that Broca's area controls expression whereas Wernicke's area is responsible for<br />

comprehension. When both areas are intact but the neural connections between them is broken, there is the<br />

curious condition in which the patient can understand what is being said but cannot repeat it (or repeats it<br />

incorrectly). Such a patient will also end up saying something inappropriate or wrong, realize his/her mistake,<br />

but continue making further mistakes while trying to correct it.<br />

References:<br />

Cummings JL, Mega MS. Neuropsychiatry and behavioral neuroscience. New York: Oxford University Press,<br />

2003.<br />

Question 122: Behavioral/Psychiatry - Behavioral Complications <strong>of</strong> Systemic Disease<br />

Discussion:<br />

Korsak<strong>of</strong>f amnestic syndrome causes impairment in declarative memory (anterograde amnesia) and forgetting<br />

<strong>of</strong> recent events (retrograde amnesia) with sparing <strong>of</strong> motor memory and semantic memory (memory for<br />

meaning <strong>of</strong> words). Digit span remains normal in this syndrome.<br />

References:<br />

Cummings JL, Mega MS. Neuropsychiatry and behavioral neuroscience. New York: Oxford University Press,<br />

2003.<br />

Question 123: Behavioral/Psychiatry - Neurobehavioral/Neuropsychological Exam<br />

Discussion:<br />

<strong>2007</strong> <strong>RITE</strong> Discussion & Reference Manual<br />

The Clinical Dementia Rating (CDR) Scale is a dementia staging instrument used to rate cognitive function<br />

along five levels <strong>of</strong> impairment from none to maximal (rated as 0, 0.5, 1, 2, or 3) in each <strong>of</strong> six domains: (1)<br />

memory, (2) orientation, (3) judgment and problem solving, (4) function in community affairs, (5) home and<br />

hobbies, and (6) personal care. (Personal care has no 0.5 impairment level.) Only impairment caused by<br />

cognitive dysfunction is rated. Community affairs and home and hobbies assess instrumental activities <strong>of</strong> daily<br />

living relevant to the individual and hence vary according to that person’s accustomed activities; examples<br />

include job performance for those who still are employed and skills in driving, home repairs, household<br />

finances, shopping, cooking, and card games. Personal care represents basic activities <strong>of</strong> daily living common to<br />

almost all individuals (dressing, bathing and grooming, eating, and continence). Based on the collateral source<br />

and participant interviews, a global CDR score is derived from individual ratings in each domain such that CDR<br />

0 indicates no dementia and CDR 0.5, 1, 2, and 3 represent very mild--also referred to as mild cognitive<br />

impairment (MCI), mild, moderate, and severe dementia, respectively. Interrater reliability for the CDR has<br />

been established at about 88%. Not all domains need be rated at the same level <strong>of</strong> impairment as the global CDR<br />

score; for example, a participant may merit a box score <strong>of</strong> 1 for memory but scores <strong>of</strong> 0.5 or 0 for other domains<br />

and still have a global CDR <strong>of</strong> 0.5. The individual ratings can be totaled to yield the sum boxes, a more<br />

15


quantitative rating that ranges from 0 (or no impairment in any <strong>of</strong> the 6 domains) to 18 (or maximal impairment<br />

in each <strong>of</strong> the 6 domains).<br />

References:<br />

Morris JC, Ernesto C, Schaefer K, et al. Clinical dementia rating (CDR) training and reliability protocol: the<br />

Alzheimer Disease Cooperative Study Unit experience. Neurology 1997;48:1508-1510.<br />

Question 141: Behavioral/Psychiatry - Dementia<br />

Discussion:<br />

The Dementia with Lewy bodies (DLB) Consortium has revised criteria for the clinical and pathologic diagnosis<br />

<strong>of</strong> DLB incorporating new information about the core clinical features and suggesting improved methods to<br />

assess them. REM sleep behavior disorder, severe neuroleptic sensitivity, and reduced striatal dopamine<br />

transporter activity on functional neuroimaging are given greater diagnostic weighting as features suggestive <strong>of</strong><br />

a DLB diagnosis. When any <strong>of</strong> these are present with one <strong>of</strong> the primary findings <strong>of</strong> visual hallucinations,<br />

parkinsonism, or fluctuating attention, then the diagnosis <strong>of</strong> probable DLB is supported.<br />

References:<br />

McKeith IG, Dickson DW, Lowe J, et al. Diagnosis and management <strong>of</strong> dementia with Lewy bodies: third<br />

report <strong>of</strong> the DLB consortium. Neurology 2005;65:1863–1872.<br />

Question 146: Behavioral/Psychiatry - Dementia<br />

Discussion:<br />

Pick's disease is usually manifested by disinhibition, socially inappropriate behavior, and sometimes the<br />

Kluver-Bucy syndrome. Although some patients may develop depression, it is far less likely than the incidence<br />

<strong>of</strong> depression in Parkinson's, Wilson's, Huntington's disease, and multiple sclerosis.<br />

References:<br />

Mendez MF, Cummings JL. Dementia: a clinical approach. 3rd ed. Philadelphia: Butterworth-Heinemann, 2003.<br />

Question 147: Behavioral/Psychiatry - Psychopharmacology<br />

Discussion:<br />

Bupropion has had a low incidence <strong>of</strong> erectile dysfunction associated with its use. All <strong>of</strong> the selective serotonin<br />

reuptake inhibitors (SSRIs) have been reported to have erectile dysfunction as a side effect. Amitriptyline and<br />

venlafaxine also cause erectile dysfunction.<br />

References:<br />

Arana GW, Rosenbaum JF. Handbook <strong>of</strong> psychiatric drug therapy. 5th ed. Philadelphia: Lippincott, Williams &<br />

Wilkins, 2005.<br />

Question 149: Behavioral/Psychiatry - Language/Speech Abnormalities<br />

Discussion:<br />

An aphasia is considered fluent if: word output per minute is high; there are five or more words per phrase;<br />

content per phrase is low; paraphasias are present; and speech is nondysarthric with normal prosody.<br />

References:<br />

Benson DF, Ardila A. Aphasia: a clinical perspective. Oxford: Oxford University Press, 1996.<br />

Question 167: Behavioral/Psychiatry - Parietal Syndromes<br />

Discussion:<br />

<strong>2007</strong> <strong>RITE</strong> Discussion & Reference Manual<br />

16


Asomatognosia is a form <strong>of</strong> neglect in which patients deny ownership <strong>of</strong> their limbs; it frequently accompanies<br />

anosognosia. The lesion is generally located in the nondominant supramarginal gyrus.<br />

References:<br />

Meador KJ, Loring DW, Feinberg TE, et al. Anosognosia and asomatognosia during intracarotid amobarbital<br />

inactivation. Neurology 2000;55:816-820.<br />

Feinberg TE, Laber LD, Needs NE. Verbal asomatognosia. Neurology 1990;40:1391-1394.<br />

Question 178: Behavioral/Psychiatry - Neurobiology <strong>of</strong> Behavior<br />

Discussion:<br />

Several neurotransmitters and hormones have been implicated in the modulation <strong>of</strong> violent behavior. Most recent<br />

evidence has found low levels <strong>of</strong> CSF 5-HIAA in patients who have attempted suicide via violent means as well<br />

as in alcoholics with impulsive violent behavior. Norepinephrine and COMT have also been implicated in<br />

aggressive behavior.<br />

References:<br />

Volavka J. The neurobiology <strong>of</strong> violence: an update. J Neuropsychiatry and Clin Neurosci 1999;11:307-314<br />

Question 179: Behavioral/Psychiatry - Behavioral Complications <strong>of</strong> Systemic Disease<br />

Discussion:<br />

Numerous neuropsychiatric symptoms have been associated with epilepsy, particularly with temporal lobe<br />

epilepsy. The symptoms include psychosis, fear, anxiety, hypergraphia, hypermorality, and altered sexual<br />

function. The most commonly reported and readily treatable symptom is depression.<br />

References:<br />

Bortz JJ. Neuropsychiatric and memory issues in epilepsy. Mayo Clin Proc 2003;78:781-787.<br />

Jones JE, Hermann BP, Barry JJ, et al. Clinical assessment <strong>of</strong> Axis I psychiatric morbidity in chronic epilepsy: a<br />

multicenter investigation. J Neuropsychiatry Clin Neurosci 2005;17:172-179.<br />

Question 194: Behavioral/Psychiatry - Frontal Systems Syndromes<br />

Discussion:<br />

The thalamus is a major relay station for most inherent functions <strong>of</strong> the brain to include cognition. This nuclear<br />

structure can be subdivided into regions based on functional relationships. Lesions in specific subnuclei can lead<br />

to different clinical manifestations. For example, lesions in the anterior group are more likely to manifest<br />

amnesia, confabulation, anomia and preserved visuospatial function. Paramedian thalamic lesions can manifest<br />

wtih acute decreased consciousness followed by vertical gaze paresis, disinhibition and at times amnesia.<br />

References:<br />

Carrera E, Bogousslavsky J. The thalamus and behavior: effects <strong>of</strong> anatomically distinct strokes. Neurology<br />

2006;66(12):1817-1823.<br />

Question 197: Behavioral/Psychiatry - Frontal Systems Syndromes<br />

Discussion:<br />

Personality changes associated with dorsolateral frontal dysfunction include apathy, self-absorption,<br />

perseveration, neurovegetative symptoms (eg, eating and sleeping disturbances), irritability, and agitation.<br />

Psychosis is usually seen with temporal lobe dysfunction. Obsessive-compulsive traits, disinhibition,<br />

hypersexuality, and intrusiveness are typical <strong>of</strong> lesions affecting the orbit<strong>of</strong>rontal cortex.<br />

References:<br />

<strong>2007</strong> <strong>RITE</strong> Discussion & Reference Manual<br />

Miller BL, Cummings JL, editors. The human frontal lobes: Functions and disorders. New York: The Guilford<br />

17


Miller BL, Cummings JL, editors. The human frontal lobes: Functions and disorders. New York: The Guilford<br />

Press, 1999.<br />

Question 198: Behavioral/Psychiatry - General Psychiatry<br />

Discussion:<br />

Factitious disorder is defined as a syndrome <strong>of</strong> intentional production <strong>of</strong> psychological or physical symptoms in<br />

the absence <strong>of</strong> external incentives but in the presence <strong>of</strong> a psychological need to assume the sick role. When<br />

there are external incentives for the behavior, then malingering is the likely diagnosis. Amnestic disorder is<br />

when the individual has difficulties learning new things. Conversion symptoms are subconscious and not<br />

intentionally produced. Somat<strong>of</strong>orm disorder refers to individuals who have recurrent and multiple somatic<br />

complaints not due to any physical disorder.<br />

References:<br />

American Psychiatric Association. Diagnostic and Statistical Manual <strong>of</strong> Mental Disorders. 4th ed., text revision.<br />

Washington DC: American Psychiatric Association, 2000.<br />

Question 317: Behavioral/Psychiatry - Dementia<br />

Discussion:<br />

Numerous studies have found a serotonergic deficit in patients with frontotemporal dementia (FTD). Experts in<br />

the field will <strong>of</strong>ten treat these patients with selective serotonin reuptake inhibitors (SSRIs) even in the absence <strong>of</strong><br />

depression. There is no evidence <strong>of</strong> a cholinergic deficit in FTD, and studies evaluating the efficacy <strong>of</strong><br />

cholinesterase inhibitors have been largely neutral or negative. Purported neuroprotective and antioxidant<br />

compounds have also not been found to be beneficial.<br />

References:<br />

Graff-Radford N, Woodruff B. Frontotemporal dementia. Continuum 2004;10:58-80.<br />

Litvan I. Therapy and management <strong>of</strong> frontal lobe dementia patients. Neurol 2001;56(Suppl 4):S41-S45.<br />

Question 320: Behavioral/Psychiatry - Developmental Disorders<br />

Discussion:<br />

Attention deficit hyperactivity disorder (ADHD) is a highly heritable, disruptive, childhood-onset condition, the<br />

etiology and pathogenesis <strong>of</strong> which is poorly understood. There have been relatively few genome-wide linkage<br />

studies, and no chromosomal region has yet been unequivocally implicated. In contrast, evidence from<br />

pharmacological, neuroimaging, and animal studies has suggested the involvement <strong>of</strong> specific neurotransmitter<br />

systems, notably dopaminergic pathways. Meta-analyses or pooled data analyses have supported association<br />

between ADHD and polymorphisms in DRD4, DRD5, and SLC6A3, which encode dopamine D4 and D5<br />

receptors and the dopamine transporter, respectively.<br />

References:<br />

Waldman ID, Gizer IR. The genetics <strong>of</strong> attention deficit hyperactivity disorder. Clin Psychol Rev<br />

2006;26:396-432.<br />

Question 324: Behavioral/Psychiatry - Psychopharmacology<br />

Discussion:<br />

Olanzapine is an atypical antipsychotic that frequently causes significant weight gain. Quetiapine, risperidone,<br />

haloperidol, and molindone are less likely to do so.<br />

References:<br />

<strong>2007</strong> <strong>RITE</strong> Discussion & Reference Manual<br />

Puzantian T, Stimmel G. Review <strong>of</strong> psychotropic drugs. New York: McMahon, 2001.<br />

18


Question 328: Behavioral/Psychiatry - Behavioral Complications <strong>of</strong> Systemic Disease<br />

Discussion:<br />

Prion proteins cause spongiform encephalopathy characterized by rapidly progressive dementia with myoclonus<br />

and seizures. Progressive multifocal leukoencephalopathy caused by the JC virus traditionally presents with<br />

encephalopathy, vision loss, paralysis, and ataxia. This syndrome, like human immunodeficiency virus (HIV),<br />

usually presents in patients who are immunocompromised. The neurological manifestations <strong>of</strong> HIV are many<br />

and include myelopathy, sensory neuropathy, and dementia. Tropheryma whippelii is the organism identified as<br />

the cause <strong>of</strong> Whipple’s disease. Patients typically present with gastrointestinal complaints including diarrhea,<br />

malabsorption, and weight loss. Other symptoms may include lymphadenopathy, hyperpigmented skin,<br />

movement disorders, oculomasticatory myodysrhythmia, and dementia. Herpes simplex virus typically presents<br />

with changes in personality and seizures; gastrointestinal symptoms are uncommon.<br />

References:<br />

Manzel K, Tranel D, Cooper G. Cognitive and behavioral abnormalities in a case <strong>of</strong> central nervous system<br />

Whipple disease. Arch Neurol 2000;57:399-403.<br />

Question 334: Behavioral/Psychiatry - Dementia<br />

Discussion:<br />

Most patients with Alzheimer’s disease (AD) have a sporadic late-onset form <strong>of</strong> the disease. A small percentage<br />

<strong>of</strong> patients, however, have familial disease produced by one <strong>of</strong> three autosomal dominant genes. Among familial<br />

AD patients, 50% to 70% have the presenilin-1 mutation or an associated mutation on chromosome 14. Five<br />

percent to 10% have the presenilin-2 mutation. A small percentage will have a mutation in the gene that codes<br />

for amyloid precursor protein.<br />

References:<br />

Bird TD. Genetic factors in Alzheimer’s disease. N Eng J Med 2005;352:862-864.<br />

Question 339: Behavioral/Psychiatry - Temporal-Limbic Syndromes<br />

Discussion:<br />

Often, pseudoseizures are considered when bilateral rhythmic motor output occurs without loss <strong>of</strong><br />

consciousness, especially when an EEG captures the “spell” and no electrophysiological correlate is found with<br />

scalp or sphenoidal leads. However, the midportion <strong>of</strong> the anterior cingulate (adjacent to the supplementary<br />

motor cortex) has a bilateral motor homunculus that, when affected by an ictal focus, will produce bilateral<br />

rhythmic motor output (without loss <strong>of</strong> consciousness) detectable only with subdural brain surface electrodes.<br />

This limbic focus <strong>of</strong>ten manifests as a primary psychiatric disorder escaping routine neurological surveillance.<br />

References:<br />

Devinsky O, Morrell MJ, Vogt BA. Contributions <strong>of</strong> anterior cingulate cortex to behavior. Brain<br />

1995;118:279-306.<br />

Question 343: Behavioral/Psychiatry - Language/Speech Abnormalities<br />

Discussion:<br />

Numerous deficits <strong>of</strong> recognition following right hemisphere damage have been described. These syndromes<br />

generally do not involve deficits in discrimination and result from lesions outside <strong>of</strong> unimodal cortex.<br />

Prosopagnosia, a deficit in facial recognition, is probably the most well known. Lesser known syndromes<br />

include auditory agnosia, a deficit in recognition <strong>of</strong> verbal and nonverbal sounds; autotopagnosia, which<br />

represents the inability to localize stimuli on the affected side <strong>of</strong> the body; and phonagnosia, which is the<br />

inability to recognize familiar voices. Pure word deafness, unlike the others, results from lesions <strong>of</strong> the dominant<br />

hemisphere and leaves patients with the inability to recognize spoken language with spared nonverbal<br />

communication and fluency.<br />

References:<br />

<strong>2007</strong> <strong>RITE</strong> Discussion & Reference Manual<br />

19


Shah NJ, Marshall JC, Zafiris O, et al. The neural correlates <strong>of</strong> person familiarity. A functional magnetic<br />

resonance imaging study with clinical implications. Brain 2001;124:804-815.<br />

Van Lancker DR, Krieman J, Cummings J. Voice perception deficits: neuroanatomical correlates <strong>of</strong><br />

phonagnosia. J Clin Exp Neuropsychol 1989;11:665-674.<br />

Question 358: Behavioral/Psychiatry - Dementia<br />

Discussion:<br />

Although chromosome 3 and 9 have been linked to kindreds with frontotemporal dementia (FTD), the<br />

overwhelming majority <strong>of</strong> patients with FTD have been found to have mutations on chromosome 17. They may<br />

also demonstrate features <strong>of</strong> parkinsonism. Chromosome 17 harbors the gene for tau and at least 30 mutations<br />

have been discovered in this gene among 100+ families. Chromosomes 12 and 19 have been implicated in<br />

Alzheimer’s disease.<br />

References:<br />

Mendez MF, Cummings JL. Dementia: a clinical approach. 3rd ed. Philadelphia: Butterworth-Heinemann, 2003.<br />

Sobrido MJ, Wiedau-Pazos M, Geschwind DH. The genetics <strong>of</strong> frontotemporal dementia and related disorders.<br />

Current Genomics 2000;1:339-352.<br />

Question 363: Behavioral/Psychiatry - Behavioral Complications <strong>of</strong> Systemic Disease<br />

Discussion:<br />

Numerous neurological symptoms can be seen associated with systemic lupus erythematosis (SLE). These<br />

manifestations include peripheral neuropathy and cerebritis as well as neuropsychiatric symptoms such as<br />

depression, mania, and psychosis. SLE is more frequently associated with psychosis than are Bechet's syndrome,<br />

Ehlers-Danlos syndrome, rheumatoid arthritis, or Sjogren's syndrome.<br />

References:<br />

Cummings JL, Mega MS. Neuropsychiatry and behavioral neuroscience. New York: Oxford University Press,<br />

2003.<br />

Question 364: Behavioral/Psychiatry - Neurobehavioral/Neuropsychological Exam<br />

Discussion:<br />

Numerous neuropsychological tests have been devised that target predominantly one cognitive domains. These<br />

include tests <strong>of</strong> visuospatial abilities for parietal lobe functions, memory for mesial temporal structures and tests<br />

<strong>of</strong> sustained and complex attention for frontal lobe/executive function.<br />

References:<br />

Cummings JL, Mega MS. Neuropsychiatry and behavioral neuroscience. New York: Oxford University Press,<br />

2003.<br />

Question 370: Behavioral/Psychiatry - Behavioral Complications <strong>of</strong> Systemic Disease<br />

Discussion:<br />

HIV infection can result in minor cognitive and motor disorder, HIV-associated mild neurocognitive disorder,<br />

and HIV-associated dementia. The earliest symptoms revolve around mental slowing and processing speed.<br />

Tests that assess processing speed, including trails A and B, grooved pegboard, Symbol Digit Modalities Test,<br />

and the HIV Dementia Scale are likely to be abnormal early in the disease course.<br />

References:<br />

<strong>2007</strong> <strong>RITE</strong> Discussion & Reference Manual<br />

Mendez MF, Cummings JL. Dementia: a clinical approach. 3rd ed. Philadelphia: Butterworth-Heinemann, 2003.<br />

20


Question 371: Behavioral/Psychiatry - Language/Speech Abnormalities<br />

Discussion:<br />

Several disorders <strong>of</strong> language can result following damage to the dominant hemisphere. Detailed examination <strong>of</strong><br />

the six components <strong>of</strong> the language examination (fluency, comprehension, repetition, reading, writing, and<br />

naming) is essential to help distinguish them. Wernicke’s aphasia is characterized by fluent speech with<br />

impaired comprehension <strong>of</strong> written and spoken words. Patient’s with aphemia are relatively nonfluent in spoken<br />

language; however, comprehension and written communication are much better preserved. The hallmark <strong>of</strong><br />

conduction aphasia is impaired repetition with relative sparing <strong>of</strong> other components <strong>of</strong> language. Patients who<br />

suffer from pure word deafness are unable to repeat or comprehend spoken language; however, they can still<br />

communicate effectively via writing.<br />

References:<br />

Benson DF, Ardila A. Aphasia: a clinical perspective. New York: Oxford University Press, 1996.<br />

Question 374: Behavioral/Psychiatry - General Psychiatry<br />

Discussion:<br />

Catatonia is a syndrome manifested by a number <strong>of</strong> motor and neurobehavioral features. It may have a<br />

"retarded-stuporous" form or an "excited-delirious" form. It may be seen in over 10% <strong>of</strong> inpatient psychiatric<br />

patients. Catatonia is more prevalent in mood disorders than in schizophrenia. The most common mood disorder<br />

in which it is seen is bipolar. Catalepsy, waxy flexibility, echophenomena, and negativism including mutism are<br />

common. Many neurological and systemic illnesses may also present as catatonia. Treatments include<br />

benzodiazepines, barbiturates, and electroconvulsive therapy. Dopamine antagonists as well as bacl<strong>of</strong>en may<br />

worsen the condition.<br />

References:<br />

Taylor MA, Fink M. Catatonia in psychiatric classification: a home <strong>of</strong> its own. Am J Psychiatry<br />

2003;160:1233-1241.<br />

Question 381: Behavioral/Psychiatry - Neurobiology <strong>of</strong> Behavior<br />

Discussion:<br />

Akinetic mutism may result from anterior cingulate lesions or a disconnection <strong>of</strong> the limbic connections<br />

projecting from the anterior cingulate through subcortical circuits. Based on nonhuman primate primate tracer<br />

studies, ventral pallidal lesions should disrupt the anterior cingulate frontal-subcortical circuit. A patient will<br />

develop a rigid akinetic mute state caused by bilateral lesions <strong>of</strong> the globus pallidus interna with ventral<br />

extension.<br />

References:<br />

Cummings JL, Mega MS. Neuropsychiatry and behavioral neuroscience. New York: Oxford University Press,<br />

2003.<br />

Question 393: Behavioral/Psychiatry - Dementia<br />

Discussion:<br />

A posterior parietal-occipital functional defect is <strong>of</strong>ten seen in dementia with Lewy bodies, unlike Alzheimer's<br />

disease, which typically has a temporal parietal functional defect.<br />

References:<br />

<strong>2007</strong> <strong>RITE</strong> Discussion & Reference Manual<br />

McKeith IG, Dickson DW, Lowe J, et al. Diagnosis and management <strong>of</strong> dementia with Lewy bodies: third<br />

report <strong>of</strong> the DLB consortium. Neurology 2005;65:1863–1872.<br />

Question 402: Behavioral/Psychiatry - Developmental Disorders<br />

21


Discussion:<br />

Tuberous sclerosis is a genetic disorder associated with numeorus skin and systemic manifestations as well as<br />

intracranial tubers, mental retardation, and seizures. Although numerous neuropsychiatric symptoms have been<br />

reported to be associated with tuberous sclerosis, autism spectrum disorder is the most common.<br />

References:<br />

Wiznitzer M. Autism and tuberous sclerosis. Child Neurol 2004;19:675-679.<br />

Question 411: Behavioral/Psychiatry - Psychopharmacology<br />

Discussion:<br />

The serotonin syndrome results from concomitant administration <strong>of</strong> medications that enhance serotonin<br />

transmission via decreased breakdown or increased production. Medication combinations to use cautiously<br />

include monoamine oxidase inhibitor agents with selective serotonin reuptake inhibitors, tricyclic<br />

antidepressants, or dextromethorphan. The serotonin syndrome can be differentiated from neuroleptic malignant<br />

syndrome by the presence <strong>of</strong> shivering and myoclonus in the former.<br />

References:<br />

Boyer EW, Shannon M. Current Concepts: the serotonin syndrome. New Eng J Med 2005;352:1112-1120.<br />

Christensen RC. Identifying serotonin syndrome in the emergency department. Am J Emerg Med<br />

2005;23:406-408.<br />

Question 436: Behavioral/Psychiatry - Dementia<br />

Discussion:<br />

Subcortical dementia is characterized clinically by psychomotor slowing, forgetfulness, cognitive decline,<br />

visuospatial impairment, and personality changes, especially in mood. Bradyphrenia (slowness <strong>of</strong> mental<br />

processing) is very common.<br />

References:<br />

Mendez MF, Cummings JL. Dementia: a clinical approach. 3rd ed. Philadelphia: Butterworth-Heinemann, 2003.<br />

Clinical Adult<br />

Question 3: Clinical Adult - Neuromuscular Disorders<br />

Discussion:<br />

Charcot-Marie-Tooth disease (CMT) 1A typically shows distal weakness, nerve hypertrophy, and pes cavus<br />

associated with a duplication <strong>of</strong> the PMP22 gene. In hereditary neuropathy with liability to pressure palsies<br />

(HNPP), there is a deletion <strong>of</strong> the PMP22 gene. CMT 2 is the axonal phenotype.<br />

References:<br />

Nicholson GA. The dominantly inherited motor and sensory neuropathies: clinical and molecular advances.<br />

Muscle Nerve 2006;33:589-597.<br />

Question 4: Clinical Adult - Dementia<br />

Discussion:<br />

<strong>2007</strong> <strong>RITE</strong> Discussion & Reference Manual<br />

According to the guidelines by the NINCDS-ADRDA, the routine evaluation <strong>of</strong> the patient with dementia<br />

should include the following laboratory tests: (1) complete blood count, (2) serum electrolytes, (3) glucose, (4)<br />

blood urea nitrogen/creatinine, (5) serum B12 levels, (6) depression screening, (7) liver function test, and (8)<br />

thyroid function test. Venereal Disease Research Laboratory (VDRL), HIV, lumbar puncture, and heavy metal<br />

screen are not recommended in routine dementia screening without clinical indication.<br />

22


References:<br />

Dubinsky RM, Stein AC, Lyons K. Practice parameter: risk <strong>of</strong> driving and Alzheimer's disease (an<br />

evidence-based review): report <strong>of</strong> the Quality Standards Subcommittee <strong>of</strong> the American Academy <strong>of</strong> Neurology.<br />

Neurology 2000;54(12):2205-2211.<br />

Question 23: Clinical Adult - Movement Disorders<br />

Discussion:<br />

The onset <strong>of</strong> or<strong>of</strong>acial dyskinesias with lingual and oral dystonia in a 30-year-old patient is characteristic <strong>of</strong><br />

neuroacanthocytosis, which may also be associated with chorea and peripheral polyneuropathy.<br />

References:<br />

Bradley WG, Dar<strong>of</strong>f RB, Fenichel GM, et al, editors. Neurology in clinical practice. 3rd ed. Boston:<br />

Butterworth-Heinemann, 2000.<br />

Question 33: Clinical Adult - Neurology <strong>of</strong> Systemic Disease<br />

Discussion:<br />

Anticoagulation may produce spontaneous hemorrhage into the psoas or iliacus muscle and produce and acute<br />

ili<strong>of</strong>emoral neuropathy. CT <strong>of</strong> the abdomen an pelvis is most useful for imaging acute blood in the<br />

retroperitoneum.<br />

References:<br />

Seijo-Martinez M. Acute femoral neuropathy secondary to an iliacus muscle hematoma. J Neurol Sci 2003;<br />

209(1-2): 119-122.<br />

Question 40: Clinical Adult - Headache<br />

Discussion:<br />

Valproic acid, amytriptilline, and propranolol have the best proven benefit as prophylactic agents. Drug choice<br />

should be balanced with the patient's comorbidities. AAN practice guidelines have evaluated evidence-based<br />

data for acute and prophylactic therapy based upon stastical and clinical benefit in published clinical trials.<br />

References:<br />

Silberstein S. Practice parameter: Evidence-based guidelines for migraine headache (an evidence-based review).<br />

Report <strong>of</strong> the Quality Standards Subcommittee <strong>of</strong> the American Academy <strong>of</strong> Neurology. Neurology 2000; 55:<br />

754-763.<br />

Question 49: Clinical Adult - Cerebrovascular Disease<br />

Discussion:<br />

In the syndrome <strong>of</strong> alexia without agraphia, a complete right homonymous hemianopsia is present in many<br />

cases, but there are exceptions. Impaired naming and understanding <strong>of</strong> color names in the presence <strong>of</strong> intact or<br />

nearly intact color vision is common, although some patients suffer from an actual impairment <strong>of</strong> color vision.<br />

Mild anomia is common but not always present. In most cases, there are no other aphasic disturbances or<br />

abnormalities <strong>of</strong> the primary motor or sensory systems. The most frequently reported pathology is occlusion <strong>of</strong><br />

the dominant (left) posterior cerebral artery.<br />

References:<br />

Brazis PW, Masdeu JC, Biller J. Localization in clinical neurology. 4th ed. Philadelphia: Lippincott, Williams &<br />

Wilkins, 2001.<br />

Question 53: Clinical Adult - Motor Neuron/Nerve<br />

<strong>2007</strong> <strong>RITE</strong> Discussion & Reference Manual<br />

23


Discussion:<br />

This patient has peroneal compression neuropathy due to significant weight loss and leg crossing. The<br />

appropriate management would be to caution him against leg crossing.<br />

References:<br />

Rubin DI, Kimmel DW, Cascino TL. Outcome <strong>of</strong> peroneal neuropathies in patients with systemic malignant<br />

disease. Cancer 1998;83:1602-1606.<br />

Question 58: Clinical Adult - Critical Care/Trauma<br />

Discussion:<br />

This patient has raised intracranial pressure most probably due to a developing epidural hematoma. Pending<br />

definitive treatment with a neurosurgical procedure, intracranial pressure should be managed by putting the<br />

patient’s head up 40 degrees, intubation and hyperventilation, and administering IV mannitol.<br />

References:<br />

Wijdicks, EF. Neurologic Catastophies in the Emergency Department. Woburn, Mass: Butterworth-Heinemann,<br />

2000.<br />

Question 65: Clinical Adult - Spinal and Root Disorders<br />

Discussion:<br />

This patient has a C5 to C6 disc herniation producing a relatively mild acute C6 radiculopathy. The diagnosis is<br />

clear from the information provided, and no additional diagnostic testing (EMG, myelography) should be<br />

required. In most cases the symptoms will resolve spontaneously without need for cervical discectomy. Surgery<br />

would be required if the patient had significant radicular weakness, cervical myelopathy, or intractable radicular<br />

pain. Epidural steroid injection does not have a defined role in the treatment <strong>of</strong> radiculopathy but may improve<br />

pain control in patients with radicular pain that is severe or does not respond to oral analgesics.<br />

References:<br />

Noseworthy J. Neurological therapeutics Principles and Practice. 2nd ed. London: Informa Healthcare, 2006.<br />

Question 68: Clinical Adult - Cerebrovascular Disease<br />

Discussion:<br />

Cerebral amyloid angiopathy usually affects the elderly and accounts for up to 10% <strong>of</strong> intracranial hemorrhages.<br />

The typical location <strong>of</strong> the hemorrhages is in the lobar areas. The deposition <strong>of</strong> beta amyloid protein in the<br />

media and adventitia <strong>of</strong> small meningeal and cortical vessels result in lobar hemorrhages that frequently recur.<br />

References:<br />

Victor M, Ropper A. Adams and Victor's principles <strong>of</strong> neurology. 7th ed. New York: McGraw-Hill, 2001.<br />

Question 71: Clinical Adult - Neuromuscular Disorders<br />

Discussion:<br />

Polymyositis <strong>of</strong>ten presents as an acquired progressive proximal weakness with elevated creatine kinase levels<br />

and EMG demonstrating fibrillations and small polyphasic potentials.<br />

References:<br />

Engel AG, Franzini-Armstrong C. Myology. 3rd ed. New York: McGraw-Hill, 2004.<br />

Question 72: Clinical Adult - Neuro-ophthalmology/Neuro-otology<br />

Discussion:<br />

<strong>2007</strong> <strong>RITE</strong> Discussion & Reference Manual<br />

24


A lesion <strong>of</strong> the right upper bank <strong>of</strong> the calcarine fissure results in a homonymous left inferior quadrantanopsia.<br />

References:<br />

Haines DE. Fundamental Neuroscience. 3rd ed. Philadelphia: Churchill Livingstone/Elsevier 2006; 251.<br />

Question 95: Clinical Adult - Neuromuscular Disorders<br />

Discussion:<br />

When the examiner is testing the biceps strength, the subject's forearm should be held in supination to eliminate<br />

elbow flexion force produced by the brachioradialis muscle.<br />

References:<br />

Haerer AF. Dejong's The neurologic examination. 5th ed. Philadelphia: Lippincott, Williams & Wilkins, 1992.<br />

Question 100: Clinical Adult - Movement Disorders<br />

Discussion:<br />

This patient has drug-induced parkinsonism due to use <strong>of</strong> metoclopramide, a dopamine receptor antagonist. The<br />

other medications would not produce parkinsonism.<br />

References:<br />

Noseworthy J. Neurological therapeutics Principles and Practice. 2nd ed. London: Informa Healthcare, 2006.<br />

Question 102: Clinical Adult - Cerebrovascular Disease<br />

Discussion:<br />

This patient has a cerebral infarct in the distribution <strong>of</strong> the right anterior cerebral artery. If contrast is<br />

administered, subacute infarcts commonly enhance, usually with a gyriform pattern.<br />

References:<br />

Ropper AH, Brown RH. Adams and Victor's Principles <strong>of</strong> Neurology. 8th ed. New York: McGraw-Hill, 2005;<br />

261-264.<br />

Question 108: Clinical Adult - Demyelinating Disease<br />

Discussion:<br />

Based on the history and examination, this patient most likely has optic neuritis. By history, she has had<br />

multiple episodes <strong>of</strong> neurological dysfunction separated temporally and anatomically. This patient most likely<br />

has demyelinating disease. The CSF immunoglobulin G index and oligoclonal bands are typically elevated in<br />

the CSF <strong>of</strong> a patient with acute optic neuritis. Sarcoidosis and Lyme disease can also produce optic neuritis but<br />

are less common in a patient <strong>of</strong> this age and gender.<br />

References:<br />

Noseworthy J. Neurological therapeutics Principles and Practice. 2nd ed. London: Informa Healthcare, 2006.<br />

Question 113: Clinical Adult - Neuromuscular Disorders<br />

Discussion:<br />

The pronator quadratus is responsible for pronation <strong>of</strong> the forearm when the elbow is flexed. It is supplied by the<br />

anterior interosseous nerve, a branch <strong>of</strong> the median nerve.<br />

References:<br />

<strong>2007</strong> <strong>RITE</strong> Discussion & Reference Manual<br />

Daube JR, editor. Clinical neurophysiology. Philadelphia: FA Davis, 1996;81-84.<br />

25


Question 114: Clinical Adult - Demyelinating Disease<br />

Discussion:<br />

The daughter <strong>of</strong> a parent with multiple sclerosis has about a 5% chance <strong>of</strong> developing the disease. This<br />

percentage is higher than siblings, with the exception <strong>of</strong> monozygotic twins.<br />

References:<br />

Miller A, Bourdette D, Cohen J, et al. Multiple Sclerosis – Part B. In Miller A, editor. Continuum 1999; 5 (Oct):<br />

8.<br />

Question 118: Clinical Adult - Epilepsy<br />

Discussion:<br />

Levetiracetam, gabapentin, tiagabine, vigabatrin, zonisamide, and topiramate (


Neurology 2000;54(12):2205-2211.<br />

Question 143: Clinical Adult - Headache<br />

Discussion:<br />

Divalproex sodium is considered an effective prophylactic medication for migraines. Carbamazepine has not<br />

proven to be as effective and is reserved more for the treatment <strong>of</strong> painful peripheral neuropathies. The other<br />

medication choices (naratriptan, naproxen, and cyproheptadine) are used in the acute therapy <strong>of</strong> migraines.<br />

References:<br />

Silberstein S. Practice Parameter: Evidence-based guidelines for migraine headache (an evidence-based review):<br />

Report <strong>of</strong> the Quality Standards Subcommittee <strong>of</strong> the American Academy <strong>of</strong> Neurology. Neurology<br />

2000;55:754-762.<br />

Question 144: Clinical Adult - Neurotoxicology<br />

Discussion:<br />

This patient most likely has been exposed to an anticholinesterase toxin in liquid or gas form. The fact that the<br />

intoxication occurred in a crowded, enclosed, public space suggests terrorism. The clinical findings would be<br />

different with carbon monoxide, nitrous oxide, methane, or cyanide poisoning.<br />

References:<br />

Newmark J. Nerve agents. Neurol Clin 2005; 23(2): 623-641.<br />

Question 153: Clinical Adult - Headache<br />

Discussion:<br />

Paroxysmal hemicrania is a disorder, more common in women, characterized by frequent (7 to 22 per day)<br />

episodes <strong>of</strong> unilateral, severe, but short-lasting (5 to 45 minutes) headaches associated with ipsilateral<br />

autonomic manifestations. Indomethacin is the treatment <strong>of</strong> choice.<br />

References:<br />

Goadsby PJ, Lipton RB. A review <strong>of</strong> paroxysmal hemicranias, SUNCT syndrome and other short-lasting<br />

headaches with autonomic features, including new cases. Brain 1997;120:193-209.<br />

Question 161: Clinical Adult - Neuro-oncology<br />

Discussion:<br />

A seizure with postictal hemiparesis in a patient with known cancer should be presumed to be related to brain<br />

metastases until proven otherwise. Paraneoplastic limbic encephalitis would be much less likely, and<br />

hypertensive encephalopathy, hypoglycemia, and hyponatremia would be unlikely to produce focal deficits.<br />

Moreover, the patient does not have severe enough hypertension, hypoglycemia, or hyponatremia to produce a<br />

seizure.<br />

References:<br />

Noseworthy J. Neurological therapeutics Principles and Practice. 2nd ed. London: Informa Healthcare, 2006.<br />

Question 163: Clinical Adult - Epilepsy<br />

Discussion:<br />

This is a single unprovoked generalized seizure in a patient with a normal neurological examination and a<br />

normal evaluation. The etiology is likely idiopathic, and his risk for recurrent seizure is low (about 30% over the<br />

next 5 years); therefore, observation is warranted.<br />

References:<br />

<strong>2007</strong> <strong>RITE</strong> Discussion & Reference Manual<br />

27


Sirven J. Antiepileptic Drug Therapy for Adults: When to Initiate and How to Choose. Mayo Clinic Proceedings<br />

2002; 77(12):1367-1375.<br />

Marson A, Jacoby A, Johnson A, et al. Medical Research Council MESS Study Group. Immediate versus<br />

deferred antiepileptic drug treatment for early epilepsy and single seizures: a randomized controlled trial. Lancet<br />

2005; 365 (9476):<strong>2007</strong>- 2013.<br />

Question 169: Clinical Adult - Neuro-oncology<br />

Discussion:<br />

This patient has Foster-Kennedy syndrome. Unilateral anosmia and optic neuropathy in conjunction with signs<br />

<strong>of</strong> raised ICP (papilledema in the opposite eye) are the clinical features <strong>of</strong> this disorder, which is most<br />

commonly due to a planum sphenoidale meningioma that could involve both the olfactory groove and optic<br />

canal. The other disorders listed would not typically have this constellation <strong>of</strong> signs and symptoms.<br />

References:<br />

Demetriades AK. Victor Horsley's contribution to the Foster Kennedy syndrome. Brit J Neurosurg<br />

2004;18(4):371-374.<br />

Question 170: Clinical Adult - Spinal and Root Disorders<br />

Discussion:<br />

Saddle anesthesia, sphincter loss, and loss <strong>of</strong> ankle reflexes after a fall signify midline disk herniation with<br />

compression <strong>of</strong> the caudal equina.<br />

References:<br />

Noseworthy J. Neurological therapeutics Principles and Practice. 2nd ed. London: Informa Healthcare, 2006.<br />

Question 172: Clinical Adult - Headache<br />

Discussion:<br />

This patient has a history <strong>of</strong> migraine, a straightforward current history, and a normal neurological examination.<br />

No features suggest elevated ICP or meningeal irritation. No testing is necessary in this setting.<br />

References:<br />

Edmeads J. Migraine. In: Noseworthy JH, editor. Neurologic therapeutics: principles and practice. New York:<br />

Martin Dunitz, 2003;73-88.<br />

Question 173: Clinical Adult - Cerebrovascular Disease<br />

Discussion:<br />

This patient most likely has a vertebral dissection. This disorder is more common is women. It <strong>of</strong>ten follows<br />

neck trauma. When occurring spontaneously, it is <strong>of</strong>ten associated with a disorder <strong>of</strong> connective tissue, such as<br />

fibromuscular dysplasia. The other disorders listed (disseminated intravascular coagulation, factor V Leiden<br />

mutation, patent foramen ovale, and vasculitis) are less likely, particularly in view <strong>of</strong> the associated neck pain.<br />

References:<br />

Noseworthy J. Neurological therapeutics Principles and Practice. 2nd ed. London: Informa Healthcare, 2006.<br />

Question 176: Clinical Adult - Other Pain Syndromes<br />

Discussion:<br />

<strong>2007</strong> <strong>RITE</strong> Discussion & Reference Manual<br />

Allodynia is the perception <strong>of</strong> a non-noxious stimulus as being painful. This is an associated feature common in<br />

neuropathic pain. Incident pain refers to pain that occurs with movement rather than with touch. Paresthesia<br />

28


efers to an abnormal spontaneous sensation that has no objective cause.<br />

References:<br />

Kanner R. Pain management secrets. Philadelphia: Hanley and Belfus, 1997;2.<br />

Question 182: Clinical Adult - Critical Care/Trauma<br />

Discussion:<br />

Practice guidelines have been established for concussion in sports through the AAN. This athlete has<br />

experienced one Grade II concussion (no loss <strong>of</strong> consciousness but concussion symptoms greater than 15<br />

minutes) and a more recent Grade III concussion(any loss <strong>of</strong> consciousness). Guidelines state he should be out<br />

<strong>of</strong> play for 1 week in this setting <strong>of</strong> recurrent concussion.<br />

References:<br />

Practice Parameter: Managing Concussion in Sports. Report <strong>of</strong> the Quality Standards Subcommittee. Neurology<br />

1997;48:581-585.<br />

Question 185: Clinical Adult - Spinal and Root Disorders<br />

Discussion:<br />

Patients with transverse myelitis are at risk, but may not necessarily develop, multiple sclerosis. Presence <strong>of</strong> a<br />

partial rather than complete myelitis gives a higher risk for progression to multiple sclerosis. The presence <strong>of</strong><br />

subclinical evidence <strong>of</strong> demyelination on brain imaging at the time <strong>of</strong> initial event is the strongest predictor <strong>of</strong><br />

subsequent demyelinating disease. Multiple sclerosis remains a clinical diagnosis, and exclusion <strong>of</strong> other genetic<br />

or autoimmune mimics is important.<br />

References:<br />

Lublin F, Tullman M, editors. Multiple Sclerosis. Continuum: Lifelong Learning in Neurol 2004; 10(6).<br />

Question 189: Clinical Adult - Neuromuscular Disorders<br />

Discussion:<br />

Diazepam is the mainstay <strong>of</strong> treatment for the stiff person syndrome, which occurs about as <strong>of</strong>ten in women as<br />

in men. Onset in adult life, proximal distribution <strong>of</strong> stiffness, development <strong>of</strong> lordosis, and precipitation by<br />

motion or emotion are typical. Diabetes and organ-specific autoimmune disorders are common. Seizures<br />

sometimes occur.<br />

References:<br />

Noseworthy J. Neurological therapeutics Principles and Practice. 2nd ed. London: Informa Healthcare, 2006.<br />

Question 190: Clinical Adult - Cerebrovascular Disease<br />

Discussion:<br />

This patient has Anton's syndrome (denial <strong>of</strong> blindness). It is a result <strong>of</strong> severe acute bilateral injury to the<br />

medial occipital lobes and adjacent association cortex, usually due to ischemia in the distribution <strong>of</strong> the<br />

posterior cerebral arteries. The syndrome has also occured after traumatic injury to the optic nerve with<br />

associated bifrontal lobe contusions.<br />

References:<br />

<strong>2007</strong> <strong>RITE</strong> Discussion & Reference Manual<br />

McDaniel KD, McDaniel LD. Anton's syndrome in a patient with posttraumatic optic neuropathy and bifrontal<br />

contusions. Arch Neurol 1991;48 (1):101-105.<br />

Brazis PW, Masdeu JC, Biller J. Localization in clinical neurology. 4th ed. Philadelphia: Lippincott, Williams &<br />

Wilkins, 2001.<br />

29


Question 196: Clinical Adult - Demyelinating Disease<br />

Discussion:<br />

After an episode <strong>of</strong> optic neuritis, the best predictor for subsequent MS is an abnormal MRI <strong>of</strong> the brain. CSF<br />

findings do not add to the prognostic information available from brain MRI data.<br />

References:<br />

Jacobson D. Optic neuritis. In: Samuels MA, Feske SK, editors. Office Practice <strong>of</strong> Neurology. Philadelphia:<br />

Churchill-Livingstone, 2003; 416-420.<br />

Question 200: Clinical Adult - Demyelinating Disease<br />

Discussion:<br />

Mitoxantrone is USFDA-approved for the treatment <strong>of</strong> secondary progressive multiple sclerosis and for<br />

relapsing-progressive multiple sclerosis. The other treatments have no proven benefit for these stages <strong>of</strong> multiple<br />

sclerosis.<br />

References:<br />

AAN Clinical Practice Guidelines. The use <strong>of</strong> mitoxantrone (Novantrone) in the treatment <strong>of</strong> multiple sclerosis.<br />

Neurol 2003; 61: 1332-1338.<br />

Questions 204 - 208: Clinical Adult - Epilepsy<br />

Discussion:<br />

Choice <strong>of</strong> anticonvulsant therapy is based upon effectiveness <strong>of</strong> each drug in the reduction <strong>of</strong> specific seizure<br />

types (ie, generalized vs complex partial vs absence) and also least risk <strong>of</strong> side effects. Lamotrigine has recently<br />

been given a pregnancy category C rating, making it the safest selection for patients with refractory epilepsy in<br />

pregnancy, providing the seizure frequency is a greater risk to the pregnancy and medication risk. Valproic acid<br />

has a particular efficacy in absence seizures and juvenile myoclonic epilepsy. It also has the least risk <strong>of</strong> skin<br />

hypersensitivity. Levetriacetam is considered the safest in the setting <strong>of</strong> liver disease. Phenobarbital is used in<br />

the acute treatment <strong>of</strong> neonatal seizures. Carbamazepine is the initial monotherapy for a child with cryptogenic<br />

complex partial seizures.<br />

References:<br />

Asconape J. Some common issues in the use <strong>of</strong> antiepileptic drugs. Semin Neurol 2002;22:27-40.<br />

Wheless JW, Clarke DF, Carpenter D. Treatment <strong>of</strong> pediatric epilepsy: expert opinion. J Child Neurol 2005; 20<br />

(Suppl 1): S1-S56.<br />

Questions 219 - 223: Clinical Adult - Dementia<br />

Discussion:<br />

A subacute encephalopathy with multiple-enhancing MRI lesions and a high antinuclear antibody titer are<br />

consistent with lupus cerebritis. Creutzfeldt-Jakob disease is a prion disorder that can present subacutely with a<br />

syndrome <strong>of</strong> rapidly progressive dementia, ataxia, and stimulus-sensitive myoclonus. Joint laxity, ectopic lens,<br />

and aortic dissection can be seen as a complication <strong>of</strong> Marfan syndrome, which also carries a typical phenotype<br />

<strong>of</strong> tall stature and long arms. CADASIL is a genetic syndrome producing an arteriopathy that can present with<br />

migraine, dementia, and characteristic white matter infarcts on MRI imaging. Vascular dementia is<br />

distinguished from Alzheimer-type dementia primarily by stepwise decline in cognitive function.<br />

References:<br />

Gallagher C. Neurogenetics Review. Hospital Physician 2005;9(1):1-16.<br />

Question 330: Clinical Adult - Headache<br />

Discussion:<br />

<strong>2007</strong> <strong>RITE</strong> Discussion & Reference Manual<br />

30


This patient most likely has orthostatic headache secondary to spontaneous intracranial hypotension. The usual<br />

etiology is a CSF leak located in the spinal meninges. The leak is most readily identified with a myelogram. The<br />

other tests listed (meningeal biopsy, serum angiotensin converting enzyme, slit lamp examination, and CSF<br />

cytology) would not assist in the diagnosis <strong>of</strong> this syndrome.<br />

References:<br />

Schievink WI. Spontaneous spinal cerebrospinal fluid leaks and intracranial hypotension. JAMA 2006;<br />

295(19):2286-2296.<br />

Question 333: Clinical Adult - Neuromuscular Disorders<br />

Discussion:<br />

The clinical history and examination are most consistent with Lambert-Eaton myasthenic syndrome, which is<br />

associated with malignancy (usually small cell lung cancer) in two thirds <strong>of</strong> patients.<br />

References:<br />

Noseworthy J. Neurological therapeutics Principles and Practice. 2nd ed. London: Informa Healthcare, 2006.<br />

Question 335: Clinical Adult - Infectious Disease<br />

Discussion:<br />

This patient most likely has cerebral malaria, which is due to sludging <strong>of</strong> infected erythrocytes in the cerebral<br />

capillaries. The CSF examination is not consistent with viral encephalitis nor has she been in a geographic area<br />

in which Japanese encephalitis is endemic. A Giemsa-stained peripheral blood smear will demonstrate the<br />

malaria parasite.<br />

References:<br />

Noseworthy J. Neurological therapeutics Principles and Practice. 2nd ed. London: Informa Healthcare, 2006.<br />

Question 337: Clinical Adult - Cerebrovascular Disease<br />

Discussion:<br />

This patient has arteritic acute ischemic optic neuropathy, in which the findings are typically a swollen disc and<br />

retinal pallor. The mechanism is likely to be giant cell arteritis based on his clinical history.<br />

References:<br />

Noseworthy J. Neurological therapeutics Principles and Practice. 2nd ed. London: Informa Healthcare, 2006.<br />

Question 354: Clinical Adult - Headache<br />

Discussion:<br />

This patient has pseudotumor cerebri, which has been linked to use <strong>of</strong> isotretinoin and other vitamin<br />

A–containing compounds. The other medications listed should not cause pseudotumor cerebri.<br />

References:<br />

Wall M. Papilledema and idiopathic intracranial hypertension (pseudotumor cerebri). In: Noseworthy JH, editor.<br />

Neurologic therapeutics: principles and practice. New York: Martin Dunitz, 2003;1730-1742.<br />

Question 361: Clinical Adult - Neuromuscular Disorders<br />

Discussion:<br />

<strong>2007</strong> <strong>RITE</strong> Discussion & Reference Manual<br />

This patients has symptoms and signs consistent with Guillain-Barre syndrome (GBS). IV immunoglobulin is<br />

recommended for GBS in nonambulant patients within 2 to 4 weeks <strong>of</strong> onset <strong>of</strong> neuropathic symptoms.<br />

31


References:<br />

Dubinsky RM, Stein AC, Lyons K. Practice parameter: risk <strong>of</strong> driving and Alzheimer's disease (an<br />

evidence-based review): report <strong>of</strong> the Quality Standards Subcommittee <strong>of</strong> the American Academy <strong>of</strong> Neurology.<br />

Neurology 2000;54(12):2205-2211.<br />

Question 368: Clinical Adult - Headache<br />

Discussion:<br />

This patient has medication overuse headache related to daily use <strong>of</strong> an acetaminophen/caffeine/butalbital<br />

medication. Although the other interventions might help her headache problem, they would be less likely to do<br />

so than tapering the medication.<br />

References:<br />

Boes CJ, Black DF, Dodick DW. Pathophysiology and management <strong>of</strong> transformed migraine and medication<br />

overuse headache. Seminars in Neurology 2006;26(2):232-241.<br />

Question 376: Clinical Adult - Movement Disorders<br />

Discussion:<br />

In patients with Lewy body dementia, severe akinesia, dystonias, and neuroleptic malignant syndrome are<br />

common reactions to even very low doses <strong>of</strong> the older, typical antipsychotics.<br />

References:<br />

Ballard C, Grace J, McKeith I, et al. Neuroleptic sensitivity in dementia with Lewy bodies and Alzheimer's<br />

disease. Lancet 1998;351:1032-1033.<br />

Question 384: Clinical Adult - Spinal and Root Disorders<br />

Discussion:<br />

Disturbance <strong>of</strong> the S1 nerve root results in pain radiating down the posterior aspect <strong>of</strong> the lower extremity,<br />

sensory disturbance <strong>of</strong> the little toe, lateral foot, and most <strong>of</strong> the sole <strong>of</strong> the foot, weakness in several muscles<br />

including the gastrocnemius and soleus muscles, and reduction or absence <strong>of</strong> the ankle jerk.<br />

References:<br />

Brazis PW, Masdeu JC, Biller J. Localization in clinical neurology. 4th ed. Philadelphia: Lippincott, Williams &<br />

Wilkins, 2001.<br />

Question 387: Clinical Adult - Neurogenetics<br />

Discussion:<br />

This patient most likely has Duchenne's muscular dystrophy, which is characterized by X-linked recessive<br />

inheritance and a mutation <strong>of</strong> the dystrophin gene located on Xp21.2<br />

References:<br />

Noseworthy J. Neurological therapeutics Principles and Practice. 2nd ed. London: Informa Healthcare, 2006.<br />

Question 398: Clinical Adult - Sleep<br />

Discussion:<br />

<strong>2007</strong> <strong>RITE</strong> Discussion & Reference Manual<br />

This patient most likely has restless legs syndrome. Medications that are most helpful for this disorder are<br />

dopamine agonists, opioids, and gabapentin. Dopaminergic agents (including pramipexole, ropinarole, and<br />

carbidopa-levadopa) usually work best with fewest side effects. Opioids are not the preferred agents, as they<br />

may produce drug dependency and tolerance, requiring dose escalation to maintain drug effect.<br />

32


References:<br />

Noseworthy J. Neurological therapeutics Principles and Practice. 2nd ed. London: Informa Healthcare, 2006.<br />

Question 403: Clinical Adult - Neurology <strong>of</strong> Systemic Disease<br />

Discussion:<br />

The most common neurological complication <strong>of</strong> scleroderma is carpal tunnel syndrome.<br />

References:<br />

Rolak LA. Neurological complications <strong>of</strong> progressive systemic sclerosis. In: Noseworthy J, editor. Neurological<br />

therapeutics. London: Martin Dunitz, 2003.<br />

Question 406: Clinical Adult - Infectious Disease<br />

Discussion:<br />

This is a case <strong>of</strong> progressive multifocal leukoencephalopathy (PML), caused by the JC virus. Highly active<br />

antiretroviral therapy (HAART) is the only proven treatment <strong>of</strong> HIV-associated PML. Pyrimethamine is used to<br />

treat toxoplasmosis. Amphotericin B is an antifungal agent. The other drugs (AraC and interferon alpha) are not<br />

effective for HIV-associated PML.<br />

References:<br />

AAN Clinical Practice Guidelines. Evaluation and Management <strong>of</strong> Intracranial Mass Lesions in AIDS. Neurol<br />

1998; 50: 21-26.<br />

Question 410: Clinical Adult - Dementia<br />

Discussion:<br />

The patient has many clinical features consistent with a frontal dementia. The trails B test is a measure <strong>of</strong><br />

executive function. It assesses sustained attention, visual scanning, and the ability to change sets (inhibit certain<br />

stimuli while facilitating others). It is a timed task, so speed <strong>of</strong> performance is also important. Finger tapping<br />

assesses motor speed. The Wechsler Memory Scale and California Verbal Learning Test assess memory. The<br />

Boston Naming Test evaluates language.<br />

References:<br />

Bartleson JD, Schierman B, Edlund W, editors. Summaries <strong>of</strong> AAN Clinical Practice Guidelines for Adult<br />

Neurology: Neuropsychological Testing. 2006-2006 ed. American Academy <strong>of</strong> Neurology<br />

Feinberg TE, Farah MJ. Behavioral neurology and neuropsychology. 2nd ed. New York: McGraw-Hill, 2003.<br />

Question 413: Clinical Adult - Demyelinating Disease<br />

Discussion:<br />

Adrenoleukodystrophy is an X-linked disorder characterized by defective beta oxidation and accumulations <strong>of</strong><br />

very-long chain fatty acids. This metabolic defect is detectable in heterozygotes and prenatally. The adult-onset<br />

phenotype (progressive spastic paraparesis) is called adrenomyeloneuropathy.<br />

References:<br />

Moser HW, Raymond GV, Dubey P. Adrenoleukodystrophy: new approaches to a neurodegenerative disease.<br />

JAMA 2005;294(24):3131-3134.<br />

Question 424: Clinical Adult - Neurorehabilitation<br />

Discussion:<br />

<strong>2007</strong> <strong>RITE</strong> Discussion & Reference Manual<br />

Although the pattern <strong>of</strong> recovery following a stroke may be variable, the earliest evidence <strong>of</strong> a return <strong>of</strong><br />

33


neurological function is usually an increase in tone. This generally occurs before improvement in strength or a<br />

return <strong>of</strong> reflexes.<br />

References:<br />

DeLisa JA, Gans BM. Rehabilitation medicine: Principles in Practice. 2nd ed. Philadelphia: Lippincott,<br />

1993;458.<br />

Question 427: Clinical Adult - Spinal and Root Disorders<br />

Discussion:<br />

This represents a Brown-Sequard syndrome with ipsilateral vibration/position sense involvement and<br />

contralateral pain/temperature abnormality below the level <strong>of</strong> the lesion.<br />

References:<br />

Haines DE, editor. Fundamental Neuroscience. 3rd ed. Philadelphia: Churchill Livingstone,2006;156.<br />

Question 428: Clinical Adult - Neurogenetics<br />

Discussion:<br />

Examples <strong>of</strong> X-linked inheritance include Duchenne/Becker muscular dystrophy, Kennedy’s disease, fragile X<br />

syndrome, adrenoleukodystrophy, and Fabry lipid storage disease. Characteristics <strong>of</strong> X-linked inheritance<br />

include that males are almost exclusively affected but symptomatic female carriers can have a mild phenotype as<br />

seen in Duchenne dystrophy. Transmission occurs through carrier females to their sons, each <strong>of</strong> whom have a<br />

50% chance <strong>of</strong> being affected. Daughters <strong>of</strong> affected males are obligate carriers, and male-to-male transmission<br />

does not occur.<br />

References:<br />

Gallagher C. Neurology Board Review Manual: Neurogenetic Review. Hospital Physician 2005; Vol 9: Part 1.<br />

Souayah N, Khella S. Neurology Examination and Board Review. New York:McGraw-Hill, 2005;221-230.<br />

Question 432: Clinical Adult - Neuro-ophthalmology/Neuro-otology<br />

Discussion:<br />

The history and examination is most consistent with benign positional vertigo. The other disorders listed would<br />

be associated with additional neurological deficits and/or the vertigo would be less likely to be associated with<br />

positional changes.<br />

References:<br />

Blumenfeld H. Neuroanatomy through clinical cases. 1st ed. Sunderland: Sinauer Associates, Inc., 2002.<br />

Ropper AH, Brown RH. Adams and Victor's Principles <strong>of</strong> Neurology. 8th ed. New York: McGraw-Hill, 2005;<br />

261-264.<br />

Question 438: Clinical Adult - Motor Neuron/Nerve<br />

Discussion:<br />

This patient most likely has motor neuron disease, characterized by a mixture <strong>of</strong> upper and lower motor neuron<br />

signs in all four limbs. Cervical spinal stenosis could also produce painless four-limb weakness with upper motor<br />

neuron signs in the legs. Fasciculations and wasting in the lower extremities are not symptoms <strong>of</strong> cervical<br />

stenosis as they imply a disorder <strong>of</strong> the lower motor neuron at the lumbosacral level. Although patients with<br />

cervical stenosis may have spine and radicular pain, this is not always the case, and the absence <strong>of</strong> pain does not<br />

exclude cervical stenosis as a diagnosis.<br />

References:<br />

<strong>2007</strong> <strong>RITE</strong> Discussion & Reference Manual<br />

Noseworthy J. Neurological therapeutics Principles and Practice. 2nd ed. London: Informa Healthcare, 2006.<br />

34


Noseworthy J. Neurological therapeutics Principles and Practice. 2nd ed. London: Informa Healthcare, 2006.<br />

Question 441: Clinical Adult - Movement Disorders<br />

Discussion:<br />

A levodopa or apomorphine challenge should be considered when the diagnosis <strong>of</strong> Parkinson’s disease is in<br />

doubt. The other tests listed (olfaction, repeat MRI, PET, and autonomic testing) are not clinically useful in<br />

distinguishing Parkinson’s disease from other parkinsonian syndromes based upon insufficient evidence from<br />

clinical studies.<br />

References:<br />

AAN Clinical Practice Guidelines. Diagnosis and Prognosis <strong>of</strong> New Onset Parkinson Disease. Neurol<br />

2006;66:968-975.<br />

Question 442: Clinical Adult - Cerebrovascular Disease<br />

Discussion:<br />

This patient has suffered a transient ischemic event most likely related to either intracranial or extracranial<br />

atheromatous disease. There is no evidence to suggest a cardiac embolic source. The most appropriate therapy at<br />

this time is an antiplatelet agent. There is no advantage <strong>of</strong> clopidogrel over aspirin. Aspirin is less expensive.<br />

There is no role in this case for either anticoagulation or tissue plasminogen activase (tPA).<br />

References:<br />

Noseworthy J. Neurological therapeutics Principles and Practice. 2nd ed. London: Informa Healthcare, 2006.<br />

Clinical Pediatrics<br />

Question 8: Clinical Pediatrics - Headache<br />

Discussion:<br />

The most important determinant <strong>of</strong> the differential between pseudotumor cerebri and chronic daily headache is<br />

the presence <strong>of</strong> papilledema, which mandates additional testing, including measurement <strong>of</strong> CSF pressure.<br />

References:<br />

Silberstein SD. Wolff's headache. In: Silberstein SD, Lipton RB, Dalessio DJ, editors. Wolff's headache and<br />

other head pain. 7th ed. New York: Oxford University Press, 2001.<br />

Question 13: Clinical Pediatrics - Heritable Metabolic or Degenerative<br />

Discussion:<br />

Maple syrup urine disease (MSUD) is due to a congenital inability to decarboxylate leucine, isoleucine, and<br />

valine. There is an associated severe metabolic acidosis and hypoglycemia. Infants with MSUD develop poor<br />

feeding and lethargy progressing to coma during the first week or two <strong>of</strong> life; examination reveals fluctuating<br />

hypertonicity and opisthotonus. Blood pH and CO2 are typically normal in the aminoacidopathies (such as<br />

phenylketonuria and homocystinuria), galactosemia, and urea cycle defects. Plasma ammonia is elevated in the<br />

urea cycle disorders.<br />

References:<br />

Swaiman KF, Ashwal S, Ferriero DM. Pediatric neurology: principles and practice. 4th ed. Philadephia: Mosby,<br />

2006.<br />

Question 22: Clinical Pediatrics - Learning/Language Disorders<br />

Discussion:<br />

<strong>2007</strong> <strong>RITE</strong> Discussion & Reference Manual<br />

35


This child has tuberous sclerosis complex, an autosomal dominant disorder associated with mutations in the<br />

tuberin and merlin genes. Intellectual outcome is best correlated with seizure control.<br />

References:<br />

Gomez MR, Sampson J, Whittemore V, editors. Tuberous sclerosis complex. 3rd ed. New York: Oxford<br />

University Press, 1999.<br />

Curatolo P, Verdecchia M, Bombardieri R. Vigabatrin for tuberous sclerosis complex. Brain Dev<br />

2001;23(7):649-653.<br />

Question 43: Clinical Pediatrics - Headache<br />

Discussion:<br />

Migraines are more likely to be frontal than unilateral in children. Photophobia and other criteria can be inferred<br />

by the parents' observations, such as pulling curtains. Ibupr<strong>of</strong>en has been demonstrated efficacious in children;<br />

to date oral triptans have not.<br />

References:<br />

AAN Practice parameter: Pharmacological treatment <strong>of</strong> migraine headaches in children and adolescents.<br />

Neurology 2004;63;2215-2224.<br />

Question 44: Clinical Pediatrics - Learning/Language Disorders<br />

Discussion:<br />

Asperger syndrome is classified with the autistic spectrum disorders (pervasive developmental disorders). It<br />

shares the features <strong>of</strong> autistic disorder except that language is spared. Children with Asperger syndrome are <strong>of</strong>ten<br />

referred to as "little pr<strong>of</strong>essors" owing to their extensive knowledge <strong>of</strong> very restricted fields <strong>of</strong> interest.<br />

References:<br />

American Psychiatric Association. Diagnostic and statistical manual <strong>of</strong> mental disorders (DSM IV-TR). 4th ed.<br />

Washington DC: American Psychiatric Association, 2000.<br />

Question 59: Clinical Pediatrics - Disturbances <strong>of</strong> Consciousness<br />

Discussion:<br />

The Presidents Commission recommeded 100% oxygen ventilation for 10 minutes, followed by passive 100%<br />

oxygen for a period long enough to achive a PCO2 <strong>of</strong> 60. The other tests have high incidence <strong>of</strong> false-negative<br />

results.<br />

References:<br />

Swaiman KF, Ashwal S, Ferriero DM. Pediatric neurology: principles and practice. 4th ed. Philadephia: Mosby,<br />

2006.<br />

Question 66: Clinical Pediatrics - Motor Neuron/Neuromuscular<br />

Discussion:<br />

Examination <strong>of</strong> the affected infant is extremely nonspecific. Infants with congenital myotonic dystrophy present<br />

with a variety <strong>of</strong> findings in the newborn period, including hypotonia, contractures, poor suck, respiratory<br />

distress, cardiac arrhythmias, but not characteristic facies or myotonia seen later. Examination <strong>of</strong> the mother is<br />

much more likely to reveal the typical facial features and mild weakness <strong>of</strong> eye closure and neck flexors <strong>of</strong><br />

myotonic dystrophy. DNA testing is definitive, but clinical diagnosis is usually possible with examination <strong>of</strong><br />

mother, not infant.<br />

References:<br />

<strong>2007</strong> <strong>RITE</strong> Discussion & Reference Manual<br />

Swaiman KF, Ashwal S, Ferriero DM. Pediatric neurology: principles and practice. 4th ed. Philadephia: Mosby,<br />

36


Swaiman KF, Ashwal S, Ferriero DM. Pediatric neurology: principles and practice. 4th ed. Philadephia: Mosby,<br />

2006.<br />

Question 81: Clinical Pediatrics - Neurosurgery<br />

Discussion:<br />

At birth the median head circumference <strong>of</strong> a normal term infant is 35 cm. A head circumference <strong>of</strong> 45 cm is<br />

greater than the 98th percentile. In the absence <strong>of</strong> other cerebral malformations, congenital aqueductal stenosis is<br />

the most common cause <strong>of</strong> noncommunicating hydrocephalus. A familial X-linked form accounts for 2% <strong>of</strong><br />

these. Increased intracranial pressure in the context <strong>of</strong> hypoxic-ischemic encephalopathy does not cause<br />

increased head circumference at birth. Klippel-Feil syndrome (fusion <strong>of</strong> the cervical vertebra) may be associated<br />

with Chiari malformation but is a much less common cause <strong>of</strong> hydrocephalus. Choroid plexus papillomas are<br />

typically located in one lateral ventricle and become symptomatic after the perinatal period, usually by<br />

obstructing ventricular outflow. Lissencephaly is typically associated with microcephaly.<br />

References:<br />

Bradley WG, Dar<strong>of</strong>f RB, Fenichel GM, et al, editors. Neurology in clinical practice. 3rd ed. Boston:<br />

Butterworth-Heinemann, 2000.<br />

Question 90: Clinical Pediatrics - Behavioral/Psychiatric<br />

Discussion:<br />

Tourette syndrome is a disorder including motor tics over a year and vocal tics and is <strong>of</strong>ten associated with<br />

attention deficit hyperactivity disorder and/or obsessive-compulsive disorder. Coprolalia is rare and typically<br />

occurs in adolescence or later.<br />

References:<br />

Swaiman KF, Ashwal S, Ferriero DM. Pediatric neurology: principles and practice. 4th ed. Philadephia: Mosby,<br />

2006.<br />

American Psychiatric Association. Diagnostic and statistical manual <strong>of</strong> mental disorders (DSM IV-TR). 4th ed.<br />

Washington DC: American Psychiatric Association, 2000.<br />

Question 120: Clinical Pediatrics - Tumors<br />

Discussion:<br />

Optic gliomas are the most common CNS tumor for children with neur<strong>of</strong>ibromatosis type 1, but other cranial<br />

nerves can be affected by neur<strong>of</strong>ibromas or schwannomas. Astrocytomas, meningiomas, medulloblastomas,<br />

ependymomas, and hamartomas also occur with increased frequency in patients with neur<strong>of</strong>ibromatosis type 1.<br />

References:<br />

Berg BO, editor. Principles <strong>of</strong> child neurology. New York: McGraw-Hill, 1996.<br />

Question 126: Clinical Pediatrics - Motor Neuron/Neuromuscular<br />

Discussion:<br />

The patient has the typical history and findings <strong>of</strong> subacute or acute onset <strong>of</strong> descending paralysis involving<br />

cranial nerves, neck, and shoulder girdle characteristic <strong>of</strong> infant botulism. Sluggish or fatiguable pupils are a<br />

characteristic finding <strong>of</strong> botulism. Nonketotic hyperglycinemia would not usually have a descending paralysis,<br />

nor would pupils be sluggish. Spinal muscular atrophy would likely have proximal weakness but rarely has<br />

ptosis and never has pupillary involvement. Myotonic discharges are not usually seen in infants, even with<br />

myotonic dystrophy. The clinical picture is not one <strong>of</strong> myotonic dystrophy. The child described has findings<br />

suggesting peripheral weakness, not central.<br />

References:<br />

<strong>2007</strong> <strong>RITE</strong> Discussion & Reference Manual<br />

37


Thompson JA, Filloux FM, Van Orman CB, et al. Infant botulism in the age <strong>of</strong> botulism immune globulin.<br />

Neurology 2005; 64:2029-2032.<br />

Question 127: Clinical Pediatrics - Motor Neuron/Neuromuscular<br />

Discussion:<br />

The patient has the typical history and findings <strong>of</strong> subacute or acute onset <strong>of</strong> descending paralysis involving<br />

cranial nerves, neck, and shoulder girdle characteristic <strong>of</strong> infant botulism. Sluggish or fatiguable pupils are a<br />

characteristic finding <strong>of</strong> botulism. Human botulism immunoglobulin has been shown to significantly shorten the<br />

course <strong>of</strong> the condition. Broad-spectrum antibiotics may acutely worsen weakness.<br />

References:<br />

Thompson JA, Filloux FM, Van Orman CB, et al. Infant botulism in the age <strong>of</strong> botulism immune globulin.<br />

Neurology 2005; 64:2029-2032.<br />

Question 138: Clinical Pediatrics - Developmental Disorders<br />

Discussion:<br />

Lack <strong>of</strong> appropriate representative play in a young child presenting with developmental delay strongly suggests a<br />

diagnosis <strong>of</strong> autism.<br />

References:<br />

Swaiman KF, Ashwal S, Ferriero DM. Pediatric neurology: principles and practice. 4th ed. Philadephia: Mosby,<br />

2006.<br />

Question 142: Clinical Pediatrics - Behavioral/Psychiatric<br />

Discussion:<br />

Masturbation is common in infancy and early childhood and <strong>of</strong>ten not recognized. It does not require<br />

intervention or futher testing. Videotapes <strong>of</strong> episodes by parents are <strong>of</strong>ten most helpful in establishing the<br />

diagnosis.<br />

References:<br />

Yang ML, Fullwood E, Goldstein J, Mink J. Masturbation in infancy and early childhood presenting as a<br />

movement disorder: 12 cases and a review <strong>of</strong> the literature. Pediatrics 2005;116:1427-1432.<br />

Question 175: Clinical Pediatrics - Epilepsy<br />

Discussion:<br />

Benign focal epilepsy <strong>of</strong> childhood with centrotemporal sharp waves presents from 2 to 13 years <strong>of</strong> age. About<br />

20% <strong>of</strong> patients only have one seizure and in two thirds <strong>of</strong> the cases seizures are infrequent. Children with this<br />

disorder are neurologically normal and the majority <strong>of</strong> patients outgrow the disorder. Carbemazepine is the<br />

treatment <strong>of</strong> choice if treatment is necessary.<br />

References:<br />

Wyllie, E. The treatment <strong>of</strong> epilepsy, principles and practice. 3rd ed. Philadelphia: Lippincott, Williams &<br />

Wilkins, 2001.<br />

Question 177: Clinical Pediatrics - Heritable Metabolic or Degenerative<br />

Discussion:<br />

Metachromatic leukodystrophy has striking peripheral neuropathy. Rett's syndrome has microcephaly.<br />

Alexander, Canavan, and Tay-Sachs diseases have megaloencephaly and hyperreflexia.<br />

References:<br />

<strong>2007</strong> <strong>RITE</strong> Discussion & Reference Manual<br />

38


Swaiman KF, Ashwal S, Ferriero DM. Pediatric neurology: principles and practice. 4th ed. Philadephia: Mosby,<br />

2006.<br />

Question 183: Clinical Pediatrics - Critical Care/Trauma<br />

Discussion:<br />

Appropriate therapies for refractory status epilepticus include phenobarbital, pentobarbital, or benzodiazepine<br />

drips. The correct dose for phenobarbital loading is 20mg/kg. If a second dose <strong>of</strong> fosphenytoin is given after a<br />

20mg/kg load, it is generally limited to an additional 5 mg/kg. Midazolam and pentobarbital doses are incorrect<br />

in the options. They should be: midazolam 0.2 mg/kg slow IV bolus followed by 0.75 micrograms/kg/min to 10<br />

micrograms/kg/min IV drip; pentobarbital 5 mg/kg to 15 mg/kg IV bolus over 1 hour followed by 0.5 mg/kg/h<br />

to 3.0 mg/kg/h IV drip. Ventilatory assistance is usually necessary, as are vasopressors. Continuous EEG<br />

monitoring is ideal.<br />

References:<br />

Wyllie, E. The treatment <strong>of</strong> epilepsy, principles and practice. 3rd ed. Philadelphia: Lippincott, Williams &<br />

Wilkins, 2001.<br />

Question 184: Clinical Pediatrics - Movement Disorders<br />

Discussion:<br />

The basal globus pallidus lesions produce an "eye <strong>of</strong> the tiger" on MRI, seen with pantothenate kinase<br />

deficiency (PKAN). The globus pallidus lesions are from iron deposition. Huntington's disease produces caudate<br />

atrophy, Niemann-Pick disease may have dystonia and rigidity but does not produce isolated globus pallidus<br />

lesions. Idiopathic torsion dystonia and Sandifer's syndrome have normal MRI. PKAN is no longer called<br />

Hallervorden-Spatz disease because <strong>of</strong> Hallervorden's role in Nazi genocide.<br />

References:<br />

Swaiman KF, Ashwal S, Ferriero DM. Pediatric neurology: principles and practice. 4th ed. Philadephia: Mosby,<br />

2006.<br />

Question 192: Clinical Pediatrics - Headache<br />

Discussion:<br />

The majority <strong>of</strong> children with recurrent headaches with complete clearing between episodes do not need<br />

neuroimaging. The following would mandate neuroimaging in a child presenting with headaches: auras lasting<br />

more than an hour, persistent neurological findings, abnormal neurological examination between headaches,<br />

predominantly occipital headaches, loss <strong>of</strong> vision at headache peak, or recent significant change in headache<br />

pattern.<br />

References:<br />

Silberstein SD. Wolff's headache. In: Silberstein SD, Lipton RB, Dalessio DJ, editors. Wolff's headache and<br />

other head pain. 7th ed. New York: Oxford University Press, 2001.<br />

Question 201: Clinical Pediatrics - Vascular Disorders<br />

Discussion:<br />

Moyamoya syndrome can be primary genetic or secondary to various conditions, including radiation therapy<br />

involving the circle <strong>of</strong> Willis, sickle cell disease, neur<strong>of</strong>ibromatosis, and Down syndrome. Primary moyamoya<br />

syndrome is more frequent in persons <strong>of</strong> Asian descent.<br />

References:<br />

<strong>2007</strong> <strong>RITE</strong> Discussion & Reference Manual<br />

Swaiman KF, Ashwal S, Ferriero DM. Pediatric neurology: principles and practice. 4th ed. Philadephia: Mosby,<br />

2006.<br />

39


Questions 214 - 218: Clinical Pediatrics - Infectious Disease<br />

Discussion:<br />

The most common cause <strong>of</strong> early neonatal sepsis and meningitis is group B beta-hemolytic streptococcus.<br />

Tuberculous meningitis presents insidiously, then in later stages causes severe basilar meningitis, strokes due to<br />

involvement <strong>of</strong> basilar vessels, and hydrocephalus. It is poorly reversible at this stage. Young adults living in<br />

dormitories or barracks are at risk for meningococcal disease. A newborn with microcephaly, organ<br />

involvement, and subependymal calcifications most likely has congenital Cytomegalovirus, although other<br />

congenital infections may occasionally mimic this. A child from a hilly region in the Upper Midwest presenting<br />

during the late summer with fever, headache, stiff neck, and focal seizures has La Crosse encephalitis.<br />

References:<br />

Swaiman KF, Ashwal S, Ferriero DM. Pediatric neurology: principles and practice. 4th ed. Philadephia: Mosby,<br />

2006.<br />

Question 233: Clinical Pediatrics - Developmental Disorders<br />

Discussion:<br />

Aicardi syndrome is an X-linked dominant syndrome, presumed lethal in males, with agenesis <strong>of</strong> corpus<br />

callosum, retinal lacunae, development disabilities, and frequently associated with infantile spasms.<br />

References:<br />

Swaiman KF, Ashwal S, Ferriero DM. Pediatric neurology: principles and practice. 4th ed. Philadephia: Mosby,<br />

2006.<br />

Question 240: Clinical Pediatrics - Headache<br />

Discussion:<br />

The lesion is typical <strong>of</strong> the acute inflammatory phase <strong>of</strong> neurocysticercosis. In the United States, most children<br />

with cysticercosis live in households with recent immigrants, but many are lifelong US residents. Most<br />

neurocysticercosis lesions will regress without treatment. Anticonvulsants are needed for several months, but<br />

only about 25% ultimately have chronic epilepsy requiring ongoing treatment. CT is generally adequate for<br />

diagnosis. Nearly all single lesion neurocysticercosis resolves spontaneously in 3 to 6 months.<br />

References:<br />

Ong S, Talan DA, Moran GJ, et al. Neurocysticercosis in radiologically imaged seizure patients in U.S.<br />

emergency departments. Emerg Infect Dis 2002;8:608-613.<br />

Question 323: Clinical Pediatrics - Epilepsy<br />

Discussion:<br />

The seizure pattern described is one <strong>of</strong> benign rolandic epilepsy, for which the EEG signature is<br />

central-temporal spikes, particularly in sleep, <strong>of</strong>ten bilateral independent.<br />

References:<br />

Swaiman KF, Ashwal S, Ferriero DM. Pediatric neurology: principles and practice. 4th ed. Philadephia: Mosby,<br />

2006.<br />

Question 345: Clinical Pediatrics - Neonatal<br />

Discussion:<br />

<strong>2007</strong> <strong>RITE</strong> Discussion & Reference Manual<br />

MRI and, when available, diffusion-weighted imaging and magnetic resonance spectroscopy gives the most<br />

information on outcome <strong>of</strong> a newborn with persistent low Apgar scores and neurological complications.<br />

40


References:<br />

Ment LR, Bada HS, Barnes P, et al. Practice parameter: Neuroimaging <strong>of</strong> the neonate. Neurology<br />

2002;58:1726-1738.<br />

Question 346: Clinical Pediatrics - Neonatal<br />

Discussion:<br />

The drop in hematocrit may indicate an intracranial bleed. Compared to MRI, a CT scan without contrast is<br />

safest and will detect most bleeds. EEG is not indicated, and contrast will add little and may pose a risk because<br />

<strong>of</strong> age and systemic illness.<br />

References:<br />

Ment LR, Bada HS, Barnes P, et al. Practice parameter: Neuroimaging <strong>of</strong> the neonate. Neurology<br />

2002;58:1726-1738.<br />

Question 350: Clinical Pediatrics - Movement Disorders<br />

Discussion:<br />

Clusters <strong>of</strong> head drops with flexor spasms <strong>of</strong> the trunk occuring around meal time in an otherwise healthy<br />

7-month-old with a normal EEG suggest benign myoclonus <strong>of</strong> infancy. The normal EEG effectively rules out<br />

infantile spasms. Benign occipital epilepsy <strong>of</strong> childhood, Lafora body disease, and Tourette syndrome occur in<br />

older children.<br />

References:<br />

Bradley WG, Dar<strong>of</strong>f RB, Fenichel GM, et al, editors. Neurology in clinical practice. 3rd ed. Boston:<br />

Butterworth-Heinemann, 2000.<br />

Question 366: Clinical Pediatrics - Inflammatory Disease<br />

Discussion:<br />

Fever is present in 90% <strong>of</strong> children with systemic lupus erythematosus, and about the same number have joint<br />

involvement. It is more common in African Americans than whites or Hispanics. Seizures and cranial<br />

neuropathies are common presenting features. Acute encephalopathy is less common and is life-threatening.<br />

Sickle cell disease usually presents at a younger age. Lyme disease rarely causes seizures or confusion and has a<br />

different rash. MELAS and moyamoya syndrome ususally present with isolated infartions or transient ischemic<br />

attacks and do not have systemic symptoms.<br />

References:<br />

Bradley WG, Dar<strong>of</strong>f RB, Fenichel GM, et al, editors. Neurology in clinical practice. 3rd ed. Boston:<br />

Butterworth-Heinemann, 2000.<br />

Question 382: Clinical Pediatrics - Spine<br />

Discussion:<br />

Hallmarks <strong>of</strong> Friedreich's ataxia are a combination <strong>of</strong> weakness, ataxia, absent tendon jerks, Babinski sign, and<br />

abnormal position sense. Romberg sign is the best indication <strong>of</strong> loss <strong>of</strong> position sense, not cerebellar<br />

functioning. Hammer toes, pes cavus deformities, and scoliosis are frequent findings.<br />

References:<br />

Swaiman KF, Ashwal S, Ferriero DM. Pediatric neurology: principles and practice. 4th ed. Philadephia: Mosby,<br />

2006.<br />

Question 383: Clinical Pediatrics - Neonatal<br />

<strong>2007</strong> <strong>RITE</strong> Discussion & Reference Manual<br />

41


Discussion:<br />

Neonatal seizures usually have a serious underlying reason. Treatable causes need to be recognized since delay<br />

in treatment can aggrevate serious neurological comprimise. Group B Streptococcus is the most common cause<br />

<strong>of</strong> bacterial meningitis/septicemia in the neonate and has about a 25% mortality rate; <strong>of</strong> the survivors, about<br />

20% will have severe deficits. If seizures in the neonate are secondary to subarachnoid hemorrhage, there<br />

usually is a good outcome. Benign epileptic syndromes and electrolyte disturbances (hypocalcemia or<br />

hypomagnesemia) in the newborn can generally present by the end <strong>of</strong> the first week, but infants are not febrile.<br />

References:<br />

Volpe JJ. Neurology <strong>of</strong> the newborn. 4th ed. Philadelphia: WB Saunders, 2001.<br />

Question 386: Clinical Pediatrics - Vascular Disorders<br />

Discussion:<br />

Cerebrovascular disease occurs in 25% <strong>of</strong> sickle cell disease. Eighty percent <strong>of</strong> events occur at or before age 15,<br />

and most are caused by a progressive cerebral vasculopathy that can be partially arrested by chronic transfusion<br />

therapy to keep hemoglobin S below 30%. Most events are thrombotic.<br />

References:<br />

Swaiman KF, Ashwal S, Ferriero DM. Pediatric neurology: principles and practice. 4th ed. Philadephia: Mosby,<br />

2006.<br />

Question 396: Clinical Pediatrics - Disturbances <strong>of</strong> Consciousness<br />

Discussion:<br />

Lead poisoning typically presents in young children living in substandard housing with peeling paint. However,<br />

children living in environments where paint is being removed or scraped may be at risk, even in upper-income<br />

environments. The other toxins, which are sometimes ingested by toddlers, do not present with increased<br />

intracranial pressure.<br />

References:<br />

Nordin J, Rolnick S, Ehlinger E, et al. Lead Levels in High-risk and Low-risk Young Children in the<br />

Minneapolis-St Paul Metropolitan Area. Pediatrics 1998;101:72-76.<br />

Question 409: Clinical Pediatrics - Developmental Disorders<br />

Discussion:<br />

Cerebral palsy is nonprogressive disease, and the study with the highest yield is MRI. The proper diagnosis <strong>of</strong><br />

cerebral palsy excludes progressive or degenerative disorders. If the MRI is normal or nondiagnostic, further<br />

tests could be ordered. An EEG should be obtained only with a positive history <strong>of</strong> seizurelike events.<br />

References:<br />

Ashwal S, Russman B, Blasco P, et al. Practice Parameter: Diagnostic assessment <strong>of</strong> the child with cerebral<br />

palsy: Report <strong>of</strong> the Quality Standards Subcommittee <strong>of</strong> the American Academy <strong>of</strong> Neurology. Neurology<br />

2004;62;851-863.<br />

Question 414: Clinical Pediatrics - Headache<br />

Discussion:<br />

Caffeine withdrawal is a common cause <strong>of</strong> acute severe headache in migraineurs. Vitamins D or A can cause<br />

pseudotumor cerebri, but onset is not abrupt. B vitamins are not associated with exacerbation <strong>of</strong> migraine.<br />

References:<br />

<strong>2007</strong> <strong>RITE</strong> Discussion & Reference Manual<br />

Silberstein SD. Wolff's headache. In: Silberstein SD, Lipton RB, Dalessio DJ, editors. Wolff's headache and<br />

42


other head pain. 7th ed. New York: Oxford University Press, 2001.<br />

Question 415: Clinical Pediatrics - Headache<br />

Discussion:<br />

This is a classic presentation <strong>of</strong> benign paroxysmal vertigo, a syndrome that occurs in children in the first 5<br />

years <strong>of</strong> life and is a frequent precursor <strong>of</strong> migraine. Investigations (MRI, EEG) are normal. The episodes<br />

usually resolve by the end <strong>of</strong> the first decade.<br />

References:<br />

Bradley WG, Dar<strong>of</strong>f RB, Fenichel GM, et al, editors. Neurology in clinical practice. 3rd ed. Boston:<br />

Butterworth-Heinemann, 2000.<br />

Question 417: Clinical Pediatrics - Epilepsy<br />

Discussion:<br />

The 12-year-old boy has a characteristic history for juvenile myoclonic epilepsy with early-morning myoclonic<br />

and generalized tonic-clonic seizures, beginning in the second decade <strong>of</strong> life. Absence seizures may also occur.<br />

The EEG reveals generalized, fast (4 Hz to 6 Hz) spike-and-wave complexes. Valproic acid is the treatment <strong>of</strong><br />

choice and recurrence <strong>of</strong> seizures is likely to occur if treatment is stopped.<br />

References:<br />

Menkes JH, Sarnat HB. Textbook <strong>of</strong> child neurology. 6th ed. Philadelphia: Lippincott, Williams & Wilkins,<br />

2000.<br />

Question 418: Clinical Pediatrics - Epilepsy<br />

Discussion:<br />

This 12-year-old boy has a characteristic history for juvenile myoclonic epilepsy with early-morning myoclonic<br />

and generalized tonic clonic seizures, beginning in the second decade <strong>of</strong> life. Absence seizures may also occur.<br />

The EEG reveals generalized, fast (4 Hz to 6 Hz) spike-and-wave complexes. Valproic acid is the treatment <strong>of</strong><br />

choice and recurrence <strong>of</strong> seizures is likely to occur if treatment is stopped.<br />

References:<br />

Menkes JH, Sarnat HB. Textbook <strong>of</strong> child neurology. 6th ed. Philadelphia: Lippincott, Williams & Wilkins,<br />

2000.<br />

Question 429: Clinical Pediatrics - Disturbances <strong>of</strong> Consciousness<br />

Discussion:<br />

Breath-holding spells occur in infants, usually triggered by sudden pain or frustration. Consciousness is lost<br />

prior to variable brief clonic jerking. These are common and occur in up to 5% <strong>of</strong> infants.<br />

References:<br />

Stephenson JPB. Fits and Faints. Philadelphia: JB Lippincott Co., 1990.<br />

Question 437: Clinical Pediatrics - Epilepsy<br />

Discussion:<br />

This is a typical presentation for infantile spasms. Adrenocorticotropic hormone (ACTH) is an accepted<br />

treatment. Colic or reflux are <strong>of</strong>ten mistaken for infantile spasms, although flexor spasms are not at all typical<br />

for reflux. Phenobarbital has not been shown effective in treatment and may worsen the disorder.<br />

References:<br />

<strong>2007</strong> <strong>RITE</strong> Discussion & Reference Manual<br />

43


Wyllie, E. The treatment <strong>of</strong> epilepsy, principles and practice. 3rd ed. Philadelphia: Lippincott, Williams &<br />

Wilkins, 2001.<br />

Practice parameter: Medical treatment <strong>of</strong> infantile spasms: Report <strong>of</strong> the American Academy <strong>of</strong> Neurology and<br />

the Child Neurology Society. Neurology 2004;62:1668-1681.<br />

Question 439: Clinical Pediatrics - Motor Neuron/Neuromuscular<br />

Discussion:<br />

The congenital myasthenic syndromes are not related to an immune process but are caused by genetic defects<br />

affecting the neuromuscular junction. These include defects in acetylcholine synthesis and packaging (familial<br />

infantile myasthenia gravis), end-plate deficiency <strong>of</strong> acetylcholinesterase, acetylcholine receptor deficiency, and<br />

the slow channel syndrome.<br />

References:<br />

Dubowitz V. Muscle disorders in childhood. Philadelphia: WB Saunders, 1995.<br />

Question 443: Clinical Pediatrics - Disturbances <strong>of</strong> Consciousness<br />

Discussion:<br />

The child's presentation (opsoclonus, myoclonus, ataxia syndrome, alternatively called "OMS", "dancing eyes<br />

syndrome" or Kinsbourne's Syndrome) is most <strong>of</strong>ten a paraneoplastic manifestation <strong>of</strong> occult neuroblastoma in<br />

this age group. The most common initial misdiagnosis is acute cerebellar ataxia <strong>of</strong> childhood, a benign<br />

condition. The most helpful test in determining etiology <strong>of</strong> suspected OMS is CT <strong>of</strong> the chest, abdomen, and<br />

pelvis using oral contrast and fine cut images.<br />

References:<br />

Mitchell WG, Davalos-Gonzales Y, Brumm V, et al. Opsoclonus-ataxia caused by childhood neuroblastoma:<br />

Developmental and neurological sequelae. Pediatrics, 2002;109:86-98.<br />

Contemporary Issues<br />

Question 6: Contemporary Issues - Ethics<br />

Discussion:<br />

Physicians and other health care pr<strong>of</strong>essionals must report suspected cases <strong>of</strong> child abuse. It is not the<br />

physician's obligation to prove the abuse, and testing to rule out other etiologies should not postpone reporting.<br />

Glutaric aciduria, Menkes disease, and hemophilia may present with intracranial or subdural bleeding but are<br />

less likely in this circumstance. Retinal hemorrhages are unusual in these conditions, however. Physicians are<br />

protected from liability for reporting in good faith, even if the allegation is ultimately not supported.<br />

References:<br />

Swaiman KF, Ashwal S, Ferriero DM. Pediatric neurology: principles and practice. 4th ed. Philadephia: Mosby,<br />

2006.<br />

Question 37: Contemporary Issues - Ethics<br />

Discussion:<br />

<strong>2007</strong> <strong>RITE</strong> Discussion & Reference Manual<br />

Requests to lie to patients about their diagnoses arise from time to time. Some are based on the patient and<br />

family's cultural traditions, and some are based on fear <strong>of</strong> consequences. In this case, the son's request is based<br />

on fear <strong>of</strong> depression and is not a sufficient justification to lie to the patient. Telling the patient her son asked that<br />

her diagnosis should be withheld inappropriately undermines trust between the physician and the son, and<br />

between the son and his mother. To honor the son's request while later telling the patient her diagnosis by<br />

telephone consitutes lying to the son and is inappropriate. The AAN Code <strong>of</strong> Pr<strong>of</strong>essional Conduct recommends<br />

telling the truth to patients who have decision-making capacity. AAN guidelines on humanistic dimension <strong>of</strong><br />

44


pr<strong>of</strong>essionalism in the practice <strong>of</strong> neurology state that neurologists have a role in helping patients and families<br />

overcome their fears <strong>of</strong> neurological disorders. Truth-telling allows patients with probable Alzheimer's disease<br />

to seek appropriate treatment and to make arrangements for their treatment preferences (eg, end-<strong>of</strong>-life care<br />

planning) and to select their surrogate decision maker. Not all patients want to know everything about their<br />

diagnosis and treatment, and some may wish to have their family handle this information and decisions for<br />

them. Asking the patient in advance how much information she wishes to receive respects her autonomy by<br />

allowing her to make a choice.<br />

References:<br />

Donatelli LA, Geocadin RG, Williams MA. Ethical issues in critical care and cardiac arrest: clinical research,<br />

brain death, and organ donation. Semin Neurol 2006;26:452–460.<br />

Ethics, Law and Humantities Committee, American Academy <strong>of</strong> Neurology. Humanistic dimension <strong>of</strong><br />

pr<strong>of</strong>essionalism in the practice <strong>of</strong> neurology. Neurology 2001; 56:1261-1263<br />

Question 48: Contemporary Issues - HIPAA<br />

Discussion:<br />

HIPAA regulations regarding patients' protected health information (PHI) are important to safeguard patients'<br />

privacy and confidentiality. HIPAA regulations are not intended to interfere with the legitimate exchange <strong>of</strong> PHI<br />

between health pr<strong>of</strong>essionals caring for patients, as delay <strong>of</strong> information sharing could result in patient harm.<br />

The United States Department <strong>of</strong> Health and Human Services Office <strong>of</strong> Civil Rights states that "(physicians)<br />

may disclose protected health information for the treatment activities <strong>of</strong> any health care provider...." For<br />

example, a primary care provider may send a copy <strong>of</strong> an individual’s medical record to a specialist who needs<br />

the information to treat the individual, or vice versa. In addition, the circumstance need not be a medical<br />

emergency, nor is there a requirement that the request be made in writing as long as copies <strong>of</strong> the records are<br />

being sent for the purpose <strong>of</strong> patient care.<br />

References:<br />

United States Department <strong>of</strong> Health and Human Services Office for Civil Rights--HIPAA. Standards for Privacy<br />

<strong>of</strong> Individually Identifiable Health Information [45 CFR Parts 160 and 164].<br />

http://www.hhs.gov/ocr/hipaa/guidelines/guidanceallsections.pdf.<br />

Question 79: Contemporary Issues - Driving<br />

Discussion:<br />

Driving is mildly impaired in drivers with probable Alzheimer's disease (AD) at a severity <strong>of</strong> Clinical Dementia<br />

Rating (CDR) 0.5. Patients and their families should be told that patients with possible AD with a severity <strong>of</strong><br />

CDR 0.5 pose a significant traffic safety problem when compared to other elderly drivers. Referral <strong>of</strong> the patient<br />

for a driving performance evaluation by a qualified examiner should be considered. Drivers with AD at a<br />

severity <strong>of</strong> CDR 1 pose a significant traffic safety problem both from crashes and from driving performance<br />

measurements. Patients and their families should be told that patients with AD with a severity <strong>of</strong> CDR 1 or<br />

greater have a substantially increased accident rate and driving performance errors, and, therefore,<br />

discontinuation <strong>of</strong> driving should be strongly considered.<br />

References:<br />

Dubinsky RM, Stein AC, Lyons K. Practice parameter: risk <strong>of</strong> driving and Alzheimer's disease (an<br />

evidence-based review): report <strong>of</strong> the Quality Standards Subcommittee <strong>of</strong> the American Academy <strong>of</strong> Neurology.<br />

Neurology 2000;54(12):2205-2211.<br />

Question 83: Contemporary Issues - Ethics<br />

Discussion:<br />

<strong>2007</strong> <strong>RITE</strong> Discussion & Reference Manual<br />

In the United States, the laws that govern withdrawal <strong>of</strong> artificial hydration and nutrition (AHN) are established<br />

by the individual states, territories, and District <strong>of</strong> Columbia. There is significant variation in the laws regarding<br />

the withdrawal <strong>of</strong> AHN in the absence <strong>of</strong> an advance directive, with state laws categorized as permissive and<br />

45


estrictive. In more than two thirds <strong>of</strong> the states, it is settled that surrogate decision makers are allowed to decide<br />

to withdraw AHN without review and approval by a judge if the patient is in PVS and there is no AD. On the<br />

other hand, in the presence <strong>of</strong> an advance directive (AD), state laws are consistent in permitting withdrawal <strong>of</strong><br />

AHN if it is directed in the AD. There are no laws that specify the nature <strong>of</strong> diagnostic testing that must be<br />

performed to confirm the diagnosis <strong>of</strong> PVS.<br />

References:<br />

North Dakota Century Code. 23-06.5 Health Care Directives. http://www.legis.nd.gov/cencode/t23c065.pdf.<br />

Larriviere D, Bonnie RJ. Terminating artificial nutrition and hydration in persistent vegetative state patients:<br />

current and proposed state laws. Neurology 2006;66;1624-1628.<br />

Question 152: Contemporary Issues - Ethics<br />

Discussion:<br />

Huntington's disease is an autosomal dominant trait with complete penetrance, and <strong>of</strong>fspring from affected<br />

patients have a 50% chance <strong>of</strong> developing the disease. Most patients become symptomatic in the third or fourth<br />

decade. Presymptomatic testing for Huntington's disease should be performed only voluntarily at the request <strong>of</strong><br />

an at-risk patient. Genetic testing should be accompanied by pretest and posttest counseling. Guidelines are<br />

available and emphasize voluntariness, confidentiality, safety, absence <strong>of</strong> coercion, and availability <strong>of</strong><br />

counseling.<br />

References:<br />

Case adapted from American Academy <strong>of</strong> Neurology Ethics, Law and Humanities Committee. Ethical<br />

Dimensions <strong>of</strong> Neurologic Practice: A case-based curriculum for neurology residents. March 2000.<br />

Went L. Ethical issues policy statement on Huntington's disease molecular genetics predictive test. International<br />

Huntington Association. World Federation <strong>of</strong> Neurology. J Med Genet 1990:27;34-38.<br />

Question 171: Contemporary Issues - End-<strong>of</strong>-Life/Palliative Care<br />

Discussion:<br />

Organ donation most commonly occurs in the context <strong>of</strong> severe brain injury and brain death. Public awareness<br />

and support <strong>of</strong> organ donation and transplantation has increased in the last decade. As a result, families <strong>of</strong>ten<br />

recognize that the patient's circumstances may be such that organ donation is a possibility. The family's request is<br />

not illegal, nor does it indicate they are inappropriate decision makers for the patient because the patient has<br />

previously indicated the desire to be an organ donor. While it is usually considered improper for physicians to<br />

raise the topic <strong>of</strong> organ donation with a family before brain death has been determined, it is proper to respond to<br />

family requests about organ donation whenever they are made. Organ donation can occur even for "coroner's<br />

cases" as the coroner can be in the operating room at the time the organs are removed for transplantation.<br />

Because patients must be evaluated for medical suitability for organ donation, and because conversations with<br />

families about organ donation require considerable skill, physicians should work closely with representatives <strong>of</strong><br />

organ procurement organizations when organ donation is considered a possibility.<br />

References:<br />

Williams MA, Lipsett PA, Rushton CH, et al. The physician's role in discussing organ donation with families.<br />

Crit Care Med 2003;31(5):1568-1573.<br />

Question 188: Contemporary Issues - Core Competencies<br />

Discussion:<br />

<strong>2007</strong> <strong>RITE</strong> Discussion & Reference Manual<br />

Medical errors occur frequently and are currently thought to result from systems errors. The AAN Patient Safety<br />

Committee web page (http://www.aan.com/pr<strong>of</strong>essionals/patient/patient_saf.cfm) <strong>of</strong>fers patient safety tips as<br />

well as a link to the 2006 Patient Safety Colloquium syllabus, which addresses many <strong>of</strong> the issues in this test<br />

question (http://www.aan.com/pr<strong>of</strong>essionals/patient/pdfs/colloquium_syllabi_06.pdf). The AAN, the American<br />

Medical Association, and other societies urge that errors should be disclosed to patients under most<br />

circumstances and should include an apology and a promise to investigate the causes <strong>of</strong> the error. Although in<br />

46


circumstances and should include an apology and a promise to investigate the causes <strong>of</strong> the error. Although in<br />

this case the patient was not harmed, the additional laboratory tests and extra hospital day are serious<br />

consequences <strong>of</strong> the error that justify the disclosure. The purpose <strong>of</strong> disclosure is not to place blame on<br />

individuals, but rather to inform the patient <strong>of</strong> the events. Errors should be disclosed by persons with the<br />

greatest experience, and those with responsiility for the patient's care--in this case, the attending physician. It is<br />

inappropriate to ask a resident to make an error disclosure alone, although it would be appropriate for the<br />

resident to be present with the attending physician when the error is disclosed. It is inappropriate to expect any<br />

health care pr<strong>of</strong>essional will never make an error; instead, it is important to acknowledge that we all committ<br />

errors and have an obligation to investigate errors to prevent future recurrence. Failure to disclose the error for<br />

fear <strong>of</strong> malpractice lawsuit is inappropriate, as it constitutes concealment.<br />

References:<br />

Patient Safety Colloquium. http://www.aan.com/pr<strong>of</strong>essionals/patient/pdfs/colloquium_syllabi_06.pdf<br />

Institute for Safe Medical Practices. List <strong>of</strong> error-prone abbreviations, symbols and dose designations.<br />

http://www.ismp.org/tools/errorproneabbreviations.pdf.<br />

Question 191: Contemporary Issues - End-<strong>of</strong>-Life/Palliative Care<br />

Discussion:<br />

Although advances in understanding the pathophysiology <strong>of</strong> ALS have stimulated the development <strong>of</strong> new drug<br />

therapies, the mainstay <strong>of</strong> treatment for ALS patients remains symptomatic management. The AAN Practice<br />

Parameter on the care <strong>of</strong> the patient with ALS recommends frequent discussions <strong>of</strong> the goals <strong>of</strong> care because<br />

decision making may be subject to change as the disease becomes more severe. Between 40% and 73% <strong>of</strong><br />

patients experience pain in later stages <strong>of</strong> ALS, and AAN guidelines recommend use <strong>of</strong> opioids when<br />

non-narcotic treatment fails. While the use <strong>of</strong> carefully titrated benzodiazepines to alleviate anxiety and<br />

respiratory distress is indicated, the use <strong>of</strong> neuromuscular blockade is not supported by AAN guidelines. Few<br />

patients on long-term invasive ventilation regret their choice, but because patient satisfaction is higher with<br />

noninvasive than invasive ventilation, noninvasive ventilation should be considered first.<br />

References:<br />

Quality Standards Subcommittee <strong>of</strong> the American Academy <strong>of</strong> Neurology. Practice parameter: the care <strong>of</strong> the<br />

patient with amyotrophic lateral sclerosis. Neurology 1999; 52:1311-1323.<br />

http://www.neurology.org/cgi/reprint/52/7/1311.pdf<br />

Question 332: Contemporary Issues - Driving<br />

Discussion:<br />

Determining the ability to drive following stroke is a difficult task. Numerous studies have been performed<br />

suggesting the most reliable determinant <strong>of</strong> driving ability is the roadside test. The simulated driving test and the<br />

roadside test are more predictive than neuropsychological measures and neurologist opinion.<br />

References:<br />

Akinwuntan AE, Feys H, De Weerdt W, et al. Prediction <strong>of</strong> driving after stroke: a prospective study.<br />

Neurorehabil Neural Repair 2006;20:417-423.<br />

Akinwuntan AE, De Weerdt W, Feys H, et al. The validity <strong>of</strong> a road test after stroke. Arch Phys Med Rehabil<br />

2005;86:421-426.<br />

Question 359: Contemporary Issues - Core Competencies<br />

Discussion:<br />

<strong>2007</strong> <strong>RITE</strong> Discussion & Reference Manual<br />

The American Medical Association has recently published guidelines regarding the ethical use <strong>of</strong><br />

industry-sponsored funds for social gatherings and other <strong>of</strong>ferings provided by pharmaceutical companies to<br />

physicians in the interest <strong>of</strong> endorsing products. Pharmaceutical companies are restricted from providing strictly<br />

meals or entertainment to physicians or their families. It is appropriate to accept educational grants from<br />

pharmaceutical companies if the source is disclosed, the educational content and speakers are selected by the<br />

47


pharmaceutical companies if the source is disclosed, the educational content and speakers are selected by the<br />

program director, and the presentation is free from bias.<br />

References:<br />

Accreditation Council for Continuing Medical Education. The Standards for Commercial Support.<br />

http://www.accme.org/dir_docs/doc_upload/ 69466f45-ca31-4cc0-b88e-a38d10de5ee2_uploaddocument.doc<br />

Question 372: Contemporary Issues - Business<br />

Discussion:<br />

When choosing an ICD-9-CM diagnosis code for a diagnostic test, the code most consistent with the test result<br />

or confirmed diagnosis should be used. A principle rule for choosing an ICD-9-CM code is to code to the highest<br />

level <strong>of</strong> specificity. Based on the history and clinical findings, the most specific choice is carpal tunnel<br />

syndrome. Codes that describe symptoms and signs as opposed to diagnoses are acceptable for reporting<br />

purposes when a related definitive diagnosis has not been established or confirmed. V-codes are considered by<br />

Medicare as "Supplementary Classification <strong>of</strong> Factors Influencing Health Status and Contact With Health<br />

Services” and are not used for diagnoses.<br />

References:<br />

CMS. ICD-9-CM Official Guidelines for Coding and Reporting. Available online at:<br />

http://www.cdc.gov/nchs/datawh/ftpserv/ftpicd9/icdguide05.pdf<br />

Question 408: Contemporary Issues - Core Competencies<br />

Discussion:<br />

In the NICU, implied consent to medical treatment is suitable in emergency circumstances, as patients would<br />

otherwise be harmed by the delay needed to obtain informed consent from a surrogate. However, the concept <strong>of</strong><br />

implied consent for emergency treatment cannot be expanded to apply to other procedures or interventions<br />

performed on the same patient in the absence <strong>of</strong> an emergency. The tracheostomy and percutaneous endoscopic<br />

gastrostomy (PEG), while necessary, are elective and not emergency procedures, and informed consent from a<br />

surrogate decision maker is necessary. The hospital attorney does not have the legal authority to approve<br />

treatment decisions, and, as a representative <strong>of</strong> the hospital's interests, cannot appropriately advocate for the<br />

patient's interests. Life-sustaining therapies cannot be withdrawn on the basis <strong>of</strong> the ICU teams' decision alone.<br />

Therefore a surrogate decision maker must be found. The ethics consultation team can serve to represent the<br />

patient's interests and decide whether the treatment recommended by the NICU team is ethically permissible. A<br />

court <strong>of</strong> law would take the ethics consultation team's recommendations seriously when approving a guardian<br />

for the patient. The NICU team may make the same recommendations to the ethics consultation team or<br />

guardian that would be made to a family in a similar situation, including either withdrawal <strong>of</strong> life-sustaining<br />

therapies or insertion <strong>of</strong> a tracheostomy and PEG.<br />

References:<br />

Donatelli LA, Geocadin RG, Williams MA. Ethical issues in critical care and cardiac arrest: clinical research,<br />

brain death, and organ donation. Semin Neurol 2006;26:452–460.<br />

Question 431: Contemporary Issues - End-<strong>of</strong>-Life/Palliative Care<br />

Discussion:<br />

In emergency circumstances where irreversible harm or death would occur by withholding requested treatment,<br />

it is best to initiate the treatment and then seek consultation from an ethics committee. An ethics consultation can<br />

rarely be done immediately as there would be insufficient time to gather the information necessary to <strong>of</strong>fer a<br />

recommendation. Refusals to treat on the basis <strong>of</strong> medical futility require a due process <strong>of</strong> evaluation <strong>of</strong> the<br />

patient's interests, the family's wishes, and the medical facts <strong>of</strong> the case, including prognosis. Medical futility<br />

cannot be determined unilaterally without ethics consultation. Anencephaly is not equivalent to brain death, as<br />

there is function <strong>of</strong> the brain stem and there are respiratory efforts.<br />

References:<br />

<strong>2007</strong> <strong>RITE</strong> Discussion & Reference Manual<br />

48


Anomynous. In the matter <strong>of</strong> baby "K" No. 93-1899, No. 93-1923, No. 93-1924. United States Court <strong>of</strong> Appeals<br />

16F.3d590; 1994 US app. (LEXIS 2215)<br />

Question 434: Contemporary Issues - HIPAA<br />

Discussion:<br />

The HIPAA Privacy Rule at 45 CFR 164.510(b) specifically permits physicians to share information that is<br />

directly relevant to the involvement <strong>of</strong> a spouse, family members, friends, or other persons identified by a<br />

patient in the patient’s care or payment for health care. If the patient is present or is otherwise available prior to<br />

the disclosure and has the capacity to make health care decisions, physicians may discuss this information with<br />

the family and these other persons if the patient agrees or, when given the opportunity, does not object.<br />

Physicians may also share relevant information with the family and these other persons if they can reasonably<br />

infer, based on pr<strong>of</strong>essional judgment, that the patient does not object. In this case, the neurologist does not have<br />

sufficient information to infer whether the patient would allow this information to be shared with her husband,<br />

and it is better to ask the patient's permission first. Information can be shared in medical emergencies; however,<br />

weight gain is not an emergency. HIPAA regulations apply equally to specialists and primary care physicians,<br />

and HIPAA regulations apply equally to verbal and written communication.<br />

References:<br />

United States Department <strong>of</strong> Health and Human Services. Health Information Privacy and Civil Rights<br />

Questions & Answers; Disclosure to Family and Friends.<br />

http://healthprivacy.answers.hhs.gov/cgi-bin/hipaa.cfg/php/enduser/<br />

std_alp.php?p_cv=1.7%3B2.u0&%20p_cats=7%2C0&%20cat_lvl1=7&cat_lvl2=0<br />

United States Department <strong>of</strong> Health and Human Services Office for Civil Rights--HIPAA. Standards for Privacy<br />

<strong>of</strong> Individually Identifiable Health Information [45 CFR Parts 160 and 164].<br />

http://www.hhs.gov/ocr/hipaa/guidelines/guidanceallsections.pdf.<br />

Neuroimaging<br />

Question 51: Neuroimaging - Stroke<br />

Discussion:<br />

Parenchymal hemorrhages can be detected on MRI imaging documented within 2 hours <strong>of</strong> onset with gradient<br />

T2 (also called T2*) and EPI sequences due to magnetic susceptibility effects <strong>of</strong> deoxyhemoglobin, which is the<br />

earliest step in the breakdown <strong>of</strong> oxyhemoglobin.<br />

References:<br />

Patel M, Edelman R, Warach S. Detection <strong>of</strong> Hyperacute Primary Intraparenchymal Hemorrhage by Magnetic<br />

Resonance Imaging. Stroke 1996;27:2321-2324.<br />

Question 230: Neuroimaging - CSF Circulation Disorders<br />

Discussion:<br />

The CT shown is consistent with normal pressure hydrocephalus. The clinical presentation associated with<br />

normal pressure hydrocephalus consists <strong>of</strong> gait apraxia, executive function impairment, and urinary<br />

incontinence. There is no evidence <strong>of</strong> any lesions to suggest the clinical presentations <strong>of</strong> aphasia, hemisensory<br />

neglect, Parkinson's disease, or seizures.<br />

References:<br />

Osborne AG. .. Diagnostic Neuroradiology. St Louis: Mosby, 1994. pp. 154-68. neuroradiology. Am]<br />

Roentgenol 1991; 159:137-47.<br />

Question 231: Neuroimaging - Critical Care/Trauma<br />

<strong>2007</strong> <strong>RITE</strong> Discussion & Reference Manual<br />

49


Discussion:<br />

The scans show multiple small ring-enhancing lesions consistent with cerebral abscesses located at the<br />

supratentorial gray-white junction. In addition, a posterior occipital enhancing lesion is noted. The appearance is<br />

most consistent with septic emboli. By the imaging appearance alone, cerebral metastases are not ruled out. In<br />

the current case, the patient had endocarditis secondary to intravenous drug abuse. Uncomplicated pyogenic<br />

meningitis typically shows intense meningeal enhancement <strong>of</strong> the cerebral convexity, tentorium, and falx.<br />

Multiple sclerosis may show lesions with a similar appearence, but they would not involve the cortex, as seen on<br />

the sagittal image, and the enhancement ring would tend to open to the side <strong>of</strong> the gray matter, not the white<br />

matter as seen in the axial image <strong>of</strong> this case.<br />

References:<br />

Greenberg JO, editor. Neuroimaging: a companion to Adams and Victor's principles <strong>of</strong> neurology. 2nd ed. New<br />

York: McGraw-Hill, 1999.<br />

Question 232: Neuroimaging - Infection<br />

Discussion:<br />

CT shows multiple parenchymal calcifications with mild atrophy; this is characteristic <strong>of</strong> a host <strong>of</strong> infections,<br />

including toxoplasmosis, rubella, Cytomegalovirus, and herpes. Calcification seen in Sturge-Weber syndrome is<br />

typically cortical and gyriform. Although lipomas may calcify, they are extraaxial, usually single, and are<br />

hypodense on CT. The scan does not show malformations <strong>of</strong> the sulci or ventricles to suggest schizencephaly or<br />

colpocephaly.<br />

References:<br />

Greenberg JO. Neuroimaging: a companion to Adams and Victor's principles <strong>of</strong> neurology. New York:<br />

McGraw-Hill, 1999.<br />

Question 235: Neuroimaging - Spine<br />

Discussion:<br />

MR is the imaging modality <strong>of</strong> choice for evaluation <strong>of</strong> spinal infections. The typical MRI appearance is loss <strong>of</strong><br />

distinction between the endplates, disks, and adjacent vertebral bodies on T1-weighted views and increased<br />

T2-weighted signal. These areas show enhancement. The findings with high sensitivity for spinal infections<br />

include evidence <strong>of</strong> paraspinal, or epidural, inflammatory tissues, contrast enhancement <strong>of</strong> the disc, and<br />

hyperintensity on fluid-gradient T2-weighted views. Spinal infections rarely involve only one vertebral body<br />

and adjacent disc or only the epidural space.<br />

References:<br />

Gutmann DH, Rasmussen SA, Wolkenstein P, et al. MR Imaging gs in Spinal Infections: Rules or Myths?<br />

Radiology 2003;228:506-514.<br />

Question 237: Neuroimaging - Spine<br />

Discussion:<br />

A hyperintense signal is seen on both T1-weighted and T2-weighted images, suggesting that the lesion is a<br />

lipoma. Neurosurgical and radiation therapy consultations are not necessary for the management <strong>of</strong> this lesion.<br />

A lumbar puncture and blood cultures are not recommended. Lipomas are benign.<br />

References:<br />

Edelman R, Hesselink J, editors. Clinical Magnetic Resonance Imaging. Philadelphia: WB Saunders, 1990.<br />

Question 238: Neuroimaging - Brain Tumors<br />

Discussion:<br />

<strong>2007</strong> <strong>RITE</strong> Discussion & Reference Manual<br />

50


MR is effective in demonstrating a displaced optic chiasm because the optic chiasm is usually slightly more<br />

intense than pituitary tumors.<br />

References:<br />

Daniels DL, Haughton VM, Czervionke LF. MR <strong>of</strong> the skull base. In: Bradley WG, Stark D. Magnetic<br />

resonance imaging. St. Louis: Mosby, 1988.<br />

Greenberg JO. Neuroimaging: a companion to Adams and Victor's principles <strong>of</strong> neurology. New York:<br />

McGraw-Hill, 1999.<br />

Question 241: Neuroimaging - Autoimmune Disorders (Non-MS)<br />

Discussion:<br />

Leptomeningeal enhancement, parenchymal enhancement, and, especially, pituitary stalk/hypothalamic<br />

involvement in a patient with cranial nerve symptoms is characteristic <strong>of</strong> neurosarcoidosis. About 5% to 10% <strong>of</strong><br />

sarcoidosis patients develop CNS involvement. Lack <strong>of</strong> hemiatrophy and prominent (asymmetrical) choroid<br />

plexus enhancement make the diagnosis <strong>of</strong> Sturge-Weber syndrome less likely. No gyral enhancement is<br />

present. Low CSF pressure would not be limited to one hemisphere and would not have masslike and<br />

leptomeningeal enhancement. The enhancing lesion is intraaxial and therefore is not a meningioma.<br />

References:<br />

Osborn AG. Diagnostic neuroradiology. St. Louis: CV Mosby, 1994.<br />

Question 242: Neuroimaging - Dementia<br />

Discussion:<br />

The most striking change between the two studies is the pronounced decrease in glucose metabolism in the<br />

frontal and anterior temporal regions. This pattern is characteristic <strong>of</strong> frontotemporal dementia. Alzheimer’s<br />

disease tends to decrease metabolism in the parietotemporal association cortex, relatively spared in the scan<br />

shown. Parkinson’s disease does not cause a decrease in metabolism. As in this case, in Huntington’s chorea<br />

there is decreased metabolism in the head <strong>of</strong> the caudate nuclei, but the frontal and temporal lobes are not as<br />

affected. Finally, diffuse Lewy body disease causes decreased metabolism in the occipital regions, relatively<br />

spared in this scan.<br />

References:<br />

Masdeu JC, Zubieta JL, Arbizu J. Neuroimaging as a marker <strong>of</strong> the onset and progression <strong>of</strong> Alzheimer's<br />

disease. J Neurol Sci 2005;236:55-64.<br />

Kantarci K, Jack Jr C. Neuroimaging in Alzheimer disease: an evidence-based review. Neuroimaging Clin N<br />

Am 2003;13:197-209.<br />

Coleman RE. Positron emission tomography diagnosis <strong>of</strong> Alzheimer's disease. Neuroimaging Clin N Am<br />

2005;15:837-846.<br />

Question 243: Neuroimaging - Brain Tumors<br />

Discussion:<br />

The most likely diagnosis is glioblastoma multiforme. On the FLAIR image, the lesion presents with mass<br />

effect, too pronounced for an arteriovenous malformation, choroid plexus calcification, or an infarction. The<br />

abnormal high-intensity region extends across the corpus callosum to the right hemisphere, a feature common in<br />

malignant gliomas. The abnormal contrast enhancement suggests that the tumor has abnormal vessels with<br />

marked permeability, another finding in glioblastoma. However, macroscopically dilated vessels, characteristic<br />

<strong>of</strong> an arteriovenous malformation are not present. A choroid plexus papilloma is very unusual in this age group.<br />

A toxoplasma abscess tends to show enhancement at the periphery and with a more regular appearance than in<br />

the present case.<br />

References:<br />

<strong>2007</strong> <strong>RITE</strong> Discussion & Reference Manual<br />

51


Greenberg JO, editor. Neuroimaging: A Companion to Adams and Victor's Principles <strong>of</strong> Neurology. 2nd ed.<br />

New York: McGraw-Hill, 1999.<br />

Question 244: Neuroimaging - Infection<br />

Discussion:<br />

Multiple basal ganglia high-signal foci are seen bilaterally. They are more numerous than normal<br />

Virchow-Robin spaces. Toxoplasmosis resides also in basal ganglia, but then there is usually also mass effect.<br />

Cryptococcus enters the intracranial space through the perivascular spaces, and the organisms are surrounded by<br />

gelatinous material that is seen as high signal foci. This correlates with "soap-bubble" abscesses seen grossly.<br />

The correct answer is cryptococcus infection.<br />

References:<br />

Osborn AG. Diagnostic neuroradiology. St. Louis: CV Mosby, 1994.<br />

Question 245: Neuroimaging - Developmental/Neurogenetic Disorders<br />

Discussion:<br />

Sagittal T1-weighted image shows a Chiari II malformation with deformity <strong>of</strong> the tectum <strong>of</strong> the mesencephalon,<br />

caudalization <strong>of</strong> the cerebellar vermis into the cervical spinal canal, and a deformity <strong>of</strong> the medial aspect <strong>of</strong> the<br />

cerebral hemisphere, with an absent posterior corpus callosum. The cerebral aqueduct is not visualized, and the<br />

ventricles do not appear enlarged, suggesting there is aqueductal stenosis and that a functioning shunt should be<br />

present as was the case. There is a high degree <strong>of</strong> association between the cerebral changes <strong>of</strong> Chiari II<br />

malformation and the presence <strong>of</strong> a lumbar myelomeningocele, thus the lumbar spine should be and was<br />

dysraphic. Patients with Chiari II malformations do not have to be and are frequently not mentally retarded.<br />

There is no evidence <strong>of</strong> contusion, subdural hematoma, or other forms <strong>of</strong> trauma on the section shown.<br />

References:<br />

McLone DG, Naidich TP. Developmental morphology <strong>of</strong> the subarachnoid space, brain vasculature, and<br />

contiguous structures, and the cause <strong>of</strong> the Chiari II malformation. AJNR Am J Neuroradiol 1992;13(2):463-482.<br />

Question 246: Neuroimaging - Epilepsy<br />

Discussion:<br />

The image shows heterotopic gray matter in the left frontal region. Deformity <strong>of</strong> the left frontal horn is seen with<br />

ex-vacuo hydrocephalus. No mass effect is seen to suggest tumor. Rather, the lesion is destructive. Heterotopic<br />

gray matter refers to cortical neurons in abnormal location. Heterotopia is caused by migrational arrest <strong>of</strong> the<br />

affected neuroblasts. The cause <strong>of</strong> the incomplete migration is unknown. Three kinds <strong>of</strong> heterotopic gray matter<br />

have been described: (1) nodular (periventricular), (2) laminar (in deep white matter), like in this patient, and (3)<br />

band heterotopia (double cortex).<br />

References:<br />

Guerrini R, Filippi T. Neuronal migration disorders, genetics, and epileptogenesis. J Child Neurol<br />

2005;20:287-299.<br />

Question 247: Neuroimaging - Stroke<br />

Discussion:<br />

<strong>2007</strong> <strong>RITE</strong> Discussion & Reference Manual<br />

The large vascular malformation is an aneurysm <strong>of</strong> the vein <strong>of</strong> Galen. The sagittal time-<strong>of</strong>-flight image shows<br />

particularly well the blood backing up into the posterior portion <strong>of</strong> the superior sagittal sinus, which is markedly<br />

dilated. The fast flow in the malformation creates a flow void, visible as a dark structure in both the T2- and<br />

T1-weighted images. The morphology <strong>of</strong> this malformation is characteristic and differentiates it from the other<br />

answer options. Cavernous and venous angiomas are much smaller. A dural fistula has an important arterial<br />

component, absent in this case. A hemangioma has the appearance <strong>of</strong> a tumor, without the large vessels visible<br />

in this patient.<br />

52


References:<br />

Greenberg JO, editor. Neuroimaging: A Companion to Adams and Victor's Principles <strong>of</strong> Neurology. 2nd ed.<br />

New York: McGraw-Hill, 1999.<br />

Question 250: Neuroimaging - Spine<br />

Discussion:<br />

Diastematomyelia is a rare form <strong>of</strong> spinal dysraphism. On MRI the cord appears split with a sagittal cleft <strong>of</strong><br />

varying extent. Fiber, cartilaginous, or osseous septums produce complete or incomplete divisions <strong>of</strong> the spinal<br />

cord into hemicords. Patients may have neurological defects in the lower limbs such as gait disorders, sphincter<br />

disturbance, or muscle atrophy.<br />

References:<br />

Sami H, Ross E, Walter M, Goli S. Split Spinal Cord (Diastematomyelia). Neurology 2003; 60:491.<br />

Question 253: Neuroimaging - Stroke<br />

Discussion:<br />

The distribution <strong>of</strong> the changes on postendarterectomy MRI are consistent with an acute infarct in the<br />

distribution <strong>of</strong> the anterior cerebral artery.<br />

References:<br />

Greenberg JO, editor. Neuroimaging: a companion to Adams and Victor's principles <strong>of</strong> neurology. 2nd ed. New<br />

York: McGraw-Hill, 1999.<br />

Question 254: Neuroimaging - Stroke<br />

Discussion:<br />

The MRI shows a subacute left middle cereberal infarct with a subacute hematoma. The hematoma has increased<br />

T1 signal and decreased T2 signal. There is mild ipsilateral dilation <strong>of</strong> the left ventricle indicating an anteceding<br />

chronic infarct. There is no mass effect or midline shift to indicate an acute left infarct. Chronic hemorrhage<br />

would cause hypointense signal on both pulse sequences<br />

References:<br />

Barnett HJM. Stroke: pathophysiology, diagnosis, and management. New York: Churchill Livingstone, 1992.<br />

Question 255: Neuroimaging - Epilepsy<br />

Discussion:<br />

The axial T2-weighted image shows a symmetrical brain with normal size ventricles. The frontal lobe cortex is<br />

flat, and the white matter, which is hypointense, does not show the normal interdigitation into the gray matter in<br />

this region. Thus, the cortex is thick, and the patient has pachygyria. Such an abnormality develops during the<br />

period <strong>of</strong> neuronal migration, occurring during the second trimester.<br />

References:<br />

Zimmerman RA, Bilaniuk LT. Pediatric central nervous system. In: Stark DD, Bradley WG, editors. Magnetic<br />

resonance imaging. St. Louis: CV Mosby, 1988.<br />

Question 256: Neuroimaging - Developmental/Neurogenetic Disorders<br />

Discussion:<br />

<strong>2007</strong> <strong>RITE</strong> Discussion & Reference Manual<br />

MR images demonstrate dilatation <strong>of</strong> the cord around a central cystic cavity, consistent with<br />

syringohydromyelia. Note also on the sagittal images that the inferior portion <strong>of</strong> the cerebellum has herniated<br />

53


down into the upper cervical canal, diagnostic <strong>of</strong> a Chiari malformation. Hydromyelia is commonly seen in<br />

association with both Chiari type I and Chiari type II malformation.<br />

References:<br />

Greenberg JO. Neuroimaging: a companion to Adams and Victor's principles <strong>of</strong> neurology. New York:<br />

McGraw-Hill, 1999.<br />

Question 258: Neuroimaging - Multiple Sclerosis<br />

Discussion:<br />

The MRI reveals far too much white matter disease for Alzheimer's disease. Metachromatic leukodystrophy<br />

typically produces by this age more diffuse and confluent areas <strong>of</strong> demyelination than those shown. Progressive<br />

multifocal leukoencephalopathy is less common, and large regions (rather than small plaques) <strong>of</strong> white matter<br />

signal change are expected. Given the diffuse white matter lesions that are characteristic <strong>of</strong> multiple sclerosis<br />

plaques, her age, and gender, multiple sclerosis is the most likely diagnosis. Vasculitis typically involves the<br />

gray/white matter junction.<br />

References:<br />

Benedict RH, Weinstock-Guttman B, Fishman I, et al. Prediction <strong>of</strong> neuropsychological impairment in multiple<br />

sclerosis: comparison <strong>of</strong> conventional magnetic resonance imaging measures <strong>of</strong> atrophy and lesion burden. Arch<br />

Neurol 2004;61:226-230.<br />

Question 259: Neuroimaging - Movement Disorders<br />

Discussion:<br />

The patient has cerebellar and pontine atrophy. This finding and the abnormal signal in the pons, in the form <strong>of</strong> a<br />

cross, are found in olivopontocerebellar atrophy but not the other diagnostic possibilities. In the Chiari<br />

malformation the posterior fossa is crowded, the opposite <strong>of</strong> what happens here. With an arachnoid cyst, the<br />

subarachnoid space could be dilated but not the fourth ventricle. Finally, the inferior vermis is not atretic as it is<br />

in the Dandy-Walker syndrome.<br />

References:<br />

Srivastava T, Singh S, Goyal V, et al. "Hot cross bun" sign in two patients with multiple system<br />

atrophy-cerebellar. Neurology 2005;64:128.<br />

Question 260: Neuroimaging - Critical Care/Trauma<br />

Discussion:<br />

Anoxia causes increased T2 signal in the globus pallidus, deep portions <strong>of</strong> the sulci, and superior cerebellum,<br />

which are all anatomical regions that are susceptible to hypoxia. The necrotic globus pallidus enhances after<br />

gadolinium administration.<br />

References:<br />

Greenberg JO. Neuroimaging: a companion to Adams and Victor's principles <strong>of</strong> neurology. New York:<br />

McGraw-Hill, 1999.<br />

Question 261: Neuroimaging - Spine<br />

Discussion:<br />

<strong>2007</strong> <strong>RITE</strong> Discussion & Reference Manual<br />

Although the MRI tends to rule out an astrocytoma and ependymoma because there is only mild mass effect, it<br />

could still be an inflammatory lesion, giving rise to the increased signal on the T2 images. A heavily<br />

T2-weighted sequence (T2-CISS) shows a cluster <strong>of</strong> vessels on the dorsal aspect <strong>of</strong> the cord compatible with<br />

venous dilation due to arteriovenous shunting by a dural fistula. The selective angiogram confirms this<br />

impression because the venous system is filled so rapidly by a brief arterial injection that both arterial and<br />

venous vessels are seen on the same angio frame. This disorder corresponds to the myelopathy <strong>of</strong><br />

54


Foix-Alajouanine.<br />

References:<br />

Greenberg JO, editor. Neuroimaging: A Companion to Adams and Victor's Principles <strong>of</strong> Neurology. 2nd ed.<br />

New York: McGraw-Hill, 1999.<br />

Koch C. Spinal dural arteriovenous fistula. Curr Opin Neurol 2006;19:69-75.<br />

Question 262: Neuroimaging - Critical Care/Trauma<br />

Discussion:<br />

The patient was a diabetic with severe vascular disease. The morphology <strong>of</strong> the hemispheric lesions resembles<br />

vascular disease more than either multiple sclerosis or a lymphoma. Of the three vessels mentioned in the<br />

options, the right carotid does not show the flow void characteristic <strong>of</strong> circulating blood.<br />

References:<br />

Greenberg JO, editor. Neuroimaging: A Companion to Adams and Victor's Principles <strong>of</strong> Neurology. 2nd ed.<br />

New York: McGraw-Hill, 1999.<br />

Question 265: Neuroimaging - Stroke<br />

Discussion:<br />

The FLAIR MRI image shows an area <strong>of</strong> high signal intensity in the distribution <strong>of</strong> the recurrent artery <strong>of</strong><br />

Heubner. It is consistent with an infarct in this tissue. The recurrent artery <strong>of</strong> Heubner arises from the A1<br />

segment <strong>of</strong> the anterior cerebral artery and supplies the anteroinferior portion <strong>of</strong> the caudate nucleus, the<br />

putamen, and the anterior limb <strong>of</strong> the internal capsule.<br />

References:<br />

Netter FH. Nervous system anatomy and physiology. New Jersey: CIBA Medical Education Division, 1986.<br />

Question 268: Neuroimaging - Dementia<br />

Discussion:<br />

The patient had a subdural hematoma with mixed densities, some corresponding to a more recent bleeding. The<br />

shape <strong>of</strong> the lesion and the low density <strong>of</strong> the deeper portion are uncharacteristic <strong>of</strong> an epidural hematoma, with<br />

a lens shape and the high density <strong>of</strong> fresh blood. The lesion is outside the brain, compressing the cortex <strong>of</strong> the<br />

right hemisphere. For this reason, infarction is not an option. A meningioma "en plaque" would be more<br />

homogeneous and have higher density.<br />

References:<br />

Greenberg JO, editor. Neuroimaging: A Companion to Adams and Victor's Principles <strong>of</strong> Neurology. 2nd ed.<br />

New York: McGraw-Hill, 1999.<br />

Question 270: Neuroimaging - Brain Tumors<br />

Discussion:<br />

This is a typical location for a colloid cyst. This usually causes low signal in the T2-weighted image; however,<br />

the MRI signal can be variable.<br />

References:<br />

Dalessio DJ. Diagnosing the severe headache. Neurology 1994;44(Suppl 3):S6-S12.<br />

Question 272: Neuroimaging - Brain Tumors<br />

Discussion:<br />

<strong>2007</strong> <strong>RITE</strong> Discussion & Reference Manual<br />

55


Pleomorphic xanthoastrocytoma typically produce a large cyst with superficially positioned mural nodule. Most<br />

lesions arise within the temporal or parietal lobes. This lesion is surrounded with minimal hemosiderin rim but<br />

no edema, and this excludes abscess and hematomas. The signal characteristics <strong>of</strong> cavernous hemangioma are<br />

heterogeneous (popcorn). The mass effect is too little for a hematoma <strong>of</strong> this size. Porencephalic cysts do not<br />

exert mass effect.<br />

References:<br />

Osborne A. Diagnostic Imaging: Brain. Philadelphia: Elsevier, 2004.<br />

Question 274: Neuroimaging - Spine<br />

Discussion:<br />

Transverse myelitis affects predominantly the thoracic spinal cord and on imaging reflects as an increased signal<br />

in the spinal cord on T2 images extending several segments. The spinal tap would show lympocytic<br />

predominance and increased protien. There is no evidence <strong>of</strong> disc herniation, tumor, or spinal artery stroke on<br />

the image. Gullain-Barre syndrome would not cause any increased signal in the spinal cord.<br />

References:<br />

Edelman R, Hesselink J, editors. Clinical Magnetic Resonance Imaging. Philadelphia: WB Saunders, 1990.<br />

Question 276: Neuroimaging - Spine<br />

Discussion:<br />

Axial image demonstrates a small, constricted spinal canal. This is mainly due to degenerative facet joint<br />

disease. No disc herniation or synovial cyst is present. The nerve roots are coiled above the narrow area since<br />

they cannot slide freely though the narrow part. Distal nerve roots are stretched. The correct answer is spinal<br />

stenosis.<br />

References:<br />

Ross J, Zawadzki MB, Chen M, et al. Diagnostic Imaging: Spine. Salt Lake City: Saunders-Elsevier, 2004.<br />

Question 277: Neuroimaging - Dementia<br />

Discussion:<br />

The MRI study shows a recent subcortical hemorrhage in the subcortical white matter <strong>of</strong> the right paracentral<br />

region. This hemorrhage is responsible for the recent hemiparesis. The subcortical location <strong>of</strong> the hemorrhage is<br />

typical <strong>of</strong> amyloid (or congophilic) angiopathy and very atypical for a hypertensive hemorrhage. Hemorrhages<br />

do not tend to happen with either CADASIL or subcortical arteriosclerotic encephalopathy. Finally, the pattern<br />

<strong>of</strong> multiple lesions in the white matter is not present in frontotemporal dementia.<br />

References:<br />

Chao CP, Kotsenas AL, Broderick DF. Cerebral amyloid angiopathy: CT and MR imaging findings.<br />

Radiographics 2006;26:1517-1531.<br />

Greenberg JO, editor. Neuroimaging: A Companion to Adams and Victor's Principles <strong>of</strong> Neurology. 2nd ed.<br />

New York: McGraw-Hill, 1999.<br />

Question 279: Neuroimaging - Spine<br />

Discussion:<br />

An intradural, extramedullary mass in the thoracic spine is causing significant cord compression. This<br />

represents either neur<strong>of</strong>ibroma or meningioma and requires surgical removal. Neither tumor is radiosensitive.<br />

Imaging with CT or myelography is not likely to add any further useful information.<br />

References:<br />

<strong>2007</strong> <strong>RITE</strong> Discussion & Reference Manual<br />

56


Osborn A. Diagnostic Neuroradiology. St. Louis: Mosby, 1994.<br />

Question 280: Neuroimaging - Stroke<br />

Discussion:<br />

The internal carotid artery segment shown is immediately proximal to the take<strong>of</strong>f <strong>of</strong> the middle cerebral artery<br />

(which goes to the right) and the anterior cerebral artery (which goes to the left).<br />

References:<br />

Osborn A. Introduction to cerebral angiography. Hagerstown, MD: Harper & Row, 1980.<br />

Question 281: Neuroimaging - Critical Care/Trauma<br />

Discussion:<br />

The scan demonstrates findings highly suggestive <strong>of</strong> acute or subacute herpes simplex virus (HSV) encephalitis<br />

involving the temporal lobe. The lesion crosses the vascular boundaries <strong>of</strong> the middle and posterior cerebral<br />

arteries, making an ischemic insult unlikely. The contrast-enhancing pattern would be unusual for glioblastoma<br />

or lymphoma. A contusion could occur in this location but would contain evidence <strong>of</strong> hemorrhage and MR<br />

evidence <strong>of</strong> parenchymal blood degradation products, which is not seen in this case. HSV encephalitis primarily<br />

involves limbic structures. MR typically shows temporal and inferior frontal lobe swelling, with low signal on<br />

T1-weighted images and high signal on T2-weighted images. Postcontrast studies in the acute and subacute<br />

setting <strong>of</strong>ten show cortical and leptomeningeal enhancement. The lesion may not be hemorrhagic in the early<br />

stages.<br />

References:<br />

de Almeida Magalhaes AC, Bacheschi LA. Viral and nonviral infections <strong>of</strong> the nervous system. In: Greenberg<br />

JO, editor. Neuroimaging: a companion to Adams and Victor's principles <strong>of</strong> neurology. 2nd ed. New York:<br />

McGraw Hill, 1999.<br />

Question 283: Neuroimaging - Critical Care/Trauma<br />

Discussion:<br />

The CT scan shows a collection <strong>of</strong> air in the right frontal lobe, with the bottom portion having an air fluid level.<br />

The finding is not consistent with an artifact nor with a vascular event. Infection could be present; however, the<br />

way in which the air would gain access to the intracranial compartment would have to involve a break in the<br />

skull, most <strong>of</strong>ten in the air containing sinuses <strong>of</strong> the cranial base. In this case, the air collection entered through<br />

a fracture <strong>of</strong> the frontal sinus. In order for the air to occupy as much space within the brain parenchyma as it<br />

does, there would have to be a loss <strong>of</strong> substance <strong>of</strong> the brain focally at the site. This is consistent with the patient<br />

having had a loss <strong>of</strong> brain substance (volume) due to a prior focal hemorrhagic contusion.<br />

References:<br />

Woodruff WW. Fundamentals <strong>of</strong> neuroimaging. Philadelphia: WB Saunders, 1993.<br />

Question 284: Neuroimaging - CSF Circulation Disorders<br />

Discussion:<br />

The correct diagnosis is communicating hydrocephalus. Dilation <strong>of</strong> the Sylvian fissure and other sulci may<br />

suggest atrophy. However, this feature is not uncommon in shunt-responsive hydrocephalus, as documented in<br />

the references. On the coronal images note that the sulci at the high parietal convexity are compressed, as<br />

compared to the markedly dilated posterior extent <strong>of</strong> the Sylvian fissure.<br />

References:<br />

<strong>2007</strong> <strong>RITE</strong> Discussion & Reference Manual<br />

Kitagaki H, Mori E, Ishii K, et al. CSF spaces in idiopathic normal pressure hydrocephalus: morphology and<br />

volumetry. AJNR Am J Neuroradiol 1998;19:1277-1284.<br />

57


Question 285: Neuroimaging - Brain Tumors<br />

Discussion:<br />

The postinjection T1-weighted MR image shows enhancement <strong>of</strong> an extra-axial small mass that displaces the<br />

brain parenchyma away from the inner table <strong>of</strong> the skull. The inner table <strong>of</strong> the skull is focally expanded at the<br />

center <strong>of</strong> the mass. The findings are characteristic <strong>of</strong> a meningioma. A "dural tail sign" is present on the<br />

postcontrast image. An epidural hematoma would have been <strong>of</strong> high signal intensity on the preinjection study.<br />

Without expansion <strong>of</strong> the bone, a depressed fracture would have a plane <strong>of</strong> cleavage within the calvarium and<br />

may also have had an underlying cortical contusion.<br />

References:<br />

Berry I, Brant-Zawadski M, Osaki L, et al. Gd-DTPA in clinical MR <strong>of</strong> the brain: extra-axial lesions and normal<br />

structures. AJNR Am J Neuroradiol 1986;7:789.<br />

Question 286: Neuroimaging - Multiple Sclerosis<br />

Discussion:<br />

Enhancement is circumscribed to the margins <strong>of</strong> the lesion and affects only the white matter. Several patches <strong>of</strong><br />

enhancement are in subcortical white matter. The overall appearance <strong>of</strong> the enhancement resembles an open<br />

ring, a finding characteristic <strong>of</strong> the large lesion (Schilder's) variety <strong>of</strong> multiple sclerosis. An infarction would<br />

give rise to cortical enhancement. The other lesions tend to cause closed rings, and characteristically the gray<br />

matter tends to enhance more than the white matter if the lesion is as close to the cortex as this one is. In this<br />

patient the lesion shown improved on steriods. Six months later he developed a similar lesion in the frontal lobe.<br />

References:<br />

Masdeu JC, Quinto C, Olivera C, et al. Open-ring imaging sign: highly specific for atypical brain demyelination.<br />

Neurology 2000;54:1427-1433.<br />

Question 287: Neuroimaging - Stroke<br />

Discussion:<br />

A developmental venous anomaly (DVA) or venous angioma is occasionally identified in the work-up <strong>of</strong><br />

patients with neurological symptoms. Headache is the most common symptom. Seizures are also commonly<br />

associated. There is no real evidence, however, that either headaches or seizures are caused by the DVA, and in<br />

most instances the DVA is incidental. It is very important to understand that the DVA represents the venous<br />

drainage <strong>of</strong> the brain tissue in which it is situated. Removal <strong>of</strong> the DVA may cause a venous infarction.<br />

References:<br />

Rigamonti D, Hsu FPK, Huhn S. Angiographically occult vascular malformations in neurovascular surgery. In:<br />

Carter PL, Spetzler RF, editors. New York: McGraw-Hill, 1994.<br />

Question 288: Neuroimaging - CSF Circulation Disorders<br />

Discussion:<br />

There is marked enlargement <strong>of</strong> the third and lateral ventricles, with a normal-sized fourth ventricle. Note also<br />

the peculiar configuration <strong>of</strong> the quadrigeminal plate, which appears displaced and compressed superiorly, but<br />

normally inferiorly, reflecting dilation <strong>of</strong> the proximal portion <strong>of</strong> the aqueduct only. The axial T2 image<br />

through the midbrain also fails to demonstrate the normal flow void seen at the level <strong>of</strong> the aqueduct that<br />

originated from high velocity <strong>of</strong> CSF traveling through the patent aqueduct. These findings are consistent with a<br />

diagnosis <strong>of</strong> obstructive hydrocephalus, with obstruction at the level <strong>of</strong> the aqueduct (excluding the choices <strong>of</strong><br />

atrophy or communicating hydrocephalus). In this case, there is no evidence <strong>of</strong> a mass in the quadrigeminal<br />

plate, and obstruction is due to stenosis.<br />

References:<br />

<strong>2007</strong> <strong>RITE</strong> Discussion & Reference Manual<br />

58


Bradley WB, Quencer RM. Hydrocephalus, atrophy, and intracranial CSF flow. In: Stark DD, Bradley WG.<br />

Magnetic resonance imaging, Vol. 1. 2nd ed. St Louis: CV Mosby, 1992.<br />

Question 289: Neuroimaging - Critical Care/Trauma<br />

Discussion:<br />

The sagittal two-dimensional phase contrast venogram with a velocity encoding <strong>of</strong> 15 cm/s is tailored to<br />

demonstrate slow flow such as that in the dural venous sinuses. The scan demonstrates practically no flow in the<br />

superior sagittal sinus (SSS). One prominent cortical vein shows flow. Just adjacent to this cortical vein, minor<br />

flow is seen in the SSS. This finding is consistent with near occlusion <strong>of</strong> the SSS. Higher velocity encodings are<br />

needed to demonstrate arterial flow.<br />

References:<br />

Medlock MD, Olivero WC, Hanigan WC, et al. Children with cerebral venous thrombosis diagnosed with<br />

magnetic resonance imaging and magnetic resonance angiography. Neurosurgery 1992;31:870-876.<br />

Question 290: Neuroimaging - Spine<br />

Discussion:<br />

In the images, the intervertebral disc near a postsurgical foreign body is obliterated, and there is enhancement <strong>of</strong><br />

the sheath <strong>of</strong> the psoas muscle. These findings are most suggestive <strong>of</strong> infection. Several MRI patterns have been<br />

described as indicating spinal infections, including decreased disk height, disk hypointensity on T1-weighted<br />

images, and hyperintensity on T2-weighted images, disk enhancement, erosion <strong>of</strong> vertebral endplates, the loss<br />

<strong>of</strong> distinction between endplate and disk, along with the adjacent vertebral bodies on T1-weighted signal is most<br />

frequently seen. These areas show enhancement. Spinal infections are most frequently associated with<br />

paraspinal or epidural inflammatory tissues.<br />

References:<br />

Ledermann HP, Schweitzer ME, Morrison WB, Carrino JA. MR Imaging Findings in Spinal Infections: Rules or<br />

Myths? Radiology 2003;228:506-514.<br />

Question 292: Neuroimaging - Infection<br />

Discussion:<br />

The ring-enhancing lesion in the lower portion <strong>of</strong> the precentral region has the characteristics <strong>of</strong> a brain abscess.<br />

Corresponding to the ring <strong>of</strong> enhancement in the gadolinium study is a dark rim in the FLAIR image. This rim<br />

has a rich vascular network and abundant macrophages. The core <strong>of</strong> the lesion, containing an excess <strong>of</strong> water as<br />

compared to brain, is hyperintense on T2 (FLAIR) and hypointense on T1. Around the ring there is brain<br />

edema, also hyperintense on T2 and hypointense on T1. None <strong>of</strong> the other options have similar characteristics.<br />

Astrocytomas <strong>of</strong> the types indicated seldom enhance. Areas <strong>of</strong> enhancement in an infarct tend to involve the<br />

cortical ribbon whereas in the lesion shown the ring is at the corticomedullary junction, mostly in the white<br />

matter. The lesion shown is in the brain parenchyma, not outside, as would happen with a meningioma.<br />

References:<br />

Kastrup O, Wanke I, Maschke M. Neuroimaging <strong>of</strong> infections. NeuroRx 2005;2:324-332.<br />

Greenberg JO, editor. Neuroimaging: A Companion to Adams and Victor's Principles <strong>of</strong> Neurology. 2nd ed.<br />

New York: McGraw-Hill, 1999.<br />

Question 293: Neuroimaging - Dementia<br />

Discussion:<br />

<strong>2007</strong> <strong>RITE</strong> Discussion & Reference Manual<br />

The enhanced CT image demonstrates multiple irregular ring-enhancing lesions adjacent to and compressing the<br />

lateral ventricles. There is extensive white matter vasogenic edema in both hemispheres. This appearance is most<br />

consistent with a glioblastoma that has spread through the corpus callosum to involve both hemispheres. In<br />

immunocompromised patients, lymphoma can have a similar appearance, but in immunocompetent patients,<br />

59


immunocompromised patients, lymphoma can have a similar appearance, but in immunocompetent patients,<br />

lymphoma is most commonly seen as a homogenously enhancing mass. Herpes encephalitis in adults nearly<br />

always affects the temporal lobes. Neurocysticercosis can occur in the brain parenchyma or as cysts within the<br />

ventricles. First seen as cysts in the brain, cysticercosis can incite an inflammatory reaction when the larva dies,<br />

but the lesions are usually small, well circumscribed, and located at the corticomedullary junction.<br />

References:<br />

Osborn AG. Diagnostic neuroradiology. St. Louis: CV Mosby, 1994.<br />

Question 294: Neuroimaging - Developmental/Neurogenetic Disorders<br />

Discussion:<br />

The lesion at the tip <strong>of</strong> the left temporal lobe spares the cortical ribbon and is clearly extraaxial, ruling out old<br />

head trauma or herpes simplex encephalitis. The signal characteristics are typical <strong>of</strong> CSF, ruling out a<br />

metastasis. The lesion on the right perimesencephalic cistern cannot be incisural sclerosis, characterized by<br />

gliosis not by a CSF-like lesion like the one shown. Both lesions are arachnoid cysts.<br />

References:<br />

Greenberg JO. Neuroimaging: a companion to Adams and Victor's principles <strong>of</strong> neurology. New York:<br />

McGraw-Hill, 1999.<br />

Question 296: Neuroimaging - Brain Tumors<br />

Discussion:<br />

Axial CTs show a large mass lesion filling the right orbit, compressing and deforming the posterior margin <strong>of</strong><br />

the globe. The inferior cut through the maxillary sinus shows that the lateral wall <strong>of</strong> the maxilla is destroyed and<br />

is associated with the s<strong>of</strong>t tissue mass. Metastatic disease commonly involves bone and adjacent s<strong>of</strong>t tissue, as in<br />

this case. Facial trauma does not produce a disappearance <strong>of</strong> bone but fractures it. Fibrous dysplasia expands<br />

bone but does not destroy it. Hemangioma may involve bone and s<strong>of</strong>t tissue, but is not a destructive process. In<br />

the orbit, capillary hemangiomas are commonly found in the cutaneous tissues and periorbita but do not extend<br />

retroglobar and do not destroy bone. Cavernous hemangiomas are discrete masses that may be found intraorbital<br />

or extraorbital but do not usually cross these margins.<br />

References:<br />

Zimmerman RA, Bilaniuk LT. Computed tomography <strong>of</strong> the orbit. In: Lee S, Rao K, editors. Cranial<br />

computerized tomography. New York: McGraw-Hill, 1983.<br />

Question 299: Neuroimaging - Spine<br />

Discussion:<br />

The scans show active spondylitis at L4-L5 and remote spondylitis at L2-L3. Spondylitis may include<br />

osteomyelitis, discitis, and epidural abscess formation, each <strong>of</strong> which are seen in the current case at L4-L5. The<br />

enhancement and T2 hyperintensity <strong>of</strong> the L4-L5 disc is consistent with active discitis. In contrast, metastatic<br />

disease usually spares and does not cross the disc space. Remote spondylitis <strong>of</strong>ten results in the changes seen in<br />

the current case at L2-L3; the disc space is markedly narrowed, and the adjacent vertebral bodies show<br />

degenerative changes. Incidental note is made <strong>of</strong> scoliosis, which is maximal at L2.<br />

References:<br />

Osborn AG. Diagnostic neuroradiology. St. Louis: CV Mosby, 1994.<br />

Question 300: Neuroimaging - Dementia<br />

Discussion:<br />

<strong>2007</strong> <strong>RITE</strong> Discussion & Reference Manual<br />

The lesion in the left temporal lobe is isodense on T1 and enhances with gadolinium. It is outside the temporal<br />

lobe. It causes compression <strong>of</strong> the tip and medial aspect <strong>of</strong> the temporal lobe, with partial herniation <strong>of</strong> the<br />

60


parahippocampal gyrus. These features are characteristic <strong>of</strong> a meningioma. Both Alzheimer’s disease and<br />

frontotemporal dementia would present with atrophy, not mass effect. Both an astrocytoma and a glioblastoma<br />

are intraparenchymal tumors.<br />

References:<br />

Greenberg JO. Neuroimaging: a companion to Adams and Victor's principles <strong>of</strong> neurology. New York:<br />

McGraw-Hill, 1999.<br />

Question 301: Neuroimaging - Critical Care/Trauma<br />

Discussion:<br />

Dissection and total occlusion <strong>of</strong> the left internal carotid are incorrect because there is no evidence for<br />

dissection. Emergency surgery is not required and it is clear that there is a large ulcerated plaque, not a<br />

dissection. The ulcerated plaque could have resulted from the accident, but it is far more likely that it happened<br />

spontaneously. If plaque material from within a crater this large had broken <strong>of</strong>f and embolized during the<br />

accident, the patient would have had serious neurological morbidity. Therefore, the correct answer is that the<br />

accident did not cause the findings.<br />

References:<br />

Watridge CB, Muhlbauer MS, Lowery RD. Traumatic carotid artery dissection: diagnosis and treatment. Journal<br />

<strong>of</strong> Neurosurgery 1989;71(6):854-857.<br />

Question 302: Neuroimaging - Critical Care/Trauma<br />

Discussion:<br />

The most likely diagnosis is subarachnoid hemorrhage. The high density <strong>of</strong> the recent bleeding outlines the<br />

subarachnoid space around the brain. Its shape rules out the other types <strong>of</strong> hemorrhage mentioned as options.<br />

There is hydrocephalus, but it is not idiopathic; rather, it is caused by the subarachnoid hemorrhage. The air with<br />

pneumocephalus would have a much lower density than any <strong>of</strong> the structures seen on the image.<br />

References:<br />

Greenberg JO, editor. Neuroimaging: A Companion to Adams and Victor's Principles <strong>of</strong> Neurology. 2nd ed.<br />

New York: McGraw-Hill, 1999.<br />

Question 303: Neuroimaging - Infection<br />

Discussion:<br />

Both the clinical picture and the imaging suggest a subacute process, with quick worsening. Of the given<br />

options, listeriosis and multiple sclerosis usually have a subacute course. Of the two, the imaging is more typical<br />

<strong>of</strong> listeriosis, an infection produced by Listeria monocytogenes that has a preference for the brainstem. In the<br />

second study, the larger abscess in the tegmentum <strong>of</strong> the pons, visible in the gadolinium-enhanced study, is<br />

accompanied by a subependymal cluster <strong>of</strong> smaller abscesses. In addition, there is enhancement <strong>of</strong> the<br />

ependymal lining, suggesting the infectious nature <strong>of</strong> the lesion.<br />

References:<br />

Falini A, Kesavadas C, Pontesilli S, et al. Differential diagnosis <strong>of</strong> posterior fossa multiple sclerosis<br />

lesions--neuroradiological aspects. Neurol Sci 2001;22 (Suppl 2):S79-S83.<br />

Antal EA, Loberg EM, Dietrichs E, Maehlen J. Neuropathological findings in 9 cases <strong>of</strong> listeria monocytogenes<br />

brain stem encephalitis. Brain Pathol 2005;15:187-191.<br />

Question 305: Neuroimaging - Epilepsy<br />

Discussion:<br />

<strong>2007</strong> <strong>RITE</strong> Discussion & Reference Manual<br />

The lesion in the left hippocampus has the imaging characteristics <strong>of</strong> a cavernous angioma, with a rim <strong>of</strong><br />

61


low-intensity hemosiderin surrounding a cluster <strong>of</strong> high-intensity, fluid-containing vessels. The absence <strong>of</strong><br />

draining veins rules out both an arteriovenous malformation and a venous angioma. Astrocytoma and<br />

oligodendroglioma do not have the imaging characteristics present in this MRI.<br />

References:<br />

Greenberg JO. Neuroimaging: a companion to Adams and Victor's principles <strong>of</strong> neurology. New York:<br />

McGraw-Hill, 1999.<br />

Question 307: Neuroimaging - Brain Tumors<br />

Discussion:<br />

The lesion <strong>of</strong> the left optic nerve is an astrocytoma. Neur<strong>of</strong>ibromatosis type 1 (NF1) is associated with the<br />

development <strong>of</strong> both benign and malignant tumors. The left optic nerve is thickened, suggesting the presence <strong>of</strong><br />

a tumor, but it does not enhance, suggesting that the lesion is not a meningioma, neur<strong>of</strong>ibroma, or schwannoma.<br />

Astrocytomas (glioma) are seen in 15% to 20% <strong>of</strong> these individuals. The most common locations include the<br />

optic nerve, optic chiasm, hypothalamus, and, less <strong>of</strong>ten, the posterior fossa.<br />

References:<br />

Gutmann DH, Rasmussen SA, Wolkenstein P, et al. Gliomas Presenting After Age 10 in Individuals with<br />

Neur<strong>of</strong>ibromatosis Type 1 (NF1). Neurology 2002; 59: 759-761.<br />

Question 308: Neuroimaging - Brain Tumors<br />

Discussion:<br />

This extra-axial lesion diffusely and homogenously enhances with contrast. It sits on the dura. The morphology<br />

and location make less likely the diagnoses <strong>of</strong> acoustic neurinoma and trigeminal schwannoma. Astrocytoma<br />

and hemangioma are intra-axial tumors. This is an en-plaque variant meningioma.<br />

References:<br />

Mohr JP, Gautier JC, editors. Guide to Clinical Neurology. New York: Churchill Livingstone, 1995.<br />

Question 309: Neuroimaging - Metabolic<br />

Discussion:<br />

The high subcortical signal in T2, affecting multiple gyri, suggests increased vascular permeability, as it happens<br />

in all the conditions listed as possible answers. However, only eclampsia fails to have other findings and causes<br />

white matter changes with the topography and shape present in the images. Multiple sclerosis tends to cause<br />

sharper lesions, with a periventricular distribution. In abscesses, there are round, cystic collections. There is no<br />

evidence <strong>of</strong> nodular deposits <strong>of</strong> choriocarcinoma. Emboli in these locations would be accompanied by cortical<br />

necrosis, not evident in this image.<br />

References:<br />

Orrison WW. Neuroimaging. Philadelphia: WB Saunders, 2000.<br />

Question 314: Neuroimaging - Stroke<br />

Discussion:<br />

<strong>2007</strong> <strong>RITE</strong> Discussion & Reference Manual<br />

The CT and MRI scan findings are highly consistent with Sturge-Weber syndrome (SWS). Cerebral lesions in<br />

SWS are most frequently found in the parieto-occipital area. On CT scans, parenchymal calcification is found<br />

adjacent to the leptomeningeal lesions in a pericapillary distribution in the fourth cortical layer, possibly<br />

secondary to chronic tissue hypoxia. Noncontrast MR images in SWS typically show unilateral cerebral atrophy<br />

with enlargement <strong>of</strong> the ipsilateral subarachnoid spaces and ventricular cavities. Enlarged tubular regions <strong>of</strong><br />

flow void and gyriform hypointensities (decreased signal on T1-weighted and T2-weighted images) reflect<br />

venous collateralization and cortical "serpentine" calcification, respectively. Contrast-enhanced MRI is best for<br />

demonstrating the salient CNS vascular abnormalities in SWS. The leptomeninges show intense enhancement<br />

62


overlying the involved cortical region secondary to leptomeningeal (pial) angiomatosis or a compromised<br />

blood-brain barrier secondary to chronic cortical ischemia. This meningeal enhancement <strong>of</strong>ten extends well<br />

beyond the region <strong>of</strong> parenchymal atrophy.<br />

References:<br />

Provenzale JM, Tavares JM. Clinical cases in neuroradiology. Philadelphia: Lea & Febiger, 1994.<br />

Question 316: Neuroimaging - Critical Care/Trauma<br />

Discussion:<br />

Pituitary apoplexy is a syndrome <strong>of</strong> infarction or hemorrhage <strong>of</strong> the pituitary gland. It is associated with<br />

headache, ophthalmoparesis, visual defects, and encephalopathy. It may occur acutely or subacutely and can be<br />

associated with coma or death. It occurs more <strong>of</strong>ten in pituitary adenomas.<br />

References:<br />

Rogg JM, Tung GA, Anderson G, Cortez S. Pituitary Apoplexy: Early Detection with Diffusion Weighted MR<br />

Imaging. AJNR Am J Neuroradiol 2002;23:1240-1245.<br />

Question 347: Neuroimaging - Technical<br />

Discussion:<br />

The internal cerebral vein, which is one <strong>of</strong> the major components <strong>of</strong> the deep draining system <strong>of</strong> the brain, is<br />

located in the ro<strong>of</strong> <strong>of</strong> the third ventricle between the leaflets <strong>of</strong> the veli interpositi.<br />

References:<br />

Parent A. Carpenter's human neuroanatomy. 9th ed. Baltimore: Williams & Wilkins, 1996.<br />

Question 356: Neuroimaging - Technical<br />

Discussion:<br />

Of the medical devices and conditions listed, only cardiac pacemakers constitute an absolute contraindication for<br />

MRI. Many heart valves are MR compatible, particularly those produced in the last decade. Middle ear<br />

prosthesis are not considered hazardous to patients, but the device itself can be damaged. An MRI can be<br />

performed in a patient with an aneurysm clip depending on the material used for the clip. Pregnancy is a relative<br />

but not absolute contradiction to MRI. It should be used with caution in the first trimester.<br />

References:<br />

Bushong SC. Magnetic resonance imaging: physical and biological principles. St. Louis: CV Mosby, 1996.<br />

Question 425: Neuroimaging - Stroke<br />

Discussion:<br />

Luxury perfusion refers to increased perfusion in the periphery <strong>of</strong> an ischemic area. It is best documented with<br />

PET or SPECT. Some authors have used the term to refer to gray matter enhancement, which is best seen on<br />

MRI 3 days to several weeks after a cerebral infarct and results from contrast leaking into the perivascular space<br />

in an area <strong>of</strong> ischemic insult.<br />

References:<br />

Bakshi R, Ketonen L. Brain MRI in clinical neurology. In: Joynt RJ, Griggs RC, editors. Baker’s clinical<br />

neurology. Philadelphia: Lippincott, Williams & Wilkins, 2001.<br />

Pathology<br />

Question 17: Pathology - Demyelinating Disease<br />

<strong>2007</strong> <strong>RITE</strong> Discussion & Reference Manual<br />

63


Question 17: Pathology - Demyelinating Disease<br />

Discussion:<br />

Neuromyelitis optica antibody is a newly developed serum antibody test that identifies up to 70% <strong>of</strong> cases <strong>of</strong><br />

neuromyelitis optica (Devic's disease). The test is negative in standard cases <strong>of</strong> multiple sclerosis and in other<br />

demyelinating and disorders disorders.<br />

References:<br />

Lennon VA, Wingerchuk DM, Kryzer TJ, et al. A serum autoantibody marker <strong>of</strong> neuromyelitis optica:<br />

distinction from multiple sclerosis. Lancet 2004;364:2106-2112.<br />

Question 42: Pathology - Demyelinating Disease<br />

Discussion:<br />

Huffing glue causes diffuse leukoencephalopathy, not neuronal necrosis in either cerebral or cerebellar cortices.<br />

References:<br />

Graham DI, Lantos PL. Greenfield's neuropathology. 7th ed. London: Arnold, 2002.<br />

Question 73: Pathology - Cerebrovascular Disease<br />

Discussion:<br />

Dolichoectasia (fusiform aneurysm) most commonly affects the supraclinoid segment <strong>of</strong> the internal carotid<br />

artery and the basilar artery, and is seen in patients with advanced cerebral atherosclerosis. Berry aneurysms are<br />

associated with both coarctation <strong>of</strong> the aorta and arteriovenous malformations (3% to 9% <strong>of</strong> patients with<br />

intracranial AVMs have berry aneurysms), but fusiform aneurysms are not. Fungi cause arteritis and aneurysmal<br />

change but not the elongated vessel distention <strong>of</strong> dolichoectasia. HIV occasionally is associated with but perhaps<br />

not the direct cause <strong>of</strong> smaller vessel vasculitis.<br />

References:<br />

Graham DI, Lantos PL. Greenfield's neuropathology. 7th ed. London: Arnold, 2002.<br />

Question 80: Pathology - Epilepsy<br />

Discussion:<br />

Although the etiology <strong>of</strong> hippocampal (Ammon's horn sclerosis) is controversial, with some authors contending<br />

the lesions are the cause, and others the result, <strong>of</strong> the seizures, the lesion is nonetheless the most common<br />

pathology found in patients with tissue resected for temporal lobe epilepsy. Seizure control after temporal<br />

lobectomy is best with hippocampal sclerosis but is also very good with neoplasms and vascular malformations,<br />

although these are less common causes <strong>of</strong> temporal lobe epilepsy. End folium sclerosis with neuronal loss<br />

confined to the CA4 sector <strong>of</strong> the hippocampus is rare, as is Rasmussen's encephalitis.<br />

References:<br />

Burger P,Scheithauer B,Vogel FS. Surgical Pathology <strong>of</strong> the Nervous System and Its Coverings. 4th ed.<br />

Philadelphia: Churchill Livingstone, 2002.<br />

Question 93: Pathology - Critical Care/Trauma<br />

Discussion:<br />

Hemorrhage and necrosis <strong>of</strong> the central core <strong>of</strong> the spinal cord usually results from traumatic subluxation injury<br />

<strong>of</strong> the cervical spine. Rarely, hematomyelia can also result from intraspinal vascular malformations.<br />

References:<br />

<strong>2007</strong> <strong>RITE</strong> Discussion & Reference Manual<br />

Graham DI, Lantos PL. Greenfield's neuropathology. 7th ed. London: Arnold, 2002.<br />

64


Question 104: Pathology - Epilepsy<br />

Discussion:<br />

Of the tumor types listed, the most likely to show a superficial cystic appearance and to occur in the temporal<br />

lobe <strong>of</strong> a young adult is the pleomorphic xanthoastrocytoma (PXA), a WHO grade II tumor. Eosinophilic<br />

granular bodies are histological features most commonly found in low grade gliomas, such as pleomorphic<br />

xanthoastrocytoma. They are small aggregates <strong>of</strong> eosinophilic, hyaline material situated in the processes <strong>of</strong><br />

astrocytes.<br />

References:<br />

Burger P,Scheithauer B,Vogel FS. Surgical Pathology <strong>of</strong> the Nervous System and Its Coverings. 4th ed.<br />

Philadelphia: Churchill Livingstone, 2002.<br />

Question 133: Pathology - Tumors<br />

Discussion:<br />

Angiotropic (intravscular) lymphoma is a disease <strong>of</strong> multiple organs that involves the central nervous system in<br />

greater than 30% <strong>of</strong> cases. It is usually a B cell lymphoma by immunophenotyping, and causes multiple small<br />

brain infarcts, <strong>of</strong>ten <strong>of</strong> deep white matter. A patient with CNS involvement by angiotropic lymphoma may show<br />

lymphoma cells only within the blood vessels and may not have either intraparenchymal lymphomatous masses<br />

or significant meningeal involvement. The old name for this tumor prior to the advent <strong>of</strong> specific<br />

immunohistochemical markers for lymphoma was "malignant angioendotheliomatosis", reflceting the erroneous<br />

belief that the tumor cells were endothelial in origin.<br />

References:<br />

Graham DI, Lantos PL. Greenfield's neuropathology. 7th ed. London: Arnold, 2002.<br />

Question 135: Pathology - Toxic/Metabolic Disease<br />

Discussion:<br />

Peripheral neuropathies can be caused by inorganic arsenic, lead, mercury, and thallium. Aluminum intoxication<br />

can produce an encephalopathy but does not produce a neuropathy. Cyanide, manganese, and bismuth do not<br />

cause peripheral neuropathy.<br />

References:<br />

Graham DI, Lantos PL. Greenfield's neuropathology. 7th ed. London: Arnold, 2002.<br />

Question 139: Pathology - Hypothalmus/Pituitary<br />

Discussion:<br />

Bromocriptine (Parlodel) and cabergoline are dopamine agonists that function by the same mechanism as<br />

prolactin inhibitory factor (PIF, which is dopamine) to inhibit prolactin synthesis and release by<br />

prolactin-secreting pituitary adenomas (prolactinomas). Bromocriptine or cabergoline treatment causes a<br />

decrease in the tumor cell cytoplasmic volume without actually killing the adenoma cells. Therefore, the<br />

prolactinoma may reexpand if bromocriptine is withdrawn. Bromocriptine has no significant role in<br />

neuro-oncology other than in suppression <strong>of</strong> prolactin-secreting tumors.<br />

References:<br />

Burger P,Scheithauer B,Vogel FS. Surgical Pathology <strong>of</strong> the Nervous System and Its Coverings. 4th ed.<br />

Philadelphia: Churchill Livingstone, 2002.<br />

Question 156: Pathology - Neurodegenerative Disease<br />

Discussion:<br />

<strong>2007</strong> <strong>RITE</strong> Discussion & Reference Manual<br />

65


Familial frontotemporal dementia that arises from a mutation on chromosome 17 results in a mutation <strong>of</strong> the<br />

gene encoding for tau protein.<br />

References:<br />

Ellison D, Love S, Chimelli L, et al. Neuropathology: A Reference Text <strong>of</strong> CNS Pathology. 2nd ed. Edinburgh:<br />

Mosby, 2004.<br />

Question 164: Pathology - Critical Care/Trauma<br />

Discussion:<br />

Critical care myopathy is an underrecognized disorder characterized by selective loss <strong>of</strong> myosin from my<strong>of</strong>ibers.<br />

Also known as myosin-losing myopathy, this disorder is most commonly seen in critically ill patients treated<br />

with corticosteroids and neuromuscular blockers.<br />

References:<br />

Ruff RL. Why do ICU patients become paralyzed? Ann Neurol 1998;43:154-155.<br />

Question 180: Pathology - Toxic/Metabolic Disease<br />

Discussion:<br />

Hemorrhagic necrosis <strong>of</strong> the putamen is characteristic <strong>of</strong> methanol poisoning. Methanol may be ingested as an<br />

alcohol substitute. There is selective vulnerability <strong>of</strong> the putamen, among the choices listed.<br />

References:<br />

Schochet SS, Gray F. Acquired Metabolic Disorders. In: Gray F, De Girolami U, Poirier J. Escourolle and<br />

Poirier Manual <strong>of</strong> Basic Neuropathology. 4th ed. Boston: Butterworth Heinemann 2004; 198.<br />

Question 187: Pathology - Neurodegenerative Disease<br />

Discussion:<br />

Corticobasal degeneration classically presents with progressive asymmetric apraxias, rigidity, and/or aphasia due<br />

to neuronal loss and atrophy <strong>of</strong> the peri-Sylvian region. Ballooned neurons and astrocytic plaques with<br />

tau-immunoreactivity are a diagnostic feature <strong>of</strong> corticobasal degeneration.<br />

References:<br />

Graham DI, Lantos PL. Greenfield's neuropathology. 7th ed. London: Arnold, 2002.<br />

Questions 224 - 228: Pathology - Neuromuscular Disease<br />

Discussion:<br />

Perifascicular atrophy is virtually pathognomonic <strong>of</strong> dermatomyositis and is particular characteristic <strong>of</strong> juvenile<br />

dermatomyositis. The image is <strong>of</strong> an onion bulb or hypertrophic neuropathy and best matches a clinical scenario<br />

suggestive <strong>of</strong> an indolent demyelinating neuropathy such as Charcot-Marie-Tooth disease 1a. The figure depicts<br />

a ragged-red fiber as are seen in some, but not all mitochondrial encephalomyopathies. The ocular system,<br />

including the eye and ocular muscles are frequently involved in mitochondrial disease. This shows an H&E<br />

stained muscle fiber with a rimmed vacuole, characteristic <strong>of</strong> the most common inflammatory in older adults,<br />

inclusion body myositis. Neurogenic atrophy (group atrophy) <strong>of</strong> skeletal muscle is characteristic <strong>of</strong> spinal<br />

muscular atrophy type I (Werdnig-H<strong>of</strong>fman disease).<br />

References:<br />

<strong>2007</strong> <strong>RITE</strong> Discussion & Reference Manual<br />

Lidov H, De Girolami U, Gherardi R. Skeletal muscle diseases. In: Gray F, De Girolami U, Poirier J. Escourolle<br />

and Poirier Manual <strong>of</strong> Basic Neuropathology. 4th ed. Boston: Butterworth-Heinemann, 2004;<br />

287,302-304,309-312,308-310.<br />

Vallat J-M, Anthony D, De Girolami U. Peripheral Nerve Diseases. In: Gray F, De Girolami U, Poirier J.<br />

66


Escourolle and Poirier Manual <strong>of</strong> Basic Neuropathology. 4th ed. Boston: Butterworth-Heinemann 2004; 337-338.<br />

Question 229: Pathology - Developmental<br />

Discussion:<br />

This brain shows multiple areas <strong>of</strong> cavitation (liquefaction necrosis) involving the deeper cortical layers and<br />

underlying white matter, a condition known as multicystic encephalopathy or multicystic encephalomalacia.<br />

The lesions are most pronounced in the distribution <strong>of</strong> the anterior and middle cerebral arteries. They are the<br />

result <strong>of</strong> circulatory disturbances during the latter half <strong>of</strong> pregnancy or the neonatal period.<br />

References:<br />

Nelson J, Mena H, Parisi JE, Schochet SS, editors. Principles and Practice <strong>of</strong> Neuropathology. 2nd ed. New<br />

York: Oxford University Press, 2003.<br />

Question 236: Pathology - Tumors<br />

Discussion:<br />

The biopsy <strong>of</strong> a right parietal lobe mass showed a tumor composed <strong>of</strong> cells with monotonous round nuclei<br />

surrounded by prominent perinuclear halos ("fried egg" apppearance). These features are characteristic <strong>of</strong><br />

oligodendroglioma. Of the other choices listed, ependymomas and astroblastomas characteristically show<br />

perivascular pseudorosettes, with stout nontapering cytoplasmic processes in the latter. Fibrillary astrocytomas<br />

and gemistocytic astrocytomas show eosinophilic cytoplasm rather than the clear perinuclear halos <strong>of</strong><br />

oligodendrogliomas, with multiple elongated cytoplasmic processes in fibrillary astrocytomas and large rounded<br />

globular cytoplasm in gemistocytic astrocytomas.<br />

References:<br />

Burger P,Scheithauer B,Vogel FS. Surgical Pathology <strong>of</strong> the Nervous System and Its Coverings. 4th ed.<br />

Philadelphia: Churchill Livingstone, 2002.<br />

Question 239: Pathology - Tumors<br />

Discussion:<br />

The microscopic appearance is that <strong>of</strong> a capillary-rich neoplasm with abundant foamy cells containing lipid. This<br />

is a hemangioblastoma. The cell origin <strong>of</strong> the hemangioblastoma is still unknown, despite many investigative<br />

efforts. The tumor arises independently or in conjunction with von Hippel-Lindau disease. The most common<br />

location is in the cerebellum, but the spinal cord can also be a site <strong>of</strong> origin. Pilocytic astrocytomas, also<br />

commonly found in the cerebellum, are composed <strong>of</strong> astrocytes with elongate, bipolar, eosinophilic cell<br />

processes; these tumors <strong>of</strong>ten show Rosenthal fibers. Medulloblastomas are malignant small blue cell tumors <strong>of</strong><br />

neuronal origin that manifest little visible cytoplasm. Ependymomas are glial tumors which show formation <strong>of</strong><br />

ependymal canals, or more commonly, perivascular pseudorosettes.<br />

References:<br />

Burger P,Scheithauer B,Vogel FS. Surgical Pathology <strong>of</strong> the Nervous System and Its Coverings. 4th ed.<br />

Philadelphia: Churchill Livingstone, 2002.<br />

Question 248: Pathology - Cerebrovascular Disease<br />

Discussion:<br />

Autosomal dominant cerebral cavernous angioma (malformation) (CCM1) syndrome occurs in Hispanic<br />

individuals and is characterized by intraparenchymal cavernous malformations that can produce seizures,<br />

impairment <strong>of</strong> function, or hemorrhage. The photograph illustrates a multi-channeled vascular malformation<br />

composed <strong>of</strong> closely juxtaposed hyalinized, non-arterial vessels.<br />

References:<br />

<strong>2007</strong> <strong>RITE</strong> Discussion & Reference Manual<br />

Ellison D, Love S, Chimelli L, et al. Neuropathology: A Reference Text <strong>of</strong> CNS Pathology. 2nd ed. Edinburgh:<br />

67


Mosby, 2004.<br />

Question 249: Pathology - Neuromuscular Disease<br />

Discussion:<br />

The muscle biopsy specimen shows a necrotizing vasculitis with fibrinoid necrosis <strong>of</strong> the vessel walls, not<br />

amyloid angiopathy. Vasculitis may be seen in amphetamine-induced vasculitis, polyarteritis nodosa, rheumatoid<br />

vasculitis, and Wegener's granulomatosis, but not in polymyalgia rheumatica which is associated with temporal<br />

arteritis and usually type II fiber atrophy in the muscle biopsy, not vasculitis. Takayasu's arteritis affects large<br />

blood vessels such as aortic arch, not small intramuscular vessels. ALS causes no inflammation <strong>of</strong> intramuscular<br />

blood vessels, but does show neurogenic atrophy.<br />

References:<br />

Nelson J, Mena H, Parisi JE, Schochet SS, editors. Principles and Practice <strong>of</strong> Neuropathology. 2nd ed. New<br />

York: Oxford University Press, 2003.<br />

Question 251: Pathology - Cerebrovascular Disease<br />

Discussion:<br />

The lesion shown in this image is a remote infarct. The preservation <strong>of</strong> the outer layer <strong>of</strong> the cortex would argue<br />

against a remote contusion. An old hematoma cavity would be smoother, and would not necessairly be confined<br />

to the vascular distribution territory <strong>of</strong> a branch <strong>of</strong> the middle cerebral artery as this lesion is. Both an abscess<br />

and metastatic carcinoma would be accompanied by brain swelling rather than shrinkage; note the lack <strong>of</strong> mass<br />

effect on the ventricular system.<br />

References:<br />

Graham DI, Lantos PL. Greenfield's neuropathology. 7th ed. London: Arnold, 2002.<br />

Question 252: Pathology - Cerebrovascular Disease<br />

Discussion:<br />

The gross findings are those <strong>of</strong> a remote hemorrhagic stroke. Microscopically, the blood vessels <strong>of</strong> the meninges<br />

and cortex showed amorphous eosinophilic material indicative <strong>of</strong> cerebral amyloid angiopathy. Neither primary<br />

or secondary systemic amyloidoses cause amyloid deposits within cerebral blood vessels. Cerebral autosomal<br />

dominant arteriopathy with subcortical infarcts and leukoencephalopathy (CADASIL) deposits would not stain<br />

with amyloid stains such as thi<strong>of</strong>lavin S or Congo red and would not be as homongeneous or affect the<br />

superficial cortex and meninges. Chronic hypertension causes thickened walls <strong>of</strong> small blood vessels due to<br />

hyalinization and lipohyalinosis; no amyloid is deposited.<br />

References:<br />

Graham DI, Lantos PL. Greenfield's neuropathology. 7th ed. London: Arnold, 2002.<br />

Question 263: Pathology - Developmental<br />

Discussion:<br />

The defect is anencephaly, the most common <strong>of</strong> the neural tube closure defects. Folate supplementation has been<br />

known to lower the recurrence risk <strong>of</strong> these defects however a recent report also documents the presence <strong>of</strong><br />

anti-folate receptor antibodies in some pregnancies complicated by neural tube closure defects.<br />

References:<br />

<strong>2007</strong> <strong>RITE</strong> Discussion & Reference Manual<br />

Rothenberg SP, da Casta M, Sequeira J, et al. Autoantibodies against Folate Receptors in Women with a<br />

Pregnancy Complicated by a Neural-Tube Defect. N Engl J Med 2004;350:134-142.<br />

Encha-Razavi F, Folkerth R, Harding B. Congenital Malformations and Perinatal Diseases. In: Gray F, De<br />

Girolami U, Poirier J. Escourolle and Poirier Manual <strong>of</strong> Basic Neuropathology. 4th ed. Boston:<br />

68


Butterworth-Heinemann, 2004; 250.<br />

Question 264: Pathology - Critical Care/Trauma<br />

Discussion:<br />

The graphic shows an acute subdural hematoma. Subdural hematomas result from disruption <strong>of</strong> bridging veins<br />

and are seen more commonly in older persons with cortical atrophy with resultant tension on these veins. The<br />

inciting trauma <strong>of</strong>ten is a fall, but may not be recalled.<br />

References:<br />

Graham DI, Lantos PL. Greenfield's neuropathology. 7th ed. London: Arnold, 2002.<br />

Question 269: Pathology - Demyelinating Disease<br />

Discussion:<br />

The photograph shows syringomyelia. The lesion consists <strong>of</strong> a large cystic space, which would not be seen with<br />

subacute combined degeneration, AIDS-associated myelopathy, or mutliple sclerosis. Syringomyelia is a cystic<br />

lesion in the spinal cord that interrupts the crossing pain fibers and damages anterior horn cells. Muscle atrophy,<br />

<strong>of</strong>ten results but vibratory, position, and touch senses are usually spared.<br />

References:<br />

Graham DI, Lantos PL. Greenfield's neuropathology. 7th ed. London: Arnold, 2002.<br />

Question 271: Pathology - Infectious Disease<br />

Discussion:<br />

The photograph shows abundant exudate, especially along vessels, in the subarachnoid space on the dorsal<br />

surface <strong>of</strong> the brain. This is most consistent with purulent leptomeningitis. Hemophilus influenzae rarely causes<br />

meningitis in the adult, and pneumococcal meningitis is more common than meningococcal meningitis in the<br />

elderly patient. With this history <strong>of</strong> alcoholism and asplenism, the most likely diagnosis is pneumococcal<br />

meningitis.<br />

References:<br />

Nelson J, Mena H, Parisi JE, Schochet SS, editors. Principles and Practice <strong>of</strong> Neuropathology. 2nd ed. New<br />

York: Oxford University Press, 2003.<br />

Question 273: Pathology - Basic Reactions<br />

Discussion:<br />

The photograph shows Wallerian degeneration, with shrinkage <strong>of</strong> one medullary pyramid and atrophy <strong>of</strong> the<br />

ipsilateral cerebral peduncle. The Wallerian degeneration is secondary to destruction <strong>of</strong> the corticospinal tract<br />

above this level, such as an infarction in the posterior limb <strong>of</strong> the internal capsule.<br />

References:<br />

Graham DI, Lantos PL. Greenfield's neuropathology. 7th ed. London: Arnold, 2002.<br />

Question 275: Pathology - Neurodegenerative Disease<br />

Discussion:<br />

The gross photograph <strong>of</strong> the brain shows lobar atrophy. Lobar atrophy <strong>of</strong> the frontal and temporal lobes is<br />

typical <strong>of</strong> Pick's disease. Microscopically, intracytoplasmic argyrophilic neuronal inclusions and gliosis are<br />

observed in Pick body Pick's disease.<br />

References:<br />

<strong>2007</strong> <strong>RITE</strong> Discussion & Reference Manual<br />

69


Graham DI, Lantos PL. Greenfield's neuropathology. 7th ed. London: Arnold, 2002.<br />

Question 278: Pathology - Demyelinating Disease<br />

Discussion:<br />

The spinal cord shows tract degeneration in the dorsal spinocerebellar columns and posterior columns, with<br />

myelin loss highlighted by the Luxol fast blue-periodic acid Schiff stain for myelin. Tract degeneration is seen in<br />

Friedreich's ataxia, but not multiple sclerosis or acute disseminated encephalomyelitis. Tabes dorsalis affects the<br />

posterior columns <strong>of</strong> the spinal cord, but not the dorsal spinocerebellar tracts.<br />

References:<br />

Graham DI, Lantos PL. Greenfield's neuropathology. 7th ed. London: Arnold, 2002.<br />

Question 282: Pathology - Prion Disease<br />

Discussion:<br />

The image shows spongiform change <strong>of</strong> the cerebral cotex consistent with a prion disease. Neuronal loss and<br />

gliosis complete the classic histologic triad. Prion diseases are caused by misfolding <strong>of</strong> a normal cellular protein<br />

PrP. Such misfolding may be induced by mutation or by exposure <strong>of</strong> normal cellular prion protein to pathogenic<br />

prions. Misfolding alters the secondary structure such that the protein becomes highly resistant to chemical or<br />

thermal methods <strong>of</strong> strerilization.<br />

References:<br />

Graham DI, Lantos PL. Greenfield's neuropathology. 7th ed. London: Arnold, 2002.<br />

Question 291: Pathology - Neurodegenerative Disease<br />

Discussion:<br />

The brain shows severe atherosclerosis <strong>of</strong> intracranial vessels as well as bilateral remote anterior cerebral artery<br />

infarctions. The ventricles are slightly enlarged due to hydrocephalus ex vacuo. This patient actually had a<br />

single, unpaired anterior cerebral artery, accounting for the bilateral nature <strong>of</strong> the infarctions. Bilateral anterior<br />

cerebral artery territory infarcts <strong>of</strong>ten result in pr<strong>of</strong>ound apathy, lack <strong>of</strong> motivational intent, akinetic mutism,<br />

weakness <strong>of</strong> the lower extremities and urinary incontinence.<br />

References:<br />

Ellison D, Love S, Chimelli L, et al. Neuropathology: A Reference Text <strong>of</strong> CNS Pathology. 2nd ed. Edinburgh:<br />

Mosby, 2004.<br />

Question 297: Pathology - Cerebrovascular Disease<br />

Discussion:<br />

The photograph shows a massive basal ganglionic hemorrhage with rupture into the ventricular system. The<br />

location <strong>of</strong> this hematoma is common for hypertensive intracerebral hemorrhage. Hypertension is a major risk<br />

factor for this type <strong>of</strong> hemorrhage, and hypertension is associated with pathologic and electrocardiographic<br />

evidence <strong>of</strong> left ventricular hypertrophy. Prostate cancer only rarely metastasizes to brain parenchyma and is<br />

usually not hemorrhagic. Visceral cysts are not relevant to intracerebral hemorrhage, but are frequently seen in<br />

von Hippel-Lindau disease with associated CNS hemangioblastoma.<br />

References:<br />

Graham DI, Lantos PL. Greenfield's neuropathology. 7th ed. London: Arnold, 2002.<br />

Question 298: Pathology - Infectious Disease<br />

Discussion:<br />

<strong>2007</strong> <strong>RITE</strong> Discussion & Reference Manual<br />

70


Fungal infections with Coccidioides occur in regions <strong>of</strong> semiarid climate. In the United States, the Southwest<br />

and California are the most common locations for these infections. Mature fungi in tissue are endospores, such<br />

as the one pictured, which are round and have a refractile wall.<br />

References:<br />

Nelson J, Mena H, Parisi JE, Schochet SS, editors. Principles and Practice <strong>of</strong> Neuropathology. 2nd ed. New<br />

York: Oxford University Press, 2003.<br />

Question 310: Pathology - Developmental<br />

Discussion:<br />

This photograph <strong>of</strong> a coronal section <strong>of</strong> immature brain shows small foci <strong>of</strong> dystrophic calcification in white<br />

matter corresponding to periventricular leukomalacia (PVL). This is the principal ischemic lesion <strong>of</strong><br />

prematurity. Microscopy would show coagulative necrosis, dystrophic calcifications, microglial activation, and<br />

axonal injury.<br />

References:<br />

Encha-Razavi F, Folkerth R, Harding B. Congenital Malformations and Perinatal Diseases. In: Gray F, De<br />

Girolami U, Poirier J. Escourolle and Poirier Manual <strong>of</strong> Basic Neuropathology. 4th ed. Boston:<br />

Butterworth-Heinemann, 2004; 267.<br />

Question 315: Pathology - Cerebrovascular Disease<br />

Discussion:<br />

The lesion is seen in the cortex <strong>of</strong> the gyri to the right <strong>of</strong> midline in this coronal section and is a typical example<br />

<strong>of</strong> laminar necrosis due to hypoxic/ischemic injury.<br />

References:<br />

Schochet SS, Gray F. Acquired Metabolic Disorders. In: Gray F, De Girolami U, Poirier J. Escourolle and<br />

Poirier Manual <strong>of</strong> Basic Neuropathology. 4th ed. Boston: Butterworth Heinemann 2004; 198.<br />

Question 318: Pathology - Demyelinating Disease<br />

Discussion:<br />

Marburg's disease, also known as acute multiple sclerosis, can cause severe cerebral damage and render a<br />

patient comatose within a few weeks. The patient does not go into remission and may die within weeks or<br />

months after onset <strong>of</strong> the demyelinating disorder.<br />

References:<br />

Fernandez H, Eisenschenk S, Yachnis AT, Okun MS. Ultimate review for the neurology boards. 1st ed. New<br />

York: Demos Medical Publishing, 2006.<br />

Question 329: Pathology - Neurodegenerative Disease<br />

Discussion:<br />

The nucleus <strong>of</strong> Onufrowicz <strong>of</strong> the sacral cord is spared in motor neuron disease. The remaining motor nuclei are<br />

involved to varying degrees in amyotrophic lateral sclerosis.<br />

References:<br />

Graham DI, Lantos PL. Greenfield's neuropathology. 7th ed. London: Arnold, 2002.<br />

Question 338: Pathology - Infectious Disease<br />

Discussion:<br />

<strong>2007</strong> <strong>RITE</strong> Discussion & Reference Manual<br />

71


In tabes dorsalis, the degeneration is confined to the dorsal columns, whereas in AIDS myelopathy, amyotrophic<br />

lateral sclerosis, Friedreich's ataxia, and pernicious anemia degeneration occurs in other tracts.<br />

References:<br />

Graham DI, Lantos PL. Greenfield's neuropathology. 7th ed. London: Arnold, 2002.<br />

Question 342: Pathology - Neurodegenerative Disease<br />

Discussion:<br />

Choline acetyltransferase is the synthetic enzyme for acetylcholine and is a marker <strong>of</strong> the axonal termini <strong>of</strong><br />

cholinergic neurons. The neurons <strong>of</strong> the basal nucleus are the major cholinergic projections to the cerebral<br />

cortex, and with depopulation <strong>of</strong> this projection nucleus there is a corresponding reduction in cortical choline<br />

acetyltransferase.<br />

References:<br />

Graham DI, Lantos PL. Greenfield's neuropathology. 7th ed. London: Arnold, 2002.<br />

Question 351: Pathology - Neurodegenerative Disease<br />

Discussion:<br />

Paraneoplastic cerebellar degneration is most commonly associated with ovarian carcinoma and some breast<br />

tumors. It can also be seen with Hodgkin's lymphoma and small cell carcinoma <strong>of</strong> the lung.<br />

References:<br />

Ellison D, Love S, Chimelli L, et al. Neuropathology: A Reference Text <strong>of</strong> CNS Pathology. 2nd ed. Edinburgh:<br />

Mosby, 2004.<br />

Question 357: Pathology - Toxic/Metabolic Disease<br />

Discussion:<br />

Ethylene glycol toxicity has birefringent calcium oxalate deposits in and around blood vessels. Oxalic acid<br />

crystals may be detected in the urine.<br />

References:<br />

Graham DI, Lantos PL. Greenfield's neuropathology. 7th ed. London: Arnold, 2002.<br />

Question 365: Pathology - Demyelinating Disease<br />

Discussion:<br />

Cerebral autosomal dominant arteriopathy with subcortical infarcts and leukoencephalopathy (CADASIL)<br />

results from missense mutations <strong>of</strong> the Notch 3 gene on chromosome 19. The small deep white matter vessels<br />

show replacement <strong>of</strong> the media by eosinophilic, periodic acid-Schiff positive, Congo red negative, granular<br />

material. Ultrastructurally, there is compact electron-dense material known as granular osmophilic material<br />

surrounding myocytes in the arterial media. Although the predominant effects <strong>of</strong> CADASIL are on the central<br />

nervous system, it is a systemic vasculopathy that can be appreciated on skin or nerve biopsy. Vasculitis is not<br />

seen in CADASIL.<br />

References:<br />

Ellison D, Love S, Chimelli L, et al. Neuropathology: A Reference Text <strong>of</strong> CNS Pathology. 2nd ed. Edinburgh:<br />

Mosby, 2004.<br />

Question 375: Pathology - Tumors<br />

Discussion:<br />

<strong>2007</strong> <strong>RITE</strong> Discussion & Reference Manual<br />

72


Patients with tuberous sclerosis have a variety <strong>of</strong> systemic and CNS tumors. In the brain, astrocytic hamartomas<br />

called tubers and candle gutterings are present. Subependymal giant cell astrocytomas are true neoplasms arising<br />

in the vicinity <strong>of</strong> the foramen <strong>of</strong> Monro. These tumors may produce CSF obstruction, and they may rarely<br />

undergo malignant degeneration.<br />

References:<br />

Burger P,Scheithauer B,Vogel FS. Surgical Pathology <strong>of</strong> the Nervous System and Its Coverings. 4th ed.<br />

Philadelphia: Churchill Livingstone, 2002.<br />

Kleihues P, Cavenee WK, editors. Pathology and genetics <strong>of</strong> tumors <strong>of</strong> the nervous system. 2nd ed. New York:<br />

Oxford University Press, 2000.<br />

Question 385: Pathology - Critical Care/Trauma<br />

Discussion:<br />

The injury described above fits best with a contrecoup type injury. The individual decelerates when he falls and<br />

strikes the back <strong>of</strong> his head. Because the head was in motion and he struck his occiput, the contrecoup sites will<br />

be the frontal lobes and temporal tips. The irregular bony contours within the cranial vault in the anterior and<br />

middle fossae exacerbate the injury.<br />

References:<br />

Nelson J, Mena H, Parisi JE, Schochet SS, editors. Principles and Practice <strong>of</strong> Neuropathology. 2nd ed. New<br />

York: Oxford University Press, 2003.<br />

Question 395: Pathology - Tumors<br />

Discussion:<br />

Rhabdoid meningioma is a variant <strong>of</strong> meningioma that <strong>of</strong>ten has aggressive clinical behavior with numerous<br />

recurrences and the potential for metastasis. It is a World Health Organization (WHO) grade III tumor. Many<br />

meningiomas have distinctive histological appearances, such as secretory, fibrous, and psammomatous subtypes,<br />

but have no adverse prognosis associated specifically with their morphology and are usually WHO grade I<br />

tumors.<br />

References:<br />

Burger P,Scheithauer B,Vogel FS. Surgical Pathology <strong>of</strong> the Nervous System and Its Coverings. 4th ed.<br />

Philadelphia: Churchill Livingstone, 2002.<br />

Question 426: Pathology - Basic Reactions<br />

Discussion:<br />

The type <strong>of</strong> edema seen with acute, obstructive, high-pressure hydrocephalus is interstitial edema. Hydrostatic<br />

edema is seen when high arterial pressures overcome cerebrovascular resistance and drive water across the<br />

capillary walls into the extracellular space. Vasogenic edema is the most frequent type <strong>of</strong> edema and is seen<br />

around tumors or traumatic lesions. Hypo-ostomic edema occurs with marked reduction in serum osmolality,<br />

and cytotoxic edema is most commonly caused by hypoxia.<br />

References:<br />

Graham DI, Lantos PL. Greenfield's neuropathology. 7th ed. London: Arnold, 2002.<br />

Question 445: Pathology - Hypothalmus/Pituitary<br />

Discussion:<br />

<strong>2007</strong> <strong>RITE</strong> Discussion & Reference Manual<br />

Neurosarcoidosis is uncommon, even in patients with systemic sarcoidosis. Of the anatomic sites mentioned, the<br />

hypothalamus is most commonly affected. It may be assoicated with a peripheral neuropathy or myopathy and<br />

on biopsy is a histological diagnosis <strong>of</strong> exclusion. Specifically, fungi and tuberculosis must be eliminated as<br />

73


diagnoses in cases with granulomatous inflammation.<br />

References:<br />

Ellison D, Love S, Chimelli L, et al. Neuropathology: A Reference Text <strong>of</strong> CNS Pathology. 2nd ed. Edinburgh:<br />

Mosby, 2004.<br />

Pharmacology/Chemistry<br />

Question 5: Pharmacology/Chemistry - Movement Disorders<br />

Discussion:<br />

Suboptimal doses <strong>of</strong> carbidopa are a frequent cause <strong>of</strong> nausea/vomiting at the initiation <strong>of</strong> levodopa therapy. It<br />

takes 100 mg to 150 mg <strong>of</strong> carbidopa per day to saturate the peripheral aromatic aminoacid decarboxylase<br />

enzyme to prevent peripheral side effects <strong>of</strong> levodopa. Promethaziine and metaclopramide block dopamine<br />

receptors and may worsen the parkinsonism.<br />

References:<br />

Roger E, Kurlan JB. Treatment <strong>of</strong> movement disorders. Philadelphia: Lippincott, 1995.<br />

Question 18: Pharmacology/Chemistry - Neuromuscular Disorders<br />

Discussion:<br />

Vitamin B12 (cobalamin) plays an important role in DNA synthesis and neurologic function. Deficiency can lead<br />

to a wide spectrum <strong>of</strong> hematologic, neurologic, and psychiatric manifestations. It is a common cause <strong>of</strong><br />

macrocytic (megaloblastic) anemia and, in advanced cases, pancytopenia. Neurologic sequelae from vitamin B12<br />

deficiency include paresthesias, peripheral neuropathy, and demyelination <strong>of</strong> the corticospinal tract and dorsal<br />

columns (subacute combined systems disease). The diagnosis <strong>of</strong> vitamin B12 deficiency has traditionally been<br />

based on low serum vitamin B12 levels, usually less than 200 pg per mL (150 pmol per L), along with clinical<br />

evidence <strong>of</strong> disease. However, studies indicate that older patients tend to present with neuropsychiatric disease in<br />

the absence <strong>of</strong> hematologic findings.Furthermore, measurements <strong>of</strong> metabolites such as methylmalonic acid and<br />

homocysteine have been shown to be more sensitive in the diagnosis <strong>of</strong> vitamin B12 deficiency than<br />

measurement <strong>of</strong> serum B12 levels alone<br />

References:<br />

Oh RC, Brown DL. Vitamin B12 Deficiency. Am Fam Physician 2003; 67(5):979-986.<br />

Question 20: Pharmacology/Chemistry - Headache<br />

Discussion:<br />

Giant cell emporal arteritis is a vasculitis can lead to devastating ophthalmic and systemic complications. Jaw<br />

claudication, superficial temporal artery tenderness, fever, and visual loss are seen. Treatment primarily to<br />

prevent visual loss requires extended management with corticosteroids - initially at a high dose (e.g. Prednisone<br />

80-100 mg/d).<br />

References:<br />

Bhatti MT, Tabandeh H. Giant cell arteritis: diagnosis and management. Curr Opin Ophthalmol<br />

2001;12(6):393-399.<br />

Question 21: Pharmacology/Chemistry - Headache<br />

Discussion:<br />

<strong>2007</strong> <strong>RITE</strong> Discussion & Reference Manual<br />

Sumatriptan and other triptans are felt to provide acute relief for migraine by acting as 5HT1 receptors,<br />

specifically 1b and 1d, resulting in decreased activity in the trigeminovascular system. Drugs acting at<br />

adrenergic, specifically beta and alpha, receptors provide prophylactic effects.<br />

74


References:<br />

Tepper SJ, Rapoport AM, Sheftell FD. Mechanisms <strong>of</strong> action <strong>of</strong> the 5-HT1B/1D receptor agonists. Arch Neurol<br />

2002;59(7):1084-1088.<br />

Question 25: Pharmacology/Chemistry - Other Pain Syndromes<br />

Discussion:<br />

Transient receptor potential (TRP) ion channels are molecular gateways in sensory systems, an interface between<br />

the environment and the nervous system. Several TRPs transduce thermal, chemical, and mechanical stimuli<br />

into inward currents, an essential first step for eliciting thermal and pain sensations. Precise regulation <strong>of</strong> the<br />

expression, localization, and function <strong>of</strong> the TRP channels is crucial for their sensory role in nociceptor<br />

terminals, particularly after inflammation when they contribute to pain hypersensitivity by undergoing changes<br />

in translation and trafficking as well as diverse posttranslational modifications.<br />

References:<br />

Story GM, Gereau RW 4th. Numbing the senses: role <strong>of</strong> TRPA1 in mechanical and cold sensation. Neuron<br />

2006;50(2):177-180.<br />

Wang H, Woolf CJ. Pain TRPs. Neuron 2005;46(1):9-12.<br />

Question 26: Pharmacology/Chemistry - Headache<br />

Discussion:<br />

Cluster headache will <strong>of</strong>ten respond acutely to oxygen inhalation at a flow rate <strong>of</strong> 8 L/min to 10 L/min via face<br />

mask. It may be triggered or exacerbated by vasodilating substances such as nitroglycerin, histamine, and<br />

ethanol. Drugs such as triptans, steroids, and dihydroergotamine may also be helpful in acute headache<br />

management. Verapamil, lithium, and methysergide can be effective prophylactic agents for cluster headaches.<br />

Oral analgesics such as codeine are generally ineffective in treating cluster headache, as are anticonvulsants and<br />

diuretics.<br />

References:<br />

Campbell JK, Caselli RJ. Headache and other crani<strong>of</strong>acial pain. In: Bradley WG, Dar<strong>of</strong>f RB, Fenichel GM, et al,<br />

editors. Neurology in clinical practice. 2nd ed. Boston: Butterworth-Heinemann, 1996.<br />

Question 32: Pharmacology/Chemistry - Cerebrovascular Disease<br />

Discussion:<br />

Cessation <strong>of</strong> electrical activity due to hyperpolarization that occurs acutely after an ischemic insult is the result<br />

<strong>of</strong> activation <strong>of</strong> potassium (K+) membrane channels.<br />

References:<br />

Dugan LL, Kim-Han JS. Hypoxia-ischemic brain injusry and oxidative stress. In: Siegel GJ, Albers RW, Brady<br />

ST, Price DL, editors. Basic Neurochemistry: molecular, cellular, and medical aspects. Boston: Elsevier,<br />

2006;559-573.<br />

Question 50: Pharmacology/Chemistry - Epilepsy<br />

Discussion:<br />

Praziquantel is effective in treating cerebral cysticercosis. Active uninflamed cysts are responsive. Praziquantel<br />

will produce inflammation and edema as it kills the larva, which may temporarily result in symptoms such as<br />

headache.<br />

References:<br />

<strong>2007</strong> <strong>RITE</strong> Discussion & Reference Manual<br />

Garcia HH, Evans CA, Nash TE, et al. Current consensus guidelines for treatment <strong>of</strong> neurocysticercosis. Clin<br />

75


Microbiol Rev 2002;15(4):747-756.<br />

Question 54: Pharmacology/Chemistry - Movement Disorders<br />

Discussion:<br />

Dyskinesias are more commonly associated with levodopa than direct dopamine agonists (DDAs).<br />

Hallucinations more commnon with the DDAs. Nausea, orthostatic hypotension roughly equal. Peripheral<br />

edema is not uncommon with DDAs, but rarely if ever seen with levodopa.<br />

References:<br />

Tintner R, Jankovic J. Dopamine agonists in Parkinson's disease. Expert Opin Investig Drugs<br />

2003;12(11):1803-1820.<br />

Question 61: Pharmacology/Chemistry - Movement Disorders<br />

Discussion:<br />

Midodrine is a prodrug that is transformed in the liver to a potent agonist <strong>of</strong> alpha receptors in arteries and veins.<br />

Given its predictable absorption and peak effect, it is the sympathomimetic <strong>of</strong> choice for treatment <strong>of</strong><br />

neurogenic orthostatic hypotension. One common side effect is scalp pruritus, a manifestation <strong>of</strong> alpha<br />

adrenergic mediated piloerection. Its most potentially serious side effect is supine hypertension. Midodrine<br />

should not be administered late in the evening and patients taking the drug should avoid lying flat.<br />

References:<br />

Jankovik J, Gilden JL, Hiner BC, et al. Neurogenic orthostatic hypotension: a double-blind, placebo-controlled<br />

study with midodrine. Am J Med 1993;95:38-48.<br />

Bradley WG, Dar<strong>of</strong>f RB, Fenichel GM, et al, editors. Neurology in clinical practice. 3rd ed. New York:<br />

Butterworth-Heinemann, 1999.<br />

Question 63: Pharmacology/Chemistry - Movement Disorders<br />

Discussion:<br />

Restless legs syndrome (RLS) is clinically defined as an urge to move the legs with or without paresthesia,<br />

worsening <strong>of</strong> symptoms with rest and transient improvement with activity, and worsening <strong>of</strong> symptoms in the<br />

evening and night. It is <strong>of</strong>ten genetic but may also occur in the setting <strong>of</strong> iron deficiency, uremia, pregnancy,<br />

neuropathy, and possibly other conditions. Among the current treatment options <strong>of</strong>fered for the treatment <strong>of</strong><br />

RLS, dopaminergic agents have provided the best evidence for efficacy in symptom relief.<br />

References:<br />

Ondo WG. Restless legs syndrome. Neurol Clin 2005;23(4):1165-1185, viii.<br />

Chahine LM, Chemali ZN. Restless legs syndrome: a review. CNS Spectr 2006;11(7):511-520.<br />

Question 74: Pharmacology/Chemistry - Movement Disorders<br />

Discussion:<br />

Iron deficiency can cause (secondary) restless legs syndrome (RLS). In addition, evidence suggests that there is<br />

an abnormality in brain iron that somehow translates to a hypodopaminergic state in idiopathic RLS.<br />

References:<br />

Sun ER, Chen CA, Ho G, et al. Iron and the restless legs syndrome. Sleep 1998;21(4):371-377.<br />

Question 87: Pharmacology/Chemistry - Other Pain Syndromes<br />

Discussion:<br />

<strong>2007</strong> <strong>RITE</strong> Discussion & Reference Manual<br />

76


Type 1 (Andrade) familial amyloidosis, inherited as an autosomal dominant, is associated with amyloid<br />

deposition in peripheral nerves. It is characterized by progressive loss <strong>of</strong> pain and temperature sensation,<br />

lancinating pain, and severe generalized autonomic failure. Mutations have been demonstrated in the gene that<br />

codes for transthyretin, the protein that transports thyroxine and retinol binding protein.<br />

References:<br />

Bosch EP, Smith BE. Disorders <strong>of</strong> peripheral nerves. In: Bradley WG, Dar<strong>of</strong>f RB, Fenichel GM, et al, editors.<br />

Neurology in clinical practice. 3rd ed. Boston: Butterworth-Heinemann, 2000.<br />

Question 88: Pharmacology/Chemistry - Neurogenetics<br />

Discussion:<br />

Dopa-responsive dystonia (DRD), in classic cases, manifests in early childhood with walking problems due to<br />

dystonia <strong>of</strong> the lower limbs. The dystonia is frequently accompanied by parkinsonian features such as reduced<br />

facial expression or slowing <strong>of</strong> fine finger movements. Biochemically, the disorder is typically characterized by<br />

low levels <strong>of</strong> the neurotransmitter metabolite homovanillic acid and reduced levels <strong>of</strong> neopterin and<br />

tetrahydrobiopterin (BH4) in the cerebrospinal fluid. This is due to heterozygote mutations <strong>of</strong> the GTP<br />

cyclohydrolase I gene, which is the rate-limiting enzyme in the synthesis <strong>of</strong> BH4. BH4 is an essential c<strong>of</strong>actor<br />

for tyrosine hydroxylase (TH), the rate-limiting enzyme in the synthesis <strong>of</strong> dopamine. Reduced levels <strong>of</strong> BH4<br />

lead to the dopamine-deficit syndrome DRD because <strong>of</strong> reduced TH activity. Other genes implicated in the<br />

pathogenesis <strong>of</strong> this disorder are the TH gene itself and the parkin gene.<br />

References:<br />

Bandmann O, Wood NW. Dopa-responsive dystonia -- the story so far. Neuropediatrics 2002;33(1):1-5.<br />

Question 92: Pharmacology/Chemistry - Other Pain Syndromes<br />

Discussion:<br />

Postherpetic neuralgia is the persistence <strong>of</strong> the pain <strong>of</strong> herpes zoster virus more than 3 months after resolution<br />

<strong>of</strong> the rash and is relatively common, affecting 10% to 15% <strong>of</strong> those with herpes zoster. Zoster-associated pain<br />

is used to describe the continuum <strong>of</strong> pain from acute herpes zoster to the development <strong>of</strong> postherpetic neuralgia.<br />

The time interval used in the clinical case definition <strong>of</strong> postherpetic neuralgia varies in the literature from 1 to 6<br />

months after resolution <strong>of</strong> the rash. The incidence <strong>of</strong> postherpetic neuralgia increases with age. Administration<br />

<strong>of</strong> antiviral agents within 72 hours <strong>of</strong> the onset <strong>of</strong> herpes zoster can reduce the intensity and duration <strong>of</strong> acute<br />

illness and can prevent postherpetic neuralgia. Efforts at prevention <strong>of</strong> herpes zoster and postherpetic neuralgia<br />

are important in that 40% to 50% <strong>of</strong> those with postherpetic neuralgia do not respond to any treatment.<br />

However, tricyclic antidepressants (amitriptyline, nortriptyline, desipramine, and maprotiline), gabapentin,<br />

pregabalin, opioids, and topical lidocaine patches are effective in reducing pain and improving quality <strong>of</strong> life and<br />

should be used in the treatment <strong>of</strong> postherpetic neuralgia. Aspirin in cream is possibly effective in the relief <strong>of</strong><br />

pain in patients with postherpetic neuralgia, but the magnitude <strong>of</strong> benefit is low, as is seen with capsaicin. The<br />

effectiveness <strong>of</strong> carbamazepine, nicardipine, biperiden, chlorprothixene, ketamine, He:Ne laser irradiation,<br />

intralesional triamcinolone, cryocautery, topical piroxicam, extract <strong>of</strong> Ganoderma lucidum, dorsal root entry<br />

zone lesions, and stellate ganglion block are unproven in the treatment <strong>of</strong> postherpetic neuralgia.<br />

References:<br />

Dubinsky R, Kabbani H, El-Chami Z, et al. Practice Parameter: Treatment <strong>of</strong> postherpetic neuralgia: An<br />

evidence-based report <strong>of</strong> the Quality Standards Subcommittee <strong>of</strong> the American Academy <strong>of</strong> Neurology.<br />

Neurology 2004;63:959-965.<br />

Question 98: Pharmacology/Chemistry - Epilepsy<br />

Discussion:<br />

<strong>2007</strong> <strong>RITE</strong> Discussion & Reference Manual<br />

Oxcarbazepine is a derivative <strong>of</strong> carbamazepine and shares many similarities with carbamazepine, including its<br />

mechanism <strong>of</strong> action, ability to induce hepatic metabolism <strong>of</strong> oral contraceptives and other drugs, risk <strong>of</strong><br />

hyponatremia, and indication for treatment <strong>of</strong> partial epilepsy. Unlike carbamazepine, oxcarbazepine is reduced<br />

to 10-monohydroxy-carbamazepine and does not undergo oxidation to an epoxide. This may explain its fewer<br />

77


side effects as compared to carbamazepine.<br />

References:<br />

Schmidt D, Elger CE. What is the evidence that oxcarbazepine and carbamazepine are distinctly different<br />

antiepileptic drugs? Epilepsy Behav 2004;5(5):627-635.<br />

Question 115: Pharmacology/Chemistry - Cerebrovascular Disease<br />

Discussion:<br />

The structural basis <strong>of</strong> the blood-brain barrier (BBB) is the presence <strong>of</strong> tight junctions (zonula occludens)<br />

between capillary endothelial cells. Tight junctions consist <strong>of</strong> macromolecular complexes, including the<br />

transmembrane proteins occludin and claudin, and the associated zonula occludens (ZO) proteins and cingulin.<br />

References:<br />

Huber JD, Egleton R, Davis TP. Molecular physiology <strong>of</strong> tight junctions in the blood-brain barrier. Trends<br />

Neurosci 2001;24:719-726.<br />

Question 117: Pharmacology/Chemistry - Neuromuscular Disorders<br />

Discussion:<br />

The findings are most consistent with prop<strong>of</strong>ol infusion syndrome. Prop<strong>of</strong>ol impairs mitochondrial fatty acid<br />

oxidation and oxidative phosphorylation. Prop<strong>of</strong>ol raises levels <strong>of</strong> malonylcarnitine that inhibits CPT1, blocking<br />

long chain fatty acid transport into mitochondria. Complex II function is secondarily inhibited. Children have<br />

limited carbohydrate stores, and require larger doses <strong>of</strong> prop<strong>of</strong>ol for sedation, thus placing them at higher risk <strong>of</strong><br />

this syndrome than adults. Inadequate caloric intake before or during the infusion further increases the risk, by<br />

leading to inability to meet metabolic demands and suppress fat metabolism. Hem<strong>of</strong>iltration has been shown to<br />

correct the metabolic abnormalities in prop<strong>of</strong>ol infusion syndrome. Carbohydrate intakes <strong>of</strong> 6 mg/kg/min to 8<br />

mg/kg/min during prop<strong>of</strong>ol infusion might prevent it.<br />

References:<br />

Wolf A, Weir P, Segar P, et al. Impaired fatty acid oxidation in prop<strong>of</strong>ol infusion syndrome. Lancet<br />

2001;357:606-607.<br />

Question 121: Pharmacology/Chemistry - Other Pain Syndromes<br />

Discussion:<br />

Opioid withdrawal resembles severe influenza with the additional features <strong>of</strong> mydriasis, lacrimation, rhinorrhea,<br />

piloerection, yawning, sneezing, anorexia nausea, vomiting, and diarrhea. Delirium tremens and seizures do not<br />

occur. Classic withdrawal techniques utilize long-acting oral agents such as methadone. Studies <strong>of</strong> aggressive<br />

(rapid) withdrawal regimens used over 5 days have found that the least severe symptoms and shortest duration<br />

<strong>of</strong> withdrawal was achieved with a combination <strong>of</strong> naltrexone, clonidine, and buprenorphine.<br />

References:<br />

Kosten TR, O'Connor PG. Management <strong>of</strong> drug and alcohol withdrawal. N Engl J Med 2003;348:1786-1795.<br />

Question 131: Pharmacology/Chemistry - Neurogenetics<br />

Discussion:<br />

<strong>2007</strong> <strong>RITE</strong> Discussion & Reference Manual<br />

Autosomal dominant hypokalemic periodic paralysis typically presents in the first 3 decades with episodes <strong>of</strong><br />

weakness occurring following rest after exercise, particularly if the subject is exposed to high-carbohydrate<br />

meals, emotional stress, or cold. The respiratory and cardiac muscles are almost always spared, and attacks last<br />

for hours. Symptoms <strong>of</strong>ten resolve in later life, although patients may be left with residual weakness. Most<br />

patients have mutations in the skeletal muscle calcium channel gene, although mutations may be found in<br />

sodium and potassium channels in the remainder. Secondary hypokalemic paralysis occurs in older patients with<br />

chronic medical problems predisposing them to hypokalemia. Preventive treatment with acetazolamide is usually<br />

78


effective in abolishing or attenuating episodes, and oral potassium supplements may be helpful during acute<br />

attacks. Some patients require the use <strong>of</strong> potassium-sparing diuretics such as spironolactone or triamterene.<br />

Nephrolithiasis is a complication <strong>of</strong> long-term therapy with acetazolamide and should be screened for with<br />

annual ultrasound examinations.<br />

References:<br />

Jones HR, De Vivo DC, Daras BT, editors. Neuromuscular disorders <strong>of</strong> infancy, childhood and adolescence.<br />

Philadelphia: Butterworth-Heinemann, 2003.<br />

Question 136: Pharmacology/Chemistry - Dementia<br />

Discussion:<br />

Frontotemporal dementia (FTD) is the most common syndrome in which the focus <strong>of</strong> neurodegeneration is the<br />

frontal lobes. FTD is frequently familial. It is also <strong>of</strong>ten due to a susceptibility locus on chromosome 17q21-22.<br />

Some 17q21-22-linked families have mutations in the tau gene and most have microscopically visible aggregates<br />

<strong>of</strong> hyperphosphorylated tau. Demonstrating that mutations in tau can produce neurodegeneration will necessitate<br />

a reassessment <strong>of</strong> the role <strong>of</strong> tau in the pathogenesis <strong>of</strong> the many diseases in which tau biology is disrupted.<br />

References:<br />

Wilhelmsen KC. Frontotemporal dementia genetics. J Geriatr Psychiatry Neurol 1998; 11(2):55-60.<br />

Question 145: Pharmacology/Chemistry - Headache<br />

Discussion:<br />

Chronic paroxysmal hemicrania is considered a syndrome with two pivotal characteristics: (1) unilateral<br />

headache <strong>of</strong> moderate intensity, with episodes lasting under 45 minutes on average, occurring multiple times<br />

throughout the day, and at a 3:1 ratio female:male; and (2) absolute response to indomethacin. Patients may also<br />

complain <strong>of</strong> a continuous low-grade headache in between the more severe attacks (hemicrania continua).<br />

References:<br />

Pareja JA, Antonaci F, Vincent M. The hemicrania continua diagnosis. Cephalalgia 2001;21(10):940-946.<br />

Question 151: Pharmacology/Chemistry - Movement Disorders<br />

Discussion:<br />

Anticholinergic agents such as trihexyphenidyl have been used in the treatment <strong>of</strong> Parkinson's disease (PD) since<br />

the 19th century. Anticholinergic drugs are typically used in younger PD patients (


Question 174: Pharmacology/Chemistry - Aging, Degenerative Diseases<br />

Discussion:<br />

Midodrine is a prodrug that is converted in the liver to an alpha agonist. Its predictable absorption and<br />

pharmacokinetics makes it the drug <strong>of</strong> choice for treatment <strong>of</strong> orthostatic hypotension unresponsive to<br />

fludrocortisone.<br />

References:<br />

Riley DE. Orthostatic hypotension in multiple system atrophy. Curr Treat Options Neurol 2000;2(3):225-230.<br />

Question 181: Pharmacology/Chemistry - Sleep Disorders<br />

Discussion:<br />

Beagles with inherited narcolepsy-cataplexy were intially shown to have defective orexin receptors. In humans,<br />

where narcolepsy is associated with human leucocyte antigen (HLA) abnormalities, recent studies have shown<br />

that narcolepsy with cataplexy is usually caused (>90%) by the lack <strong>of</strong> two related brain chemicals called<br />

hypocretin-1 and hypocretin-2. The cause <strong>of</strong> narcolepsy without cataplexy is still under investigation. Over 90%<br />

<strong>of</strong> patients with narcolepsy-cataplexy carry HLA-DQB1*0602. This marker is more specific and sensitive than<br />

the old marker HLA-DR2, and so it is speculated that patients with narcolepsy-cataplexy probably have an<br />

autoimmune disorder.<br />

References:<br />

Zeitzer JM, Nishino S, Mignot E. The neurobiology <strong>of</strong> hypocretins (orexins), narcolepsy and related therapeutic<br />

interventions. Trends Pharmacol Sci 2006;27(7):368-374.<br />

Question 202: Pharmacology/Chemistry - Other Pain Syndromes<br />

Discussion:<br />

A-delta nociceptive fibers use glutamate as their fast transmitter. C-fiber slower transmitted nociception involves<br />

a number <strong>of</strong> neurotransmitters and neuromodulators, the most important <strong>of</strong> which is substance P.<br />

References:<br />

Costigan M, Scholz J, Samad T, Woolf CJ. Pain. In: Siegel GJ, Albers RW, Brady ST, Price DL, editors. Basic<br />

Neurochemistry: molecular, cellular, and medical aspects. Boston: Elsevier, 2006;927-937.<br />

Questions 209 - 213: Pharmacology/Chemistry - Neuromuscular Disorders<br />

Discussion:<br />

<strong>2007</strong> <strong>RITE</strong> Discussion & Reference Manual<br />

1. Unrecognized copper deficiency appears to be a common cause <strong>of</strong> idiopathic myelopathy in adults. The<br />

clinical picture bears striking similarities to the syndrome <strong>of</strong> subacute combined degeneration associated with<br />

vitamin B12 deficiency. Early recognition and copper supplementation may prevent neurological deterioration.<br />

2. Juvenile myoclonic epilepsy is a common type <strong>of</strong> epilepsy with onset occurring during adolescence. This is a<br />

fairly common type <strong>of</strong> epilepsy, which can develop between 8 and 26 years <strong>of</strong> age but usually starts between<br />

ages 12 and 16. Historically, the large majority <strong>of</strong> patients become seizure-free when treated with valproate. 3.<br />

Neuromyotonia is a rare condition <strong>of</strong> spontaneous and continuous muscle fiber activity <strong>of</strong> peripheral nerve<br />

origin. It represents the more severe phenotype <strong>of</strong> peripheral nerve hyperexcitability and, when acquired, is <strong>of</strong>ten<br />

associated with antibodies to voltage-gated potassium channels. There are no specific published<br />

electromyographic or clinical diagnostic criteria for this disorder. Carbamazapine, topiramate, and gabapentin<br />

have all shown efficicacy. 4. Hyperkalemic periodic paralysis involves attacks <strong>of</strong> muscle weakness or paralysis,<br />

alternating with periods <strong>of</strong> normal muscle function. Attacks usually begin in early childhood. They tend to occur<br />

while resting after exercise or exertion. Attacks may also be triggerd by cold expsoure or eating small amounts<br />

<strong>of</strong> potassium. It is not associated with low potassium in the bloodstream (serum potassium). Glucose or other<br />

carbohydrates (sugars) given during an attack may reduce the severity. Intravenous calcium or diuretics such as<br />

furosemide may need to be given to stop sudden attacks. Intravenous glucose and insulin cause potassium to<br />

move into the cell and may reduce weakness without a loss <strong>of</strong> total body potassium. A high-carbohydrate diet<br />

may be recommended. Acetazolamide, a medication that prevents attacks <strong>of</strong> familial periodic paralysis, is also<br />

80


may be recommended. Acetazolamide, a medication that prevents attacks <strong>of</strong> familial periodic paralysis, is also<br />

effective in preventing attacks <strong>of</strong> hyperkalemic periodic paralysis. Thiazide diuretics such as chlorothiazide are<br />

also effective and have fewer side effects than acetazolamide. 5. Tourette syndrome (TS) is a neurological<br />

disorder characterized by repetitive, stereotyped, involuntary movements and vocalizations called tics. Tics are<br />

classified as either simple or complex. Neuroleptics are the most consistently useful medications for tic<br />

suppression; a number are available but some are more effective than others (for example, haloperidol and<br />

pimozide). Alfa-adrenergic agents such as guanifencine and clonidine are first-choice treatments for TS; typical<br />

antipsychotics are more effective but are troublesome because <strong>of</strong> their long-term side effect pr<strong>of</strong>iles.<br />

References:<br />

Verrotti A, Manco R, di Marco G, et al. The treatment <strong>of</strong> juvenile myoclonic epilepsy. Expert Rev Neurother<br />

2006;6(6):847-854.<br />

Jurkat-Rott K, Lerche H, Lehmann-Horn F. Skeletal muscle channelopathies. J Neurol 2002;249(11):1493-1502.<br />

Maddison P. Neuromyotonia. Clin Neurophysiol 2006;117(10):2118-2127.<br />

Shavitt RG, Hounie AG, Rosario Campos MC, Miguel EC. Tourette's Syndrome. Psychiatr Clin North Am<br />

2006;29(2):471-486.<br />

Kumar N, Gross JB Jr, Ahlskog JE. Copper deficiency myelopathy produces a clinical picture like subacute<br />

combined degeneration. Neurology 2004;63(1):33-39.<br />

Question 319: Pharmacology/Chemistry - Movement Disorders<br />

Discussion:<br />

Cabergoline, pergolide, bromocriptine, and dihydroergocriptine are all ergot-derived dopamine agonists that<br />

have been associated with serosal fibrosis syndromes. Ropinirole is a nonergoline and does not have this adverse<br />

effect.<br />

References:<br />

Tintner R, Manian P, Gauthier P, Jankovic J. Pleuropulmonary fibrosis after long-term treatment with the<br />

dopamine agonist pergolide for Parkinson Disease. Arch Neurol 2005;62(8):1290-1295.<br />

Question 336: Pharmacology/Chemistry - Epilepsy<br />

Discussion:<br />

Vigabatrin enhances GABAergic transmission by inhibiting GABA transaminase, thus increasing GABA<br />

concentration at the synapse. The other agents listed inactivate sodium channels.<br />

References:<br />

Mcnamara JO. Drugs effective in the therapy <strong>of</strong> the epilepsies. In: Hardman JG, Limbierd LE, Goodman AG,<br />

editors. Goodman and Gilman's The Pharmacological Basis <strong>of</strong> Therapeutics, 10/e. New York: McGraw-Hill,<br />

2001;521-547.<br />

Question 340: Pharmacology/Chemistry - Neurogenetics<br />

Discussion:<br />

<strong>2007</strong> <strong>RITE</strong> Discussion & Reference Manual<br />

Familial periodic paralyses are typical channelopathies (ie, caused by functional disturbances <strong>of</strong> ion channel<br />

proteins). The episodes <strong>of</strong> flaccid muscle weakness observed in these disorders are due to underexcitability <strong>of</strong><br />

sarcolemma leading to a silent electromyogram and the lack <strong>of</strong> action potentials even upon electrical<br />

stimulation. Interictally, ion channel malfunction is well compensated so that special exogenous or endogenous<br />

triggers are required to produce symptoms in the patients. An especially obvious trigger is the level <strong>of</strong> serum<br />

potassium (K+), the ion responsible for resting membrane potential and degree <strong>of</strong> excitability. The clinical<br />

symptoms can be caused by mutations in genes coding for ion channels that mediate different functions for<br />

maintaining the resting potential or propagating the action potential, the basis <strong>of</strong> excitability. The phenotype is<br />

determined by the type <strong>of</strong> functional defect brought about by the mutations rather than the channel effected<br />

81


ecause the contrary phenotypes hyperkalemic periodic paralysis (HyperPP) and hypokalemic periodic paralysis<br />

(HypoPP) may be caused by point mutations in the same gene. Still, the common mechanism for inexcitability<br />

in all known episodic-weakness phenotypes is a long-lasting depolarization that inactivates sodium ion (Na+)<br />

channels, initiating the action potential.<br />

References:<br />

Jurkat-Rott K, Lerche H, Lehmann-Horn F. Skeletal muscle channelopathies. J Neurol 2002;249(11):1493-1502.<br />

Question 348: Pharmacology/Chemistry - Movement Disorders<br />

Discussion:<br />

Restless legs syndrome (RLS) is clinically defined as an urge to move the legs with or without paresthesia,<br />

worsening <strong>of</strong> symptoms with rest and transient improvement with activity, and worsening <strong>of</strong> symptoms in the<br />

evening and night. It is <strong>of</strong>ten genetic but may also occur in the setting <strong>of</strong> iron deficiency, uremia, pregnancy,<br />

neuropathy, and possibly other conditions. Effective treatments include dopaminergics and narcotics. Its<br />

underlying pathogenesis is presently unknown. Women are more affected than men and early-onset disease is<br />

associated with familial cases. The excellent response to dopaminegic drugs points to a central role <strong>of</strong> dopamine<br />

in the pathophysiology <strong>of</strong> RLS. Iron may also represent a primary factor in the development <strong>of</strong> RLS, as<br />

suggested by recent pathological and brain imaging studies. However, the way dopamine and iron, and probably<br />

other compounds, interact to generate the circadian pattern in the occurrence <strong>of</strong> RLS and periodic limb<br />

movement symptoms remains unknown. Altered circadian rhythmicity in dopamine metabolism and enhanced<br />

circadian variations in dopaminergic functions have been reported in the disorder. Dysfunction or atrophy <strong>of</strong><br />

cells from the diencephalic-spinal dopamine system has been suggested to explain the efficacy <strong>of</strong> dopaminergic<br />

drugs in relieving RLS symptoms and the circadian rhythmicity <strong>of</strong> RLS. Studies support the hypothesis that<br />

these dopaminergic neurons and spinal pathways may be more involved in the pathophysiology <strong>of</strong> RLS than the<br />

nigrostriatal system. Among the current treatment options <strong>of</strong>fered for the treatment <strong>of</strong> RLS, dopaminergic agents<br />

have provided the best evidence for efficacy in symptom relief.<br />

References:<br />

Ondo WG. Restless legs syndrome. Neurol Clin 2005;23(4):1165-1185, viii.<br />

Clemens S, Rye D, Hochman S. Restless legs syndrome: revisiting the dopamine hypothesis from the spinal<br />

cord perspective. Neurology 2006 Jul 11;67(1):125-130.<br />

Barriere G, Cazalets JR, Bioulac B, et al. The restless legs syndrome. Prog Neurobiol 2005;77(3):139-165.<br />

Question 352: Pharmacology/Chemistry - Epilepsy<br />

Discussion:<br />

Drugs that inhibit carbamazepine metabolism results in elevated levels and toxicity. These include phenytoin,<br />

cimetidine, diltiazem, erythromycin, verapamil, fluoxetine, and isoniazid. Alternately, carbamazapine can<br />

accelerate hepatic breakdown <strong>of</strong> a number <strong>of</strong> drugs, including its own metabolism. The most common<br />

interaction is with oral contraceptives, sodium valproate, ethosuximide, corticosteroids, anticoagulants,<br />

antipsychotics, cyclosporine, and methylphenidate. Meperidine and methylphenyldate can lower the seizure<br />

threshold in epilepsy patients resulting in worsening seizures independent <strong>of</strong> antiepileptic therapy.<br />

References:<br />

Ruffmann C, Bogliun G, Beghi E. Epileptogenic drugs: a systematic review. Expert Rev Neurother<br />

2006;6(4):575-589.<br />

Pauwels O. Factors contributing to carbamazepine-macrolide interactions. Pharmacol Res 2002;45(4):291-298.<br />

Question 353: Pharmacology/Chemistry - Demyelinating Disorders<br />

Discussion:<br />

<strong>2007</strong> <strong>RITE</strong> Discussion & Reference Manual<br />

Interferon-beta-1b (Betaseron, Betaferon) is a nonglycosylated recombinant human interferon-beta approved for<br />

high-frequency subcutaneous (SC) administration in the treatment <strong>of</strong> multiple sclerosis (MS). Its mechanism <strong>of</strong><br />

82


action is unknown but may involve modulation <strong>of</strong> the autoimmune pathogenic processes <strong>of</strong> MS. In a<br />

randomized, double-blind trial in patients with relapsing-remitting MS (RRMS), SC interferon-beta-1b 250<br />

micrograms (8 million International Units [MIU]) every other day reduced the annual relapse rate and increased<br />

the proportion <strong>of</strong> relapse-free patients compared with placebo. It also reduced relapse severity, hospitalizations,<br />

and disease activity assessed by MRI and increased the time to first relapse.<br />

References:<br />

Jacobs LD, Beck RW, Simon JH, et al. Intramuscular interferon beta-1a therapy initiated during a first<br />

demyelinating event in multiple sclerosis. CHAMPS Study Group. N Eng J Med 2000;343(13):898-904.<br />

Question 367: Pharmacology/Chemistry - Dementia<br />

Discussion:<br />

Loss or dysfunction <strong>of</strong> the acetylcholine projection to the cerebral cortex has been shown experimentally in<br />

humans and animals to result in cognitive disturbance. Loss <strong>of</strong> neurons from the nucleus Basalis <strong>of</strong> Meynert was<br />

shown early on to characterize patients with Alzheimer's disease. Recent studies however have shown that the<br />

depletion <strong>of</strong> cortical acetylcholine has been significantly more severe in patients with dementia with Lewy<br />

bodies that even in Alzheimer's disease. Neither fund a temporal dimension or Huntington's disease has<br />

significant loss <strong>of</strong> cortical cholinergic projections, at least in early to moderate stages. Vascular dementia shows<br />

variable loss, depending on where the vascular lesions are; in addition, many <strong>of</strong> these patients have mixed<br />

Alzheimer's/vascular pictures.<br />

References:<br />

Samuel W, Alford M, H<strong>of</strong>stetter CR, Hansen L. Dementia with Lewy bodies versus pure Alzheimer disease:<br />

differences in cognition, neuropathology, cholinergic dysfunction, and synapse density. J Neuropathol Exp<br />

Neurol 1997;56(5):499-508.<br />

Question 380: Pharmacology/Chemistry - Aging, Degenerative Diseases<br />

Discussion:<br />

Amyotrophic lateral sclerosis (ALS) is a chronic neurodegenerative disease <strong>of</strong> upper and lower motor neurons.<br />

Clinically, patients present with skeletal muscle weakness, muscle atrophy, and hyperreflexia. Symptoms<br />

progress to death in 50% <strong>of</strong> those affected within 3 to 5 years after symptom onset. Death is usually related to<br />

respiratory failure. Ten percent <strong>of</strong> cases are inherited and 20% <strong>of</strong> these are linked to mutations <strong>of</strong> the superoxide<br />

dismutase 1 gene.<br />

References:<br />

Miller RG, Rosenberg JA, Gelinas DF, et al. Practice Parameter: The care <strong>of</strong> the patient with amyotrophic lateral<br />

sclerosis (an evidence-based revew): Report <strong>of</strong> the Quality Standards Subcommittee <strong>of</strong> the American Academy<br />

<strong>of</strong> Neurology. Neurology 1999;52:1311.<br />

Question 394: Pharmacology/Chemistry - Epilepsy<br />

Discussion:<br />

<strong>2007</strong> <strong>RITE</strong> Discussion & Reference Manual<br />

Patients with newly diagnosed epilepsy who require treatment can be initiated on standard antiepileptic drugs<br />

(AEDs), such as carbamazepine, phenytoin, valproic acid, phenobarbital, or on the new AEDs, lamotrigine,<br />

gabapentin, oxcarbazepine, or topiramate. The choice <strong>of</strong> the AED should depend on the characteristics <strong>of</strong> the<br />

patient, including potential interaction with concomitant medications. Carbamazapine, topiramate, valproic acid,<br />

gabapentin, and lamotrogine have reported efficacy as monotherapy or as adjunct therapy for newly diagnosed<br />

epilepsy. In addition, studies have demonstrated that carbamazepine, valproate, and lamotrigine are also<br />

effective in the treatment <strong>of</strong> acute mania and suggest efficacy as maintenance therapy in bipolar disorder. Recent<br />

studies do not support the efficacy <strong>of</strong> topiramate as monotherapy or adjunct therapy <strong>of</strong> acute mania or mixed<br />

episodes in adults with bipolar I disorder. Topiramate has also been associated with changes in serum lithium<br />

levels and lithium toxicity and therefore would not be indicated for primary treatment <strong>of</strong> epilepsy in bipolar<br />

patients well controlled on lithium.<br />

83


References:<br />

Gajwani P, Forsth<strong>of</strong>f A, Muzina D, et al. Antiepileptic drugs in mood-disordered patients. Epilepsia<br />

2005;46(Suppl 4):38-44.<br />

Abraham G, Owen J. Topiramate can cause lithium toxicity. J Clin Psychopharmacol 2004;24(5):565-567.<br />

Vasudev K, Macritchie K, Geddes J, et al. Topiramate for acute affective episodes in bipolar disorder. Cochrane<br />

Database Syst Rev 2006 Jan 25;(1):CD003384.<br />

Question 399: Pharmacology/Chemistry - Neurogenetics<br />

Discussion:<br />

Dystrophin is a membrane-bound protein distributed along the intracellular surface <strong>of</strong> the sarcolemma and is a<br />

member <strong>of</strong> the superfamily <strong>of</strong> cytoskeletal proteins; dystrophin interacts with actin and may contribute to<br />

structural and functional stability <strong>of</strong> the plasma membrane.<br />

References:<br />

Jones HR, De Vivo DC, Darras BT, editors. Neuromuscular disorders <strong>of</strong> infancy, childhood and adolescence.<br />

Philadelphia: Butterworth-Heinemann, 2003.<br />

Question 400: Pharmacology/Chemistry - Epilepsy<br />

Discussion:<br />

Valproate can produce hair loss that rarely requires discontinuation <strong>of</strong> the drug.<br />

References:<br />

Holland KD. Efficacy, pharmacology, and adverse effects <strong>of</strong> antiepileptic drugs. Neurol Clin 2001;19:313-345.<br />

Question 407: Pharmacology/Chemistry - Headache<br />

Discussion:<br />

For migraine prevention, individual medications have been put into treatment groups based on their established<br />

clinical efficacy, significant adverse events, safety pr<strong>of</strong>ile, and clinical experience <strong>of</strong> the US Headache<br />

Consortium participants: Group 1 medications (amitryptiline, divalproex sodium, propranolol/timolol) have<br />

proven high efficacy and mild to moderate adverse events. Group 2 medications (verapamil, nonsteroidal<br />

anti-inflammatory drugs, gabapentin, vitamin B2) with lower efficacy (ie, limited number <strong>of</strong> studies, studies<br />

reporting conflicting results, efficacy suggesting only “modest” improvement) and mild to moderate adverse<br />

events. Group 3 medication (topiramate, diltiazam, antidepressants) are used based on opinion, not randomized<br />

controlled trials, and may have low to moderate or frequent or severe adverse events. Group 4 medication<br />

(methysergide) has shown proven efficacy but frequent or severe adverse events (or safety concerns) or complex<br />

management issues. Group 5 (tegretol, clonidine, clonazepam) medications are proven to have limited or no<br />

efficacy.<br />

References:<br />

Silberstein S. Practice parameter: Evidence-based guidelines for migraine headache (an evidence-based review).<br />

Report <strong>of</strong> the Quality Standards Subcommittee <strong>of</strong> the American Academy <strong>of</strong> Neurology. Neurology 2000; 55:<br />

754-763.<br />

Question 422: Pharmacology/Chemistry - Epilepsy<br />

Discussion:<br />

Mutations <strong>of</strong> the FIL-1 gene in chromosome Xq28, encoding for filamin-1, produce periventricular heterotopy.<br />

This reflects a complete failure <strong>of</strong> neuronal migration that may cause seizures. It is dominant in females and<br />

lethal in males. Filamin-1 interacts with actin and is critically involved in the early phases <strong>of</strong> migration.<br />

References:<br />

<strong>2007</strong> <strong>RITE</strong> Discussion & Reference Manual<br />

84


Gleeson JG, Walsh CA. Neuronal migration disorders: from genetic disease to developmental mechanisms.<br />

Trends Neurosci 2000; 22:402-410.<br />

Question 435: Pharmacology/Chemistry - Epilepsy<br />

Discussion:<br />

Pseudoseizures represent the opposite end <strong>of</strong> the spectrum from seizures that mimic psychiatric disorders: they<br />

are paroxysmal changes in behavior that resemble epileptic seizures but are without organic cause and expected<br />

EEG changes. Accurately distinguishing pseudoseizures from epilepsy and other illnesses is difficult because <strong>of</strong><br />

the breadth and overlap <strong>of</strong> symptoms seen in each condition and because <strong>of</strong> the frequent co-occurrence <strong>of</strong><br />

pseudoseizures and epilepsy. For the assessment <strong>of</strong> true seizure versus pseudoseizure, EEG is indicated in the<br />

peri-ictal evaluation. Emergent or urgent neuroimaging should be considered if there is a new seizure pattern or<br />

new seizure type or prolonged postictal confusion or worsening mental status. Elevated serum prolactin assay,<br />

when measured in the appropriate clinical setting at 10 to 20 minutes after a suspected event, is a useful adjunct<br />

for the differentiation <strong>of</strong> generalized tonic–clonic or complex partial seizure from psychogenic nonepileptic<br />

seizure among adults and older children. However, the use <strong>of</strong> serum prolactin assay has not been established in<br />

the evaluation <strong>of</strong> status epilepticus, repetitive seizures, and neonatal seizures.<br />

References:<br />

Chen D, So Y, Fisher R. The use <strong>of</strong> serum prolactin level in diagnosisng epileptic seizures. Report <strong>of</strong> the<br />

Therapeutics and Technology Assessment Subcommittee <strong>of</strong> the American Academy <strong>of</strong> Nuerology. Neurology<br />

2005;65:668-675.<br />

Physiology<br />

Question 24: Physiology - EMG<br />

Discussion:<br />

Common peroneal nerve divides into superficial and deep branches. Injury to the deep branch weakens the toe<br />

and foot dorsiflexors, with sensory changes over the web <strong>of</strong> the skin between the first and second toe.Lesion <strong>of</strong><br />

the superficial branch affects the everters, with sensory deficit over most <strong>of</strong> the dorsum <strong>of</strong> the foot. Injury to the<br />

femoral nerve weakens the quadriceps muscle. Injury to the obturator nerve weakens the adductors, internal and<br />

external rotators <strong>of</strong> the thigh. Injury to the tibial nerve weakens the foot inverters, dorsiflexors and intrinsic foot<br />

muscles.<br />

References:<br />

Question 28: Physiology - EEG<br />

Discussion:<br />

Although periodic waveforms can be seen in other conditions, the presence <strong>of</strong> generalized periodic 1 Hz sharp<br />

waves with progressive dementia is strongly suggestive <strong>of</strong> Creutzfeldt-Jakob disease.<br />

References:<br />

Niedermeyer E, Lopes da Silva F, editors. Electroencephalography: basic principles, clinical applications, and<br />

related fields. 4th ed. Philadelphia: Lippincott, Williams & Wilkins, 1998.<br />

Question 29: Physiology - EEG<br />

Discussion:<br />

Drowsiness in a normal adult will slow background rhythm on the EEG to theta activity and sometimes bursts <strong>of</strong><br />

generalized moderate-to-high amplitude 5 Hz to 7Hz theta can be seen. Frontocentral beta activity will increase<br />

in prominence during drowsiness (16 Hz to 20 Hz) and occasionally bursts <strong>of</strong> faster activity can be seen.<br />

References:<br />

<strong>2007</strong> <strong>RITE</strong> Discussion & Reference Manual<br />

85


Daly DD, Pedley TA. Current practice <strong>of</strong> clinical electroencephalography. 2nd ed. New York: Raven Press,<br />

1990.<br />

Question 30: Physiology - Autonomic Studies<br />

Discussion:<br />

The Valsalva maneuver evaluates the baroreflex arc and its sympathetic and parasympathetic responses. The<br />

maneuver consists <strong>of</strong> four phases. In phase 4, there is overshoot <strong>of</strong> the blood pressure due to the persistent<br />

increase in peripheral resistance, normalization <strong>of</strong> venous return, and stroke volume. The mean blood pressure<br />

can increase by more than 10 mm Hg. If the blood pressure does not increase, this indicates dysfunction <strong>of</strong> the<br />

sympathetic response.<br />

References:<br />

Hilz MJ, Dutsch M. Quantitative studies <strong>of</strong> autonomic function. Muscle Nerve 2006;33:6-20.<br />

Question 34: Physiology - EEG<br />

Discussion:<br />

The presence <strong>of</strong> periodic lateralized sharp wave discharges in a patient with a recent febrile illness and onset <strong>of</strong><br />

seizures would strongly suggest herpes simplex encephalitis.<br />

References:<br />

Niedermeyer E, Lopes da Silva F, editors. Electroencephalography: basic principles, clinical applications, and<br />

related fields. 4th ed. Philadelphia: Lippincott, Williams & Wilkins, 1998.<br />

Question 36: Physiology - EMG<br />

Discussion:<br />

Early signs <strong>of</strong> Guillain-Barre syndrome include diminished or lost muscle stretch flexes. Electrophysiological<br />

studies reveal slowing <strong>of</strong> peripheral conduction velocity as well as increased central conduction time, including<br />

prolonged F-waves and distal motor latencies. Interestingly, the disease tends to spare the sural nerve sensory<br />

action potential, <strong>of</strong>ten regarded as one <strong>of</strong> the first affected in other neuropathies. Reduction <strong>of</strong> amplitude <strong>of</strong><br />

compound muscle action potentials with distal stimulation implies a poor prognosis.<br />

References:<br />

Kimura J. Electrodiagnosis in diseases <strong>of</strong> nerve and muscle: principles and practice. 2nd ed. Philadelphia: Davis,<br />

1989;470-472.<br />

Murray NMF, Wade DT. The sural sensory action potential in Gullain-Barre syndrome. Muscle Nerve<br />

1980;3:444.<br />

Question 46: Physiology - EEG<br />

Discussion:<br />

Long-term use <strong>of</strong> chlorpromazine has been reported to lead to generalized paroxysmal bursts in EEGs.<br />

References:<br />

Engel J, Pedley TA, editors. Epilepsy - a comprehensive textbook. Philadelphia: Lippincott-Raven, 1998.<br />

Question 52: Physiology - EMG<br />

Discussion:<br />

<strong>2007</strong> <strong>RITE</strong> Discussion & Reference Manual<br />

Lesions <strong>of</strong> the upper trunk <strong>of</strong> the brachial plexus will cause weakness in proximal upper extremity with<br />

abnormal SNAPs on NCS in the radial and median nerves. The deltoid muscle is innervated by the axillary<br />

86


nerve which arises from the upper trunk/posterior cord. The biceps muscle is innervated by the<br />

musculocutaneous nerve which arises from the upper trunk/lateral cord.<br />

References:<br />

Kimura J. Electrodiagnosis in disease <strong>of</strong> nerve and muscle. 3rd ed. New York: Oxford University Press, 2001.<br />

Question 55: Physiology - EMG<br />

Discussion:<br />

Increased jitter is seen in a variety <strong>of</strong> neuromuscular disorders and is not specific to any particular disease. Jitter<br />

is not increased in metabolic myopathies and steroid myopathies.<br />

References:<br />

Stalberg E, Trontelj JV. Single fiber electromyography. 2nd ed. New York: Raven Press, 1994.<br />

Question 57: Physiology - EMG<br />

Discussion:<br />

Myotonic discharges may be present as spontaneous activity on EMG in myopathies. End plate spikes originate<br />

from the motor end plate. Fasciculations, myokymia and neuromyotonia all originate from the motor axon.<br />

References:<br />

Preston DC, Shapiro BE. Electromyography and neuromuscular disorders: clinical-electrophysiologic<br />

correlations. 2nd ed. Philadelphia: Elsevier, 2005.<br />

Question 60: Physiology - EMG<br />

Discussion:<br />

Fascioscapulohumeral dystrophy patients have weakness <strong>of</strong> the facial muscles, shoulder and upper arm muscles<br />

(sparing deltoids usually), and foot dorsiflexors, frequently with asymmetry. EMG is not specific but can <strong>of</strong>ten<br />

show fibrillation potentials and short-duration, low amplitude, polyphasic motor unit action potentials.<br />

References:<br />

Kimura J. Electrodiagnosis in disease <strong>of</strong> nerve and muscle. 3rd ed. New York: Oxford University Press, 2001.<br />

Question 76: Physiology - EMG<br />

Discussion:<br />

In a postsynaptic disorder (ie, myasthenia gravis), rapid repetitive stimulation causes no change in the<br />

compound muscle action potential. There would typically be a >10% decrement seen on slow repetitive<br />

stimulation. A significant increment on rapid repetitive stimulation is typically seen in presynaptic disorders<br />

(Lambert-Eaton myasthenic syndrome, botulism).<br />

References:<br />

Katirji B, Kaminski HJ. Electrodiagnostic approach to the patient with suspected neuromuscular junction<br />

disorder. Neurol Clin N Am 2002;20:557-586.<br />

Question 78: Physiology - EEG<br />

Discussion:<br />

Generalized beta activity (activity over 14 Hz) can be seen as a consequence <strong>of</strong> drug use, particularly<br />

benzodiazepines or barbiturates.<br />

References:<br />

<strong>2007</strong> <strong>RITE</strong> Discussion & Reference Manual<br />

87


Niedermeyer E, Lopes da Silva F, editors. Electroencephalography: basic principles, clinical applications, and<br />

related fields. 4th ed. Philadelphia: Lippincott, Williams & Wilkins, 1998.<br />

Question 82: Physiology - EMG<br />

Discussion:<br />

In compression neuropathies, sensory fibers are usually affected first with conduction changes. The median<br />

palmar sensory latency prolongation is the earliest change seen on EMG/nerve conduction studies in patients<br />

with mild carpal tunnel syndrome.<br />

References:<br />

Kimura J. Electrodiagnosis in disease <strong>of</strong> nerve and muscle. 3rd ed. New York: Oxford University Press, 2001.<br />

Question 85: Physiology - EEG<br />

Discussion:<br />

Favorable prognostic factors on EEG are variability, reactivity to external stimuli, varying sleep patterns,<br />

increase in background frequencies. Poor prognostic factors are invariant pattern, no reactivity, monorthythmic<br />

pattern, burst suppression, generalized periodic discharges, very low voltage tracing, and generalized<br />

suppression. Over 96% <strong>of</strong> the patients with poor prognostic findings on the EEG following a cardiac arrest<br />

either die within a few days after the cardiopulmonary arrest or if they survive, do so in a persistent vegetative<br />

state.<br />

References:<br />

Daube JR, editor. Clinical Neurophysiology. New York: Oxford University Press, 2002;98-101.<br />

Daube JR, editor. Clinical Neurophysiology. Continuum: Lifelong Learning in Neurol 1998:4(5);41.<br />

Question 91: Physiology - Sleep<br />

Discussion:<br />

The goal <strong>of</strong> the multiple sleep latency test is to quantitate physiological sleepiness during waking hours and to<br />

determine the occurrence <strong>of</strong> REM sleep near sleep onset. The goal <strong>of</strong> polysomnography is to quantitate the<br />

amount <strong>of</strong> time spent in various stages <strong>of</strong> sleep during the night and to document clinically relevant events such<br />

as cardiopulmonary abnormalities or sleep-related abnormal motor activity. Electro-oculogram, surface<br />

electromyography <strong>of</strong> all four limbs, and EEG recording are all used while performing polysomnography and<br />

multiple sleep latency test.<br />

References:<br />

Daube JR, editor. Clinical Neurophysiology. New York: Oxford University Press, 2002;415-417.<br />

Question 94: Physiology - EMG<br />

Discussion:<br />

Preganglionic separation <strong>of</strong> the cell bodies with lesions at the root level preserves the anatomic and physiologic<br />

integrity <strong>of</strong> the peripheral axon. Despite sensory loss, nerve stimulation elicits a normal sensory action potential.<br />

A preganglionic involvement spares the sensory nerve action potential, although degeneration <strong>of</strong> the motor<br />

axons leads to muscle atrophy and reduction in amplitude <strong>of</strong> compound muscle potentials. The deep cervical<br />

muscles receive innervation from the posterior, as opposed to anterior rami <strong>of</strong> the spinal nerves. Evidence <strong>of</strong><br />

denervation here indicates an intraforaminal lesion affecting the root or spinal nerve prior to the division into<br />

the two rami.<br />

References:<br />

<strong>2007</strong> <strong>RITE</strong> Discussion & Reference Manual<br />

Kimura, J. Electrodiagnosis in diseases <strong>of</strong> nerve and muscle: principles and practice. 2nd ed. Philadelphia:<br />

Davis, 1989;448-449.<br />

88


Question 97: Physiology - EMG<br />

Discussion:<br />

Femoral nerve innervates the hip flexors and knee extensor muscles.<br />

References:<br />

Kimura J. Electrodiagnosis in disease <strong>of</strong> nerve and muscle. 3rd ed. New York: Oxford University Press, 2001.<br />

Question 101: Physiology - EEG<br />

Discussion:<br />

In elderly patients in coma, EEG showing diffuse spike-and-wave discharges is most consistent with<br />

nonconvulsive status epilepticus. Barbiturate overdose will show excessive beta activity on EEG. Alpha coma<br />

will show diffuse nonreactive alpha rhythm. Metabolic encephalopathy usually shows diffuse slowing.<br />

Pseudoseizures would have a normal EEG pattern.<br />

References:<br />

Niedermeyer E, Lopes da Silva F, editors. Electroencephalography: basic principles, clinical applications, and<br />

related fields. 4th ed. Philadelphia: Lippincott, Williams & Wilkins, 1998.<br />

Question 103: Physiology - EEG<br />

Discussion:<br />

Alzheimer’s disease is associated with a decrease or loss <strong>of</strong> alpha and beta activity at an earlier stage than other<br />

disorders associated with dementia. Generalized periodic sharp waves and invariant alpha pattern are poor<br />

prognostic indicators on EEG and are usually seen after cardiopulmonary arrest. Triphasic waves are seen in<br />

50% <strong>of</strong> patients with hepatic coma. Excessive beta activity may been seen due to medications like<br />

benzodiazepines or barbiturates.<br />

References:<br />

Daube JR, editor. Clinical Neurophysiology. New York: Oxford University Press, 2002;159-160.<br />

Question 106: Physiology - EMG<br />

Discussion:<br />

Isolated visible muscle twitching for several years with no other concurrent symptoms is most consistent with<br />

fasciculation potentials as seen in benign fasciculation syndrome.<br />

References:<br />

Kimura J. Electrodiagnosis in disease <strong>of</strong> nerve and muscle. 3rd ed. New York: Oxford University Press, 2001.<br />

Question 107: Physiology - EMG<br />

Discussion:<br />

The H-reflex is the electrophysiological equivalent <strong>of</strong> the monosynaptic tendon stretch reflex at the ankle.<br />

References:<br />

Kimura J. Electrodiagnosis in disease <strong>of</strong> nerve and muscle. 3rd ed. New York: Oxford University Press, 2001.<br />

Question 109: Physiology - EMG<br />

Discussion:<br />

<strong>2007</strong> <strong>RITE</strong> Discussion & Reference Manual<br />

Polio is a motor neuron disease. Adults who had polio when they were younger may have large polyphasic<br />

89


Polio is a motor neuron disease. Adults who had polio when they were younger may have large polyphasic<br />

motor unit action potentials, due to reinnervation. They may also have some fibrillation and fasciculation<br />

potentials.<br />

References:<br />

Kimura J. Electrodiagnosis in disease <strong>of</strong> nerve and muscle. 3rd ed. New York: Oxford University Press, 2001.<br />

Question 119: Physiology - EMG<br />

Discussion:<br />

The patient's examination localizes to the right L5 root. Both the gluteus medius and peroneus longus muscles<br />

receive their major innervation from the L5 nerve root.<br />

References:<br />

Brown WF, Bolton CF, editors. Clinical electromyography. 2nd ed. Boston: Butterworth-Heinemann, 1993.<br />

Question 132: Physiology - EMG<br />

Discussion:<br />

Inclusion body myositis is an inflammatory myopathy with common clinical presentation <strong>of</strong> distal upper<br />

extremity and proximal lower extremity weakness. It occurs more frequently in older patients. EMG shows<br />

diffuse irritability and fibrillation potentials, as in amyotrophic lateral sclerosis, but the motor unit action<br />

potentials are small and polyphasic.<br />

References:<br />

Day JW, Ranum LPW. Myotonic Disorders in Neuromuscular Disorders. In: Katirji B, Kaminski HJ, Preston<br />

DC, et al, editors. Clinical Practice. Woburn, Mass: Butterworth-Heinemann, 2002;1078-1091.<br />

Question 148: Physiology - EMG<br />

Discussion:<br />

The short head <strong>of</strong> the biceps femoris muscle is the only muscle proximal to the knee to receive innervation via<br />

the peroneal division <strong>of</strong> the sciatic nerve.<br />

References:<br />

Kimura J. Electrodiagnosis in disease <strong>of</strong> nerve and muscle. 3rd ed. New York: Oxford University Press, 2001.<br />

Question 150: Physiology - EEG<br />

Discussion:<br />

Light sensitivity manifested by photomyoclonus induced by photic stimulation can occur with abrupt alcohol<br />

withdrawal.<br />

References:<br />

Niedermeyer E, Lopes da Silva F. Electroencephalography. 4th ed. Baltimore: Lippincott, Williams & Wilkins,<br />

1998.<br />

Question 159: Physiology - EEG<br />

Discussion:<br />

The pattern most commonly seen in children between the ages <strong>of</strong> 3 and 10 who have absence seizures is<br />

generalized 3 Hz spike-and-wave discharges.<br />

References:<br />

<strong>2007</strong> <strong>RITE</strong> Discussion & Reference Manual<br />

90


Pedley TA, Mendiratta A, Walczak TS. Seizures and epilepsy. In: Ebersole JS, Pedley TA, editors. Current<br />

Practice <strong>of</strong> Clinical Electroencephalography. 3rd ed. Philadelphia: Lippincott, Williams & Wilkins, 2003;<br />

506-587.<br />

Question 160: Physiology - EMG<br />

Discussion:<br />

First dorsal interosseus muscle is innervated by the ulnar nerve and C8 and T1 nerve roots.<br />

References:<br />

Kimura J. Electrodiagnosis in disease <strong>of</strong> nerve and muscle. 3rd ed. New York: Oxford University Press, 2001.<br />

Question 162: Physiology - EMG<br />

Discussion:<br />

Hereditary neuropathy with liability to pressure palsies may present in childhood. Other family members may or<br />

may not be symptomatic. Electrophysiologic studies reveal conduction blocks at sites <strong>of</strong> pressure and mild<br />

diffuse slowing.<br />

References:<br />

Jones HR, Bolton CF, Harper CM. Pediatric clinical electromyography. Philadelphia: Lippincott, Wiliams &<br />

Wilkins, 1996.<br />

Question 165: Physiology - EEG<br />

Discussion:<br />

Lennox-Gastaut syndrome is characterized clinically by frequent generalized seizures <strong>of</strong> mixed type in<br />

association with mental retardation and a slow spike-and-wave EEG pattern with increasing disorganization<br />

during sleep.<br />

References:<br />

Niedermeyer E, Lopes da Silva F. Electroencephalography. 4th ed. Baltimore: Lippincott, Williams & Wilkins,<br />

1998.<br />

Question 168: Physiology - Basic Physiology<br />

Discussion:<br />

Depolarization <strong>of</strong> action potentials causes an initial negative departure from baseline due to rapid opening <strong>of</strong><br />

voltage-sensitive sodium channels. Closing <strong>of</strong> these channels due to an intrinsic inactivation time constant as<br />

well as slow opening <strong>of</strong> voltage-sensitive potassium channels results in repolarization <strong>of</strong> the wave.<br />

References:<br />

Daube JR, editor. Clinical neurophysiology. Philadelphia: FA Davis, 1996;81-84.<br />

Question 186: Physiology - EMG<br />

Discussion:<br />

The extensor indicis proprius (EIP) is innervated by the posterior interosseous nerve. This is derived from the<br />

posterior cord, lower trunk <strong>of</strong> the brachial plexus. The ulnar and median innervated muscles <strong>of</strong> the hand are<br />

supplied by the lower trunk, medial cord <strong>of</strong> the brachial plexus. Thus, if the EIP is involved, then the medial<br />

cord could not be the primary site <strong>of</strong> the lesion.<br />

References:<br />

<strong>2007</strong> <strong>RITE</strong> Discussion & Reference Manual<br />

Preston DC, Shapiro BE. Electromyography and neuromuscular disorders: clinical-electrophysiologic<br />

91


correlations. 2nd ed. Philadelphia: Elsevier, 2005; 59-64.<br />

Question 193: Physiology - EEG<br />

Discussion:<br />

Alpha rhythm is normal background rhythm seen in posterior head regions in adults. It ranges from 8 Hz to 13<br />

Hz.<br />

References:<br />

Daly DD, Pedley TA. Current practice <strong>of</strong> clinical electroencephalography. 2nd ed. New York: Raven Press,<br />

1990.<br />

Question 199: Physiology - EEG<br />

Discussion:<br />

Vertex or V-waves are high-voltage sharp-contoured waveforms that can occur with phase reversals on a bipolar<br />

montage over the central areas.<br />

References:<br />

Klass DW, Westmoreland BF. Electroencephalography: general principles and adult electroencephalograms. In:<br />

Daube JR, editor. Clinical Neurophysiology. Philadelphia: FA Davis, 1996.<br />

Question 203: Physiology - EEG<br />

Discussion:<br />

Photic stimulation is performed to assess for EEG evidence <strong>of</strong> susceptibility to photosensitive seizures, which is<br />

manifested as an epileptiform response, and for the occipital driving response.<br />

References:<br />

Fisch BJ, So EL. Activation methods. In: Ebersole JS, Pedley TA, editors. Current practice <strong>of</strong> clinical<br />

electroencephalography. 3rd ed. Philadelphia: Lippincott, Williams & Wilkins, 2003;246-270.<br />

Question 234: Physiology - EEG<br />

Discussion:<br />

Focal polymorphic delta is seen with hemispheric lesions such as infarct or tumor and is nonspecific as to<br />

etiology. It generally is seen with processes involving white matter. Subdural hematoma will typically produce<br />

attenuation <strong>of</strong> EEG amplitude in the region <strong>of</strong> the lesion.<br />

References:<br />

Klass DW, Westmoreland BF. Electroencephalography: general principles and adult electroencephalograms. In:<br />

Daube JR, editor. Clinical neurophysiology. Philadelphia: FA Davis, 1996.<br />

Question 257: Physiology - EEG<br />

Discussion:<br />

The EEG shows typical centrotemporal spikes, <strong>of</strong>ten associated with benign rolandic epilepsy <strong>of</strong> childhood,<br />

which is manifested by clonic movements <strong>of</strong> the face and hand that <strong>of</strong>ten progress to a more generalized seizure.<br />

References:<br />

Niedermeyer E, Lopes da Silva F. Electroencephalography. 4th ed. Baltimore: Lippincott, Williams & Wilkins,<br />

1998.<br />

Question 266: Physiology - EEG<br />

<strong>2007</strong> <strong>RITE</strong> Discussion & Reference Manual<br />

92


Discussion:<br />

Anterior eyeblinks (first second <strong>of</strong> the page), facial muscle EMG artifact (throughout the page), and an occipital<br />

dominant alpha rhythm (throughout the page) are characteristic <strong>of</strong> wakefulness.<br />

References:<br />

Blume WT, Kaibara M. Atlas <strong>of</strong> adult electroencephalography. New York: Raven Press, 1995.<br />

Question 267: Physiology - EEG<br />

Discussion:<br />

An EEG showing triphasic waves is consistent with a metabolic encephalopathy, most commonly hepatic<br />

encephalopathy.<br />

References:<br />

Niedermeyer E, Lopes da Silva F, editors. Electroencephalography: basic principles, clinical applications, and<br />

related fields. 4th ed. Philadelphia: Lippincott, Williams & Wilkins, 1998.<br />

Question 295: Physiology - EEG<br />

Discussion:<br />

The EEG was recorded in drowsiness and shows positive sharp waves in occipital leads. These are positive<br />

occipital sharp transients and are normal phenomenon in sleep.<br />

References:<br />

Daly DD, Pedley TA. Current practice <strong>of</strong> clinical electroencephalography. 2nd ed. New York: Raven Press,<br />

1990.<br />

Question 304: Physiology - EEG<br />

Discussion:<br />

Subacute sclerosing panencephalitis is associated with periodic long-interval diffuse discharges in the EEG that<br />

recur every 4 to 15 seconds.<br />

References:<br />

Klass DW, Westmoreland BF. Electroencephalography: principles and adult electroencephalograms. In: Daube<br />

JR, editor. Clinical neurophysiology. Philadelphia: FA Davis, 1996.<br />

Question 306: Physiology - EEG<br />

Discussion:<br />

This EEG shows generalized polyspikes and spike-wave discharges, the EEG correlate <strong>of</strong> primary generalized<br />

epilepsy syndromes, which are variously characterized by generalized tonic-clonic seizures, generalized<br />

myoclonic seizures, or absence seizures.<br />

References:<br />

Pedley TA, Mendiratta A, Walczak TS. Seizures and epilepsy. In: Ebersole JS, Pedley TA, editors. Current<br />

Practice <strong>of</strong> Clinical Electroencephalography. 3rd ed. Philadelphia: Lippincott, Williams & Wilkins, 2003;<br />

506-587.<br />

Question 311: Physiology - Sleep<br />

Discussion:<br />

<strong>2007</strong> <strong>RITE</strong> Discussion & Reference Manual<br />

Stage REM is identified by relatively mixed frequency EEG and the presence <strong>of</strong> rapid eye movements. In some<br />

93


subjects, a characteristic sawtooth pattern is intermittently observed in the EEG. EMG drops to the lowest level<br />

<strong>of</strong> recording.<br />

References:<br />

Rechtschaffen A, Kales A. Manual <strong>of</strong> standardized terminology, techniques and scoring systems for sleep stages<br />

<strong>of</strong> human subjects. Los Angeles: UCLA Brain Information Service/Brain Research Institute, 1968.<br />

Question 312: Physiology - Sleep<br />

Discussion:<br />

The minimum duration required for a respiratory event to be called as obstructive apnea or hypopnea is 10<br />

seconds in an adult.<br />

References:<br />

Pack AI. Advances in sleep-disordered breathing. Am J Respir Crit Care Med 2006;173:7-15.<br />

Chesson A, Ferber R, Fry J, et al. Practice parameters for the indications for polysomnography and related<br />

procedures. Sleep 1997;20:406.<br />

Question 313: Physiology - Sleep<br />

Discussion:<br />

While managing a patient with obstructive sleep apnea, current symptoms, especially daytime sleepiness, AHI,<br />

and co-morbid cardio/cerebro-vascular, metabolic, and pulmonary disorders should be taken into consideration.<br />

As indicated by the AHI, he has moderate obstructive sleep apnea (OSA), co-morbid poorly controlled<br />

hypertension and diabetes with significant daytime sleepiness. Therefore, his OSA should be aggressively<br />

managed to decrease the daytime sleepiness, and possibly better control the hypertension and diabetes. Among<br />

the options listed, CPAP if used appropriately, is 100 % effective and thus would be the best option. Surgery<br />

has a success rate <strong>of</strong> ~ 50% with a high relapse rate. A dental appliance is good for only mild obstructive sleep<br />

apnea. Weight loss should be encouraged in all sleep apnea patients with BMI > 25; however, it is not helpful by<br />

itself. Hypnotic use by itself in moderate to severe obstructive sleep apnea might worsen sleep apnea and thus is<br />

not the best option.<br />

References:<br />

Pack AI. Advances in sleep-disordered breathing. Am J Respir Crit Care Med 2006;173:7-15.<br />

Chesson A, Ferber R, Fry J, et al. Practice parameters for the indications for polysomnography and related<br />

procedures. Sleep 1997;20:406.<br />

Question 321: Physiology - EEG<br />

Discussion:<br />

Frontal intermittent rhythmic delta activity can be seen with a variety <strong>of</strong> lesions including posterior fossa lesions,<br />

encephalopathy, intracranial lesions, and increased intraventricular pressure. REM sleep is associated with a<br />

low-voltage desynchronized EEG.<br />

References:<br />

Niedermeyer E, Lopes da Silva F. Electroencephalography. 3rd ed. Baltimore: Williams & Wilkins, 1993.<br />

Question 322: Physiology - EEG<br />

Discussion:<br />

Electrocerebral inactivity is defined as no EEG activity over 2 microvolts.<br />

References:<br />

<strong>2007</strong> <strong>RITE</strong> Discussion & Reference Manual<br />

Daube JR, editor. Clinical neurophysiology. Philadelphia: FA Davis, 1996;81-84.<br />

94


Daube JR, editor. Clinical neurophysiology. Philadelphia: FA Davis, 1996;81-84.<br />

Question 325: Physiology - EEG<br />

Discussion:<br />

Normal variants during drowsiness and sleep include 14 Hz and 6 Hz positive waves, small sharp spikes, wicket<br />

spikes, 6 Hz spike-and-wave, and rhythmic temporal theta. Subclinical rhythmic electrographic discharges <strong>of</strong><br />

adults (SREDA) is a normal variant during wakefulness.<br />

References:<br />

Daube JR, editor. Clinical neurophysiology. Philadelphia: FA Davis, 1996;81-84.<br />

Question 326: Physiology - Basic Physiology<br />

Discussion:<br />

Tetanus toxin targets synaptobrevins in the presynaptic nerve terminal, blocking docking <strong>of</strong> the vesicle and<br />

release <strong>of</strong> neurotransmitters. Alpha-bungarotoxin and curare bind to the nicotinic AChR in muscle. Brevetoxin<br />

primarily causes prolonged sodium channel opening. Tetrodotoxin blocks sodium channels.<br />

References:<br />

Day JW, Ranum LPW. Myotonic Disorders in Neuromuscular Disorders. In: Katirji B, Kaminski HJ, Preston<br />

DC, et al, editors. Clinical Practice. Woburn, Mass: Butterworth-Heinemann, 2002;1078-1091.<br />

Kandel ER, Schwartz JH, Jessel TM. Principles <strong>of</strong> neural science. 4th ed. New York: McGraw-Hill, 2000.<br />

Question 327: Physiology - EMG<br />

Discussion:<br />

Both Guillain-Barré syndrome (GBS) and chronic inflammatory demyelinating polyneuropathy (CIDP) are<br />

acquired demyelinating autoimmune neuropathies. The CSF and electrodiagnostic findings are quite similar, and<br />

cannot distinguish one from the other. The interval between onset <strong>of</strong> symptoms and disease plateau is the main<br />

distinguishing feature, with the vast majority <strong>of</strong> GBS patients reaching a plateau within four weeks <strong>of</strong> disease<br />

onset.<br />

References:<br />

Kimura J. Electrodiagnosis in disease <strong>of</strong> nerve and muscle. 3rd ed. New York: Oxford University Press, 2001.<br />

Question 344: Physiology - EMG<br />

Discussion:<br />

An incremental response to tetanic stimulation indicates a presynaptic defect at the neuromuscular junction. In<br />

an infant, the most likely reason for developing this is from intestinal botulism.<br />

References:<br />

Jones HR, Bolton CF, Harper CM. Pediatric clinical electromyography. Philadelphia: Lippincott, Wiliams &<br />

Wilkins, 1996.<br />

Question 355: Physiology - EMG<br />

Discussion:<br />

The blink reflex is elicited by stimulation <strong>of</strong> the supraorbital nerve. Stimulation elicits an R1 response on the<br />

side <strong>of</strong> stimulation and R2 responses bilaterally. In a unilateral trigeminal lesion, the ipsilateral R1 and both R2<br />

latencies will be prolonged; stimulation on the normal side will show normal latencies <strong>of</strong> all responses.<br />

References:<br />

<strong>2007</strong> <strong>RITE</strong> Discussion & Reference Manual<br />

95


Preston DC, Shapiro BE. Electromyography and neuromuscular disorders: clinical-electrophysiologic<br />

correlations. 2nd ed. Philadelphia: Elsevier, 2005; 59-64.<br />

Question 360: Physiology - Basic Physiology<br />

Discussion:<br />

Ethosuximide, used to treat absence seizures, binds to calcium channels in the thalamus.<br />

References:<br />

Harden CL. New antiepileptic drugs. Neurology 1994;44:787-795.<br />

Question 362: Physiology - EMG<br />

Discussion:<br />

Nerve impulses conduct faster at a higher body temperature. Lower temperature augments the amplitude <strong>of</strong> the<br />

nerve and the muscle potential. Both motor and sensory fibers conduct substantially more slowly in the legs than<br />

in the arms. The nerve impulse propagates faster in the proximal than in the distal nerve segments. A reduction<br />

in the mean conduction rate <strong>of</strong> about 10% is seen at age <strong>of</strong> 60 and above. Aging also causes a diminution in<br />

amplitude and changes in the shape <strong>of</strong> the evoked potential. Nerve conduction velocities are higher in large<br />

myelinated fibers.<br />

References:<br />

Kimura, J. Electrodiagnosis in diseases <strong>of</strong> nerve and muscle: principles and practice. 2nd ed. Philadelphia:<br />

Davis, 1989;94-97.<br />

Question 377: Physiology - EEG<br />

Discussion:<br />

The electrical fields that generate EEG signals are the result <strong>of</strong> inhibitory and excititory postsynaptic potentials<br />

(IPSPs and EPSPs) on the apical dendrites <strong>of</strong> cortical neurons. Pyramidal neurons contribute the plurality <strong>of</strong> the<br />

signal.<br />

References:<br />

Swanson TH. Basic cellular and synaptic mechanisms underlying the electroencephalogram. In: Levin KH,<br />

Lüders HO, editors. Comprehensive clinical neurophysiology. Philadelphia: WB Saunders, 2000;349-385.<br />

Question 379: Physiology - EMG<br />

Discussion:<br />

Myotonic dystrophy is a multisystem disease that can cause cardiac conduction defects, early cataracts, frontal<br />

balding, ptosis, facial weakness, hip girdle and finger flexor weakness as well as hypogammaglobulinemia,<br />

endocrine and CNS abnormalities. Electrophysiologic studies demonstrate myotonic discharges, even when there<br />

is minimal to no clinical myotonia elicitable.<br />

References:<br />

Day JW, Ranum LPW. Myotonic Disorders in Neuromuscular Disorders. In: Katirji B, Kaminski HJ, Preston<br />

DC, et al, editors. Clinical Practice. Woburn, Mass: Butterworth-Heinemann, 2002;1078-1091.<br />

Question 388: Physiology - Evoked Potentials<br />

Discussion:<br />

<strong>2007</strong> <strong>RITE</strong> Discussion & Reference Manual<br />

A reduced P100 amplitude <strong>of</strong> a pattern reversal evoked potential on one side is most likely due to decreased<br />

visual acuity in that eye.<br />

96


References:<br />

Chiappa KH. Evoked potentials in clinical medicine. 3rd ed. New York: Lippincott-Raven, 1997.<br />

Question 389: Physiology - EEG<br />

Discussion:<br />

The EEG record is generated by recording electrical potential differences between pairs <strong>of</strong> electrodes. The<br />

universal convention is that negative potential differences are represented by a deflection above the baseline, ie,<br />

up.<br />

References:<br />

Hamer HM, Lüders HO. Electrode montages and localization potentials in clinical electroencephalography. In:<br />

Levin KH, Lüders HO, editors. Comprehensive clinical neurophysiology. Philadelphia: WB Saunders,<br />

2000;358-386.<br />

Question 391: Physiology - EMG<br />

Discussion:<br />

In Bell’s palsy, facial nerve conduction studies assess the facial nerve distal to the stylomastoid foramen only.<br />

Blink reflexes allow assessment <strong>of</strong> the nerve proximal to the stylomastoid foramen, including the intraosseus<br />

portion.<br />

References:<br />

Kimura J. Electrodiagnosis in disease <strong>of</strong> nerve and muscle. 3rd ed. New York: Oxford University Press, 2001.<br />

Question 404: Physiology - Evoked Potentials<br />

Discussion:<br />

In upper extremity somatosensory evoked potential (SEP), after stimulation <strong>of</strong> the median or ulnar nerve in the<br />

wrist, activity can be recorded at the elbow, erbs point cervical spine, and scalp. The N5 potential is recorded in<br />

the median or ulnar nerve. The N9 potential is recorded at the Erb’s point. The N11 potential is recorded at the<br />

dorsal root entry zone. The N13 potential is created by dorsal column <strong>of</strong> the cervical cord and N14 potential<br />

represents activity in the nucleus cuneatus or medial lemniscus at the cervicomedullary junction. The activity<br />

recorded over the frontal lobe is seen by the N30 peak.<br />

References:<br />

Daube JR, editor. Clinical Neurophysiology. New York: Oxford University Press, 2002;159-160.<br />

Question 405: Physiology - EMG<br />

Discussion:<br />

Damage to the long thoracic nerve can occur with neck surgical procedures, resulting in scapular winging due to<br />

weakness <strong>of</strong> the serratus anterior muscle.<br />

References:<br />

Kimura J. Electrodiagnosis in disease <strong>of</strong> nerve and muscle. 3rd ed. New York: Oxford University Press, 2001.<br />

Question 416: Physiology - Evoked Potentials<br />

Discussion:<br />

<strong>2007</strong> <strong>RITE</strong> Discussion & Reference Manual<br />

Brainstem auditory evoked potentials (BAEPs) have five measurable waves. Presumed anatomical location for<br />

the source <strong>of</strong> these waves are as follows: wave I - distal action potential <strong>of</strong> CN VIII; wave II - ipsilateral<br />

proximal CN VIII or cochlear nucleus; wave III - ipsilateral superior olivary nucleus; wave IV - nucleus or axon<br />

<strong>of</strong> lateral lemniscus; wave V - inferior colliculus; wave VI - medial geniculate body; wave VII - thalamocortical<br />

97


pathways. In brain death, peak I may still be present.<br />

References:<br />

Daube JR, editor. Clinical neurophysiology. Philadelphia: FA Davis, 1996.<br />

Question 420: Physiology - Autonomic Studies<br />

Discussion:<br />

Painful neuropathies and significant reflex sympathetic dystrophy can both cause persistent or “hung up” sweat<br />

responses with the quantitative sweat testing (QSART).<br />

References:<br />

Daube JR, editor. Clinical neurophysiology. Philadelphia: FA Davis, 1996;81-84.<br />

Question 444: Physiology - EEG<br />

Discussion:<br />

In hepatic coma, the EEG <strong>of</strong>ten shows a triphasic wave pattern consisting <strong>of</strong> medium to high-voltage broad<br />

triphasic waves that occur rhythmically and bilaterally synchronous and symmetrical fashion over the two<br />

hemispheres. They have a fronto-occipital or occipit<strong>of</strong>rontal time lag [2]. The triphasic waves usually have a<br />

frontal predominance[3].<br />

References:<br />

<strong>2007</strong> <strong>RITE</strong> Discussion & Reference Manual<br />

Kass D, Westmoreland B. Electroencephalography: general principles and adult electroncephalograms In:<br />

Daube JR, editor. Clinical neurophysiology. New York: Oxford University Press, 2000;98.<br />

Bickfor RG, Butt HR. Hepatic coma: the electroencephalographic pattern. J Clin Invest 1955;34:790-799.<br />

Fisch BJ, Klass DW. The diagnostic specificity <strong>of</strong> triphasic wave patterns. Electroencephalogr Clin<br />

Neurophysiol 1988;70:1-8.<br />

98

Hooray! Your file is uploaded and ready to be published.

Saved successfully!

Ooh no, something went wrong!